Sunteți pe pagina 1din 428

1

Table of Contents
Chapter 1 .............................................................................................................................. 9
1.1 Numbers ................................................................................................................................... 9
1.1.1 Problem Set 1.1............................................................................................................... 13
1.2 Squares, square roots, cubes and cube roots ....................................................................... 14
1.2.1 Problem Set 1.2................................................................................................................ 20
1.3 Directed numbers ................................................................................................................... 20
1.3.1 Problem Set 1.3 ............................................................................................................... 24
1.4 Vulgar and decimal fractions and percentages ...................................................................... 25
1.4.1 Problem Set 1.4................................................................................................................ 43
1.5 Ordering .................................................................................................................................. 44
Ascending Order ........................................................................................................................ 44
Descending Order ...................................................................................................................... 45
1.5.1 Problem Set 1.5............................................................................................................... 46
1.6 Standard form ......................................................................................................................... 47
1.6.1 Problem Set 1.6 ............................................................................................................... 49
1.7 Four basic operations............................................................................................................ 49
1.7.1 Problem Set 1.7................................................................................................................ 54
1.8 Estimation .............................................................................................................................. 54
1.8.1 Problem Set 1.8................................................................................................................ 63
1.9 Limits of Accuracy .................................................................................................................. 63
Limits of accuracy - discrete measures ......................................................................................... 63
1.9.1 Problem Set 1.9 ............................................................................................................... 65
1.10 Ratio, proportion and rate .................................................................................................. 66
Ratio ....................................................................................................................................... 67
Terms of the ratio................................................................................................................... 67
Properties of ratio ................................................................................................................. 67
Ratio in the simplest form ...................................................................................................... 68
Simplification of ratio............................................................................................................. 70
2

Comparison of ratios .............................................................................................................. 71


Dividing the given quantity in the given ratio ........................................................................ 72
Proportion..............................................................................................................................73
Continued Proportion ............................................................................................................ 73
Direct Variation ...................................................................................................................... 81
Inverse Variation .................................................................................................................... 81
1.10.1 Problem Set 1.10............................................................................................................ 86
1.11 Use of an electronic calculator............................................................................................. 86
1.11.1 Problem Set 1.11............................................................................................................ 87
1.12 Money and Finance .............................................................................................................. 87
List Price: ................................................................................................................................ 94
Discount ................................................................................................................................. 94
Selling price ............................................................................................................................ 94
Successive Discounts.............................................................................................................. 97
1.12.1 Problem Set 1.12........................................................................................................... 102
Chapter 2 .......................................................................................................................... 103
2.1 SI units of measures.............................................................................................................. 103
2.1.1 SI units ............................................................................................................................ 103
2.1.2 Time Format ................................................................................................................... 108
2.1.3 Problem Set 2.1............................................................................................................... 112
Chapter 3 .......................................................................................................................... 114
3.1 Perimeters, areas and volumes............................................................................................ 114
3.1.1 Basic Geometrical Shapes and Formulae ...................................................................... 114
Surface Area of a Cube ......................................................................................................... 136
3.1.2 Problem Set 3.1.............................................................................................................. 150
Chapter 4 .......................................................................................................................... 151
4.1 Geometrical Terms and Relationships ................................................................................. 151
3

4.1.1 Types of angles ............................................................................................................... 151


4.1.2 Problem Set 4.1.............................................................................................................. 166
4.2 Geometrical Constructions ................................................................................................... 167
Set Square ................................................................................................................................ 169
Parallel Lines ........................................................................................................................ 169
Drawing Parallel Lines ......................................................................................................... 170
Perpendicular Lines ................................................................................................................. 171
Drawing Perpendicular Lines ............................................................................................... 172
4.2.1 Problem Set 4.2.............................................................................................................. 173
4.3 Symmetry .............................................................................................................................. 174
4.3.1 Problem Set 4.3.............................................................................................................. 178
4.4 Angle Properties ................................................................................................................... 181
4.5 Locus ..................................................................................................................................... 202
Problem Set 4.5........................................................................................................................ 208
Chapter 5 .......................................................................................................................... 209
5.1 Algebraic Representation and Formulae ............................................................................. 209
5.1.1 Algebra ........................................................................................................................... 209
5.1.2 Four Basic Operations ........................................................................................................ 210
5.1.3 Use of Brackets ............................................................................................................... 214
5.1.4 Evaluation of Algebraic Expressions ..................................................................................215
5.1.5 Problem Set 5.1............................................................................................................... 217
5.1.6 Formulae ............................................................................................................................ 217
5.2 Algebraic Manipulation......................................................................................................... 223
5.2.1 Algebraic Expressions..................................................................................................... 224
5.2.2 Factors ................................................................................................................................ 224
Common Factors...................................................................................................................... 225
Difference of Two Squares ...................................................................................................... 225
4

5.2.3 Problem Set 5.2............................................................................................................... 228


5.3 Polynomials ........................................................................................................................... 228
What is an algebraic expression?........................................................................................ 229
Classification of algebraic expressions ................................................................................ 231
Column wise multiplication ................................................................................................. 251
IV. Multiplication by Polynomial.......................................................................................... 252
I. Division of a Monomial by a Monomial .......................................................................... 253
II. Division of a Polynomial by a Monomial......................................................................... 255
III. Division of a Polynomial by a Polynomial ....................................................................... 256
5.3.1 Problem Set 5.3.............................................................................................................. 263
5.4 Equations and Inequalities.................................................................................................... 263
5.4.1 Problem Set 5.4.............................................................................................................. 276
5.5 Sequences ............................................................................................................................. 276
5.5.1 Problem Set 5.5.............................................................................................................. 295
5.6 Indices ................................................................................................................................... 295
5.6.1 Fractions in Power........................................................................................................... 297
5.6.2 Problem Set 5.6 .............................................................................................................. 299
5.7 Logarithms ............................................................................................................................ 300
5.7.1 Log Rules ......................................................................................................................... 300
5.7.2 Problem Set 5.7............................................................................................................... 314
Chapter 6 .......................................................................................................................... 315
6.1 Graphs in practical situations............................................................................................... 315
6.1.1 Graph .............................................................................................................................. 315
6.1.2 Speed-Time Graph.......................................................................................................... 317
6.1.3 Problem Set 6.1.............................................................................................................. 318
6.2 Linear Programming ............................................................................................................. 319
6.2.1 Problem Set 6.2.............................................................................................................. 323
5

6.3 Functions .............................................................................................................................. 323


6.3.1 Problem Set 6.3.............................................................................................................. 326
6.4 Graphs of functions .............................................................................................................. 327
6.4.1 Drawing Straight Line graphs ......................................................................................... 327
6.4.2 The slope-intercept form of straight line equation ....................................................... 328
6.4.3 Problem Set 6.4.............................................................................................................. 334
Chapter 7 .......................................................................................................................... 335
7.1 Graphs and Cartesian Coordinates ...................................................................................... 335
7.1.1 Equations of a Straight Line ........................................................................................... 338
7.1.2 Problem Set 7.1.............................................................................................................. 339
Chapter 8 .......................................................................................................................... 340
8.1 Bearings and the trigonometric ratios ................................................................................. 340
8.1.1 Problem Set 8.1............................................................................................................... 360
Chapter 9 .......................................................................................................................... 363
9.1 Vectors .................................................................................................................................. 363
9.1.1 Geometric Interpretation of vectors.............................................................................. 363
9.1.2 Properties of vectors...................................................................................................... 369
9.1.3 Problem Set 9.1.............................................................................................................. 369
9.2 Transformations.................................................................................................................... 369
9.2.1 Problem Set 9.2............................................................................................................... 377
Chapter 10......................................................................................................................... 379
10.1 Statistics .............................................................................................................................. 379
10.1.1 Collection of data ......................................................................................................... 396
10.1.2 Summarizing and Presentation of data. ...................................................................... 397
10.1.2.1 Scatter Diagram.......................................................................................................397
10.1.2.2 Dot Diagram........................................................................................................... 397
6

10.1.2.3 Time Series Plot...................................................................................................... 398


10.1.2.4 Stem-Leaf Plot........................................................................................................ 398
10.1.3 Problem Set 10.1.......................................................................................................... 399
10.2 Probability ........................................................................................................................... 400
10.2.1 Random Variable.......................................................................................................... 400
10.2.2 Probability of single events ......................................................................................... 401
10.2.3 The probability scale..................................................................................................... 402
10.2.4 Rule of Subtraction ....................................................................................................... 403
10.2.5 Rule of Multiplication .................................................................................................... 404
10.2.6 Rule of Addition ............................................................................................................ 404
10.2.7 Relative Frequency ....................................................................................................... 405
10.2.8 Tree diagrams ............................................................................................................... 407
10.2.9 Probability properties ................................................................................................... 411
10.3 Problem Set 10.2.............................................................................................................. 411
Chapter 11 ......................................................................................................................412
11.1 Matrices .............................................................................................................................. 412
11.1.1 Addition of Two Matrices................................................................................................ 413
11.1.2 Multiplication of Matrices............................................................................................ 414
11.1.2.1 Multiplying 2 X 2 Matrices .......................................................................................... 414
11.1.2.2 Scalar Multiplication .............................................................................................. 415
11.1.2.3 Zero Matrix ............................................................................................................ 415
11.1.2.4 Identity Matrix...................................................................................................... 416
11.1.3 Non-Singular Matrix..................................................................................................417
11.1.3.1 Inverse of a Matrix .................................................................................................... 417
11.1.3.2 Simultaneous Equations............................................................................................... 418
11.1.3.2 Problem Set 11.1 ......................................................................................................... 420
7

Chapter 12......................................................................................................................... 422


12.1 Set Language and Notations ......................................................................................... 422
12.1.1 Set builder notations ................................................................................................... 423
12.1.2 Problem Set 12.1 ........................................................................................................... 427

WISE WORDS BY GREAT MATHEMATICIANS

“Pure mathematics is, in its way, the poetry of logical ideas.”- Albert Einstein

“Mathematics is the most beautiful and most powerful creation of the human spirit.”- Stefan
Banach

“We will always have STEM (Science, Technology, Engineering, and Mathematics) with us. Some
things will drop out of public eye and go away, but there will always be science, engineering and
technology. And there will always be mathematics.” – Katherine Johnson

“Without mathematics, there’s nothing you can do. Everything around you is mathematics.
Everything around you is numbers.”- Shakuntala Devi

“Mathematics knows no races or geographic; for mathematics, the cultural world is one
country.”- David Hilbert

“I’ve always enjoyed mathematics. It is the most precise and concise way of expressing an idea.”-
N.R Narayana

“Actually, everything that can be known has a number; for it is impossible to grasp anything with
the mind or to recognize it without this.”-Philolaus

“In mathematics, the art of proposing a question must be held of higher value than solving it.”-
Georg Cantor

“Nature is written in mathematical language.”-Galileo Galilei


8
9

Chapter 1
1.1 Numbers
General Objectives: recognize and use different types of numbers
Specific Objectives:
Identify and use natural numbers, integers (positive, negative and zero), prime
numbers, common factors and common multiples, rational and irrational
numbers and real numbers (e.g., 𝜋, √2).
Natural Numbers
The natural (or counting) numbers are 1,2,3,4,5,6,7,8,9,etc. There are infinitely
many natural numbers. The set of natural numbers, {1,2,3,4,5,...}{1,2,3,4,5,...}, is
sometimes written N for short.

The whole numbers are the natural numbers together with 00.
(Note: a few textbooks disagree and say the natural numbers include 00.)
The sum of any two natural numbers is also a natural number (for
example, 4+2000 =2004), and the product of any two natural numbers is a natural
number (4×2000=8000). This is not true for subtraction and division though.

The Integers
10

The integers are a set of real numbers consisting of the natural numbers, their
additive inverses and zero.
{...,−5,−4,−3,−2,−1,0,1,2,3,4,5,...}{...,−5,−4,−3,−2,−1,0,1,2,3,4,5,...}
The set of integers is sometimes written JJ or ZZ in short.
The sum, product, and difference of any two integers is also an integer. But this is
not true for division... just try 1÷21÷2.

The Rational Numbers


The rational numbers are those numbers which can be expressed as a
ratio between two integers. For example, the fractions 1/3 and −11118 are both
rational numbers. All the integers are included in the rational numbers, since any
integer z can be written as the ratio z/1.
All decimals which terminate are rational numbers (since 8.278.27 can be written
as 827100.) Decimals which have a repeating pattern after some point are also
rational : for example,
0.0833333....= 0.0833333....=1/12
The set of rational numbers is closed under all four basic operations, that is, given
any two rational numbers, their sum, difference, product, and quotient is also a
rational number (as long as we don't divide by 00).
The Irrational Numbers
An irrational number is a number that cannot be written as a ratio (or
fraction). In decimal form, it never ends or repeats. The ancient Greeks
11

discovered that not all numbers are rational; there are equations that cannot be
solved using ratios of integers.
The first such equation to be studied was 2=𝑥 2 . What number times itself
equals 2?

√2 is about 1.414, because 1.4142=1.9993961=1.999396, which is close to 2. But


you'll never hit exactly by squaring a fraction (or terminating decimal). The square
root of 2 is an irrational number, meaning its decimal equivalent goes on forever,
with no repeating pattern:
√2=1.41421356237309...2=1.41421356237309...
Other famous irrational numbers are the golden ratio , a number with great
importance to biology:
(1+√5)/2 = 1.61803398874989...1 = 1.61803398874989...
Pi, the ratio of the circumference of a circle to its diameter:
π=3.14159265358979...π=3.14159265358979...
and e=2.71828182845904...
12

The Negative Numbers:


These are all of the numbers which are less than 0.

Type of Properties Examples


Numbers
Natural These are counting N= {1, 2, 3, 4, 5…}
Numbers (N) numbers from 1, 2, 3, 4, 5…
Whole Numbers These are natural numbers W= {0, 1, 2, 3, 4, 5…}
(W) including zero.
Positive These are 1, 2, 3, 4, 5… P= {1, 2, 3…}
Numbers
Negative These are -3, -4, -2, -1 N= {-3,-4,-5,-6}
Numbers
Integers (Z) These are whole numbers Z= {……, -3, -2, -1, 0, 1, 2,
together with negative 3 …}
numbers.
13

Rational All numbers of the form  Proper Fraction:


Numbers (Q) where a and b are integers Numbers smaller than
(but b cannot be zero). 1 e.g., : 1/2 or 3/4
Rational numbers include  Improper Fraction:
fractions. Numbers greater than
1 e.g., : 5/2
1
 Mixed Fraction: 2
2
= 5/2
 Powers and square
roots may be rational
numbers if their
standard form is a
rational number.
Irrational These cannot be expressed √𝟐, √𝟑, √𝟕, √𝟖, 𝝅
Numbers (Q1) as ratio of integers. As
decimals they never repeat
or terminate.
Irrational Numbers are Real
Numbers, but not all Real
Numbers are Irrational
Numbers
Real Numbers Real Numbers are every 0.23, 3.14, 2/3
number, irrational or
rational.
R =Q+Q1

1.1.1 Problem Set 1.1


1. Identify the following types of numbers.
a) 0.45
b) 0
14

c) 5/3
d) √2
e) −√81
f) 𝜋
g) -3/4
h) 3.1415926535……
i) 3.14159
j) √25

2. Identify the following number with proper justification.


a) -9/3

1.2 Squares, square roots, cubes and cube roots


General Objectives: know the notation for and find squares, square roots, cubes
ad cube roots
Specific Objectives: Calculate squares, square roots, cubes and cube roots of re-al
numbers.
The Definition of Square Roots
A square root of a number is a number that when multiplied by itself yields the
original number. For example, 4 is a square root of 16, because a square of 4
yields 16 , we can say that −4 is a square root of 16 as well. Every positive real

√16=4
4² = 16
2
−4 = −4 × −4=16
15

number has two square roots, one positive and one negative.

Example : Solution:

Find the square root:

a. √𝟏𝟐𝟏 a. √𝟏𝟐𝟏 = √𝟏𝟏² = 𝟏𝟏

b. √𝟎. 𝟐𝟓 b. √𝟎. 𝟐𝟓 = √𝟎. 𝟓² = 𝟎. 𝟓

𝟒 𝟒 𝟐 𝟐
c. √𝟗 c. √𝟗 = √(𝟑) ² = 𝟑

Example : Solution:

Find the negative square root:

a. −√𝟔𝟒 a. −√𝟔𝟒 = −√𝟖𝟐 = −𝟖

b. −√𝟏 b. −√𝟏 = −√𝟏𝟐 = −𝟏

The Definition of Cube Roots


A cube root of a number is a number that when multiplied by itself three times
yields the original number. Furthermore, we denote a cube root using the symbol
³√ , where 3 is called the index. For example, ³√82 ,  because cube of 2 yields 8.
The product of three equal factors will be positive if the factor is positive, and
negative if the factor is negative. For this reason, any real number will have only
one real cube root. Hence the technicalities associated with the principal root do
3
not apply. For example, √−8 = - 2.
16

Example : Solution:

Find the cube root:


𝟑
a. √𝟏𝟐𝟓 a. ³√𝟏𝟐𝟓 = ³√𝟓𝟑 = 𝟓
𝟑
b. √𝟎 b. ³√𝟎 = ³√𝟎𝟑 = 𝟎

𝟑 𝟖 𝟖 𝟐 𝟐
c. √𝟐𝟕 c. ³√𝟐𝟕 = ³√(𝟑) ³ = 𝟑

Example : Solution:

Find the cube root:


𝟑
a. √−𝟐𝟕 a. ³√−𝟐𝟕 = ³√(−𝟑)³ = −𝟑
𝟑
b. √−𝟏 b. ³√−𝟏 = ³√(−𝟏)𝟑 = −𝟏

Simplifying Square Roots


It will not always be the case that the radicand is a perfect square. If not, we use
the following two properties to simplify the expression. Given real numbers √𝐴
and √𝐵 where B≠0,

Product Rule for Radicals √𝑨 ∙ 𝑩 = √𝑨 ∙ √𝑩

𝑨 √𝑨
Quotient rule for Radicals √𝑩 =
√𝑩
17

List of commonly used perfect Powers

Squares: Cubes: 4th Powers: 5th Powers:

2²=4 8²=64 2³=8 24 =16 25 =32

3²=9 9²=81 3³=27 34 =81 35 =243

4²=16 10²=100 4³=64 44 =256 45 =1024

5²=25 11²=121 5³=125 54 =625

6²=36 12²=144 6³=216

7²=49 13²=169 7³=343

Example :

Simplify: √135

Solution:

Begin by finding the largest perfect square factor of 135:

135 = 3 · 3 · 3 · 5

= 3² · 3 · 5

= 9 · 15

Therefore, √135 = √9 ∙ 15 Apply the product rule for radicals.

= √9 · √15 Simplify roots of perfect squares.

=3 √15

Answer: 3 √15
18

Example :

108
Simplify: √169

Solution:

Begin by finding the prime factorizations of both 108 and 169. This will enable us to easily
determine the largest perfect squares factors:
108 = 2 · 2 · 3 · 3 · 3 =2² · 3² · 3
169 = 13 · 13 = 13²

108 2² · 3² ·3
Therefore, √169 = √ Apply the product and quotient rule for radicals.
13²

√2² · √3² · √3
= Simplify square roots of perfect squares.
√13²

2 · 3· √3
= 13

6 √3
=
13

6 √3
Answer:
13

Example:

Simplify: -5√162

Solution: -5√162 = -5 · √81 · 2

= -5 · √81 · √2

= -5 · 9 · √2

= - 45√2

Answer: = - 45√2
19

To Calculate Cube root or Square root by Prime Factorization we use the


following steps:

Step I : Obtain the given number.


Step II : Resolve it into prime factors.
Step III : Group the factors into 3(For cube root) or 2(for square root) in such a
way that each number of the group is the same.
Step IV : Take one factor from each group.
Step V : Find the product of the factors obtained in step IV. This product is the
required cube root or square root.

Example : Finding the cube root by prime factorization

3
1) √64
Solution :
64 = 2 x 32
= 2 x 2 x 16
=2x2x2x8
=2x2x2x2x4
= 2 x 2 x 2 x 2 x 2 x 2 (make the groups of 3 of equal numbers)
There are two groups, so from each group take one factor.
3
∴ √64 = 2 x 2
3
∴ √64 = 4

Example : Finding the Square root by prime factorization

1) √64
Solution :
64 = 2 x 32
= 2 x 2 x 16
=2x2x2x8
=2x2x2x2x4
= 2 x 2 x 2 x 2 x 2 x 2 (make the groups of 2 of equal numbers)
There are two groups, so from each group take one factor.
20

∴ √64 = 2 x 2 x 2
∴ √64 =8

1.2.1 Problem Set 1.2


1. Simplify √24√6
2. Simplify √75
3. Simplify √4500

1.3 Directed numbers


General Objectives: understand and use directed numbers
Specific Objectives:
1. Use directed numbers in practical situations (e.g., temperature change,
flood levels).
2. Apply the four basic operations on directed numbers.
Directed Numbers

Here we will learn the use of integers as directed numbers. We know that, when
numbers represent direction, the numbers are called directed numbers.

Example: What do the following expressions mean?

(i) Profit of - N$27.

(ii) -14 km to the East.

(iii) Ascending -30 meters.

Solution:

(i) Profit of –N$27 = Loss of N$27.

(ii) -14 km to the East = 14 km towards West.

(iii) Ascending -30 meters or -30 meters upward movement = 30 meters


downward movement.
21

Example: A boy moves 32 m due East and then 27 m due West. Find his position
with respect to his starting point.

Solution:

Let 32 m due East be represented by +32, then -27 represents 27 m due West.

On adding +32 and -27, we get;

(+32) + (-27)

= +32 – 27

= +5; which is positive.

Therefore, the position of the boy with respect to his starting point is 5 m due
East.

Example: A person walks 37 m due South and then 45 m due North. Find his
position with respect to his starting point.

Solution:

Let 37 m due South be represented by +37, then -45 represents 45 m due North.

On adding +37 and -45, we get;

(+37) + (-45)

= +37 – 45

= -8; which is negative.

Therefore, the position of the person with respect to his starting point is -8 m due
South.

Example: The temperature of a body first rises by 25° C and then falls by 32° C.
Find the final temperature of the body, if its initial temperature is:

(i) 0° C
22

(ii) 18° C

(iii) -20° C

(iv) 1° C

Solution:

(i) Final temperature = 0° C + (+ 25° C) + (- 32° C)

= 0° C + 25° C - 32° C

= 25° C - 32° C

= -7° C

(ii) Final temperature = 18° C + (+ 25° C) + (- 32° C)

= 18° C + 25° C - 32° C

= 43° C - 32° C

= 11° C

(iii) Final temperature = -20° C + (+ 25° C) + (- 32° C)

= -20° C + 25° C - 32° C

= 25° C - 52° C

= -27° C

(iv) Final temperature = 1° C + (+ 25° C) + (- 32° C)

= 1° C + 25° C - 32° C

= 26° C - 32° C

= -6° C

Multiplication and Division of directed Numbers


23

1. Plus times plus is plus


(adding many hot cubes raises the temperature)

2. Minus times plus is minus


(removing many hot cubes reduces the temperature)

3. Plus times minus is minus


(adding many cold cubes reduces the temperature)

4. Minus times minus is plus


(removing many cold cubes raises the temperature)

Solved Examples:
 Simplify (–2) x (–3) x (–4).
I'll start by cancelling off one pair of "minus" signs. Then I'll multiply as usual.

(–2)(–3)(–4)

= (–2)(–3)(–4)

= (+6)(–4)

= –24

If you're given a long multiplication with negatives, just cancel off "minus" signs in
pairs:

 Simplify (–1) x (–2) x (–1) x (–3) x (–4) x (–2) x (–1).


The first thing I'll do is count up the "minus" signs. One, two, three, four, five, six,
seven. So there are three pairs that I can cancel off, with one left over. As a result,
my final answer should be negative. If I come up with a positive result, I'll know
I've done something wrong.

(–1)(–2)(–1)(–3)(–4)(–2)(–1)

= (–1)(–2)(–1)(–3)(–4)(–2)(–1)
24

= (+1)(+2)(–1)(–3)(–4)(–2)(–1)

= (1)(2)(–1)(–3)(–4)(–2)(–1)

= (1)(2)(+1)(+3)(–4)(–2)(–1)

= (1)(2)(1)(3)(–4)(–2)(–1)

= (1)(2)(1)(3)(+4)(+2)(–1)

= (1)(2)(1)(3)(4)(2)(–1)

= (2)(3)(4)(2)(–1)

= 48(–1)

= –48

I got a negative answer, so I know my sign is correct.

1.3.1 Problem Set 1.3


1. Find the correct answer:

a) (−2) ÷ (−4)
b) (8) ÷ (−2)
c) (2) × (−3)
d) (−2) × (−2)
e) (3) + (−2)
f) (−2) + (−5)
25

1.4 Vulgar and decimal fractions and percentages


General Objectives: know and use the relationships between vulgar and decimal
fractions and percentages
Specific Objectives:
1. Use the language and notation of simple vulgar and decimal fractions and
percentages in appropriate context.
2. Recognize equivalence and convert between these forms.
3. Calculate a given percentage of a quantity.
4. Express one quantity as percentage of another.
5. Calculate percentage increase and decrease.
6. Carry out calculations involving reverse percentages e.g., finding the cost
price, given the selling price and the percentage profit.

Vulgar fraction

Definition:

A vulgar fraction or common fraction or simple fraction — is a rational number


written as one integer (the numerator) m divided by a non-zero integer (the
denominator) n, such as m/n.

Proper fraction

Definition:

A vulgar fraction is said to be a proper fraction if the absolute value of the


numerator is less than the absolute value of the denominator.

5/7 , 6/9 are proper fractions.


26

Improper fraction

Definition.

A vulgar fraction is said to be an improper fraction or composed fraction if the


absolute value of the numerator is greater than or equal to the absolute value
of the denominator. 8/2, 7/4 are improper fractions.

Definition of decimal numbers:

We have learnt that decimals are an extension of our number system.


We also know that decimals can be considered as fractions whose
denominators are 10, 100, 1000, etc. The numbers expressed in the
decimal form are called decimal numbers or decimals.

For example : 5.1, 4.09, 13.83, etc.

A decimal has two parts:

(a) Whole number part

(b) Decimal part

These parts are separated by a dot ( . ) called the decimal point.

• The digits lying to the left of the decimal point form the whole
number part. The places begin with ones, then tens, then hundreds,
then thousands and so on.

• The decimal point together with the digits lying on the right of
decimal point form the decimal part. The places begin with tenths,
then hundredths, then thousandths and so on………

For example:

(i) In the decimal number 211.35; the whole number part is 211 and
the decimal part is .35
27

It can be arranged in the place-value chart as:

Converting a decimal to vulgar fraction:


Method:
a) Calculate the total numbers after decimal point.
b) Remove the decimal point from the number.
c) Put 1 under the denominator and annex it with "0" as many as the
total in step a.
d) Reduce the fraction to its lowest terms.
Example: Consider 0.44
Step a: Total numbers after decimal point are 2
Step b and c: 44/100
Step d: Reducing it to lowest terms : 44/100 = 22/50 = 11/25

Example of converting decimals to fractions.


Example 1.
4 2
0.4 = =
10 5
Example 2.
346 173
1.346 = 1 =1
1000 500
28

Converting a recurring decimal to vulgar fraction

Follow the steps for the conversion of pure recurring decimal into vulgar fraction:

(i) First write the decimal form by removing the bar from the top and put it equal
to n (any variable).

(ii) Then write the repeating digits at least twice.

(iii) Now find the number of digits having bars on their heads.

● If the repeating decimal has 1 place repetition, then multiply both sides by 10.

● If the repeating decimal has 2 place repetitions, then multiply both sides by
100.

● If the repeating decimal has 3 place repetitions, then multiply both sides by
1000 and so on.

(iv) Then subtract the number obtained in step (i) from the number obtained in
step (ii).

(v) Then divide both sides of the equation by the coefficient of n.

(vi) Therefore, we get the required vulgar fraction in the lowest form.

Worked-out examples for the conversion of pure recurring decimal into vulgar
fraction:

Example: Express ⃗⃗⃗


. 4 as a vulgar fraction.

Solution:

Let n = 0.4

n = 0.444 ----------- (i)

Since, one digit is repeated after the decimal point, so we multiply both sides by
10.
29

Therefore, 10n = 4.44 ----------- (ii)

Subtracting (i) from (ii) we get;

10n - n = 4.44 - 0.44

9n = 4

n = 4/9 [dividing both sides of the equation by 9]

Therefore, the vulgar fraction = 4/9

Example: Express .⃗⃗⃗⃗⃗⃗⃗⃗⃗


532 as a vulgar fraction.

Solution:

Let n = 0.532

n = 0.532532 ----------------- (i)

Since, three digits are repeated after the decimal point, so we multiply both sides
by 1000.

Therefore, 1000n = 532.532 ----------------- (ii)

Subtracting (i) from (ii) we get;

1000n - n = 532.532 - 0.532

999n = 532

n = 532/999

Therefore, the vulgar fraction = 532/999

Shortcut Method for conversion:

A decimal with recurring value is called recurring decimal. E.g: 2/9 will give
0.22222222...... where 2 is a recurring number.
Method: a) Separate the recurring number from the decimal fraction.
30

b) Annex denominator with "9" as many times as the length of the recurring
number.
c) Reduce the fraction to its lowest terms.
Example: Consider 0.2323232323 or can be written as .23
Step a: The recurring number is 23
Step b: 23/99 [the number 23 is of length 2 so we have added two nines]
Step c: Reducing it to lowest terms : 23/99 [it cannot be reduced further].
Percentages:

How to find the percentage of the given quantity?

We know, a percentage is a fraction with denominator as 100 i.e. % = 1/100.


Therefore, to determine the exact value of a percent of a given quantity we need
to express the given percent as fraction and multiply it by the given number.

We will follow the following steps for finding a percentage of a given number:

Step I: Obtain the number. Let the number be m.

Step II: Obtain the required percentage. Let it be R %.

Step III: To find R % of m, multiply m by R and then divide by 100; i.e. R % of m =


R/100 × m

Following examples will help us to find the percentage of the given quantity using
the above procedure.

Solved examples to find percent of a given number:

Example: Find 40 % of 240

Solution:

We know that R % of m is equal to R/100 × m.

So, we have 40 % of 240


31

40/100 × 240

= 96

Example: 10 % of 1 hour

Solution:

We know that R % of m is equal to R/100 × m.

So, we have 10 % of 1 hour

10 % of 60 minutes (Since, 1 hour = 60 minutes)

= 10/100 × 60 minutes

= 6 minutes

Example: Find 15 % of N$250.

Solution:

We know that R % of m is equal to R/100 × m.

So, we have 15 % of N$250.

15/100 × 250

= N$75/2

= N$37.50

Example: What is the sum of the money of which 15 % is N$225?

Solution:

Let the required sum of money be N$m.


32

15 % of N$m = N$225

⇒ 15/100 × m = 225

⇒ m = (225 × 100)/15

⇒ m = 1500

Therefore, sum of the money = N$1500

Example: In a public show 75 % of the seats were filled. If there were 600 seats in
the hall, how many seats were vacant?

Solution:

First method:

75 % of 600

= 75/100 × 600

= 450

Therefore, the number of vacant seats = 600 - 450 = 150

Second method:

Total percentage of seats = 100

Percentage of filled seats = 75

Therefore, percentage of vacant seats = 100 – 75 = 25.

25 % of 600

= 25/100 × 600

= 150.

Thus, the number of vacant seats is 150.

Note: Total is always 100%.


33

How to covert a percentage into fraction?

We will follow the steps for converting a percentage into a fraction:

Step I: Obtain the given percentage. Let it be 𝑥 %.

Step II: Remove the percentage sign (%) and then divide the number by 100.
Therefore, 𝑥 % = 𝑥/100

Step III: Reduce the fraction obtained to its lowest terms as required.

1. Express each of the following percentage into fraction in lowest terms:

(i) 16 % = 16/100 = 4/25

(ii) 48 % = 48/100 = 12/25

(iii) 5 % = 5/100 = 1/20

(iv) 25 % = 25/100 = 1/4

2. Convert 27 percent to fraction.

(i) 27 % = 27/100

3. Convert each of the following percentage as fractions to lowest form:

(i) 24 % = 24/100 = 6/25

(ii) 62 % = 62/100 = 31/50

(iii) 30 % = 30/100 = 3/10

(iv) 75 % = 75/100 = ¾

(v) 10 % = 10/100 = 1/10

4. Convert each of the decimal percentage as fractions to lowest form:

(i) 3.5 % = 35/10 % = 35/10 × 1/100 = 7/200


34

(ii) 0.5 % = 5/10 % = 5/10 × 1/100 = 1/200

(iii) 30.2% = 302/10 % = 302/10 × 1/100 = 302/1000 = 151/500

(iv) 0.4 % = 4/10 % = 4/10 × 1/100 = 4/1000 = 1/250

(v) 0.375 % = 375/1000 % = 375/1000 × 1/100 = 375/100000 = 3/800

5. Convert each of the fraction percentage as fractions to lowest form:

(i) 32/5 % = 17/5 % = 17/5 × 1/100 = 17/500

(ii) 2/5 % = 2/5 × 1/100 = 1/250

(iii) 162/3 % = 50/3 % = 50/3 × 1/100 = 50/300 = 1/6

(iv) 51/8 % = 41/8 % = 41/8 × 1/100 = 41/800

(v) 2¾ % = 11/4 % = 11/4 × 1/100 = 11/400

How to convert a decimal into percentage?

We will follow the following steps for converting a decimal into a percentage:

Step I: Obtain the number in decimal form.

Step II: Multiply the number in decimal form by 100 and put percent sign (%)

Note: When we multiply the decimal number by 100, we need to shift the decimal
point two places to the right (add zeros if necessary).

I. Express each of the following decimal as percent:

(i) 0.8 = 0.8 × 100 % = 80 %

(ii) 1.2 = 1.2 × 100 % = 120 %

(iii) 7.1 = 7.1 × 100 % = 710 %


35

(iv) 10.1 = 10.1 × 100 % = 1010 %

II. Conversion of decimal into percent:

(i) 0.29 = 0.29 × 100 % = 29 %

(ii) 0.25 = 0.25 × 100 % = 25 %

(iii) 0.01 = 0.01 × 100 % = 1 %

(iv) 0.51 = 0.51 × 100 % = 51 %

III. Convert the following decimal into a percentage:

(i) 0.083 = 0.083 × 100 % = 8.3 %

(ii) 0.225 = 0.225 × 100 % = 22.5 %

(iii) 0.001 = 0.001 × 100 % = 0.1 %

(iv) 0.003 = 0.003 × 100% = 0.3 %

Note: Decimals shifts by 2 places to the right.

Increase percentage

It can easily be understood if it is expressed as percent. We will follow the


following steps to convert the increase into percent.

Step I: First find the increase in value

Step II: Divide it by the original quantity

Step III: Multiply the fraction by 100 and put percent sign (%)
Increase in value
Formula for finding the increase % is × 100%
Original value

Note: Increase percent is calculated on the original value.


36

For example:

If price of milk increases from N$4 per litre to N$5.40 per litre.

Increase in price = N$5.40 - N$4 = N$1.40

and increase % = Increase in price/Original price × 100 %

= 1.40/4 × 100 %

= 140/4 %

= 35 %

We will apply the concept of solving some real-life problems by using the formula
for finding the increase percent.

Solved examples:

Example: The price of rice is increased from N$10 to N$12.50 per kg. Find the
percentage increase in price?

Solution:

Price of rice before = N$10

Price of rice now = N$12.50

Increase in price = current price – original price

=N $12.50 - N$10

= N$2.50

Therefore, percentage increase in price = Increase in price/Original price × 100 %

= 2.50/10 × 100 %

= 250/10 %

= 25 %
37

Thus, increase in price= 25 %

Example: The population in Tsumeb increases from 20000 to 21250 in one year.
Find the percentage increase in population.

Solution:

Population in Tsumeb last year = 20000

Population in Tsumeb after one year = 21250

Increase in population = 21250 - 20000 = 1250

Therefore, percentage increase in population = Increase in population/Last year


population × 100 %

= 1250/20000 × 100 %

= 125000/20000 %

= 25/4 %

= 6.25%

Thus, the increase in population is 6.25%

Example: Find the increase value if 150 is increased by 30 %.

Solution:

Increase = 30 % of 150

= 30/100 × 150

= 4500/100

= 45
38

Therefore, increase value = 150 + 45 = 195

Example: By what number must the given number be multiplied to increase the
number by 50 %.

Solution:

Let the number be m

Increase in its value = 50 % of m

= 50/100 × m

= m/2

Therefore, increase value = m + m/2

= (2m + m)/2

= 3m/2

Therefore, the given number must be multiplied by 3/2 to increase the number by
50 %.

Decrease percentage

It can easily be understood if it is expressed as percent. We will follow the


following steps to convert the decrease into percent.

Step I: first find the decrease

Step II: divide it by the original quantity

Step III: multiply the fraction by 100%

Formula for finding the Decrease% is = Decrease in value/Original value × 100%

Note: decrease percent is calculated on the original value.


39

For example:

The price of sugar decreases from N$8 per kg to N$6.40 per kg.

Decrease in price = N$8 - N$6.40 = N$1.60

and decrease % = decrease in price/Original price × 100 %

= 1.60/8 × 100 %

= 160/8 %

= 20 %

We will apply the concept of solving some real-life problems by using the formula
for finding the decrease percent.

Solved examples:

Example: The cost of an item is decreased by 15%. If the original cost is N$80, find
the decrease cost.

Solution:

Original cost = N$80

Decrease in it = 15% of N$80

= 15/100 × 80

= 1200/100

= N$12

Therefore, decrease cost = N$80 - N$12 = N$68

Example: A television manufacturing company declares that a television is now


available for N$5600 compared to N$8400 one year before. Find the percentage
reduction in the price of television offered by the company.
40

Solution:

Price of the television a year before = N$8400

Price of the television after a year = N$5600

Decrease in price = N$(8400 - 5600) = N$2800

Therefore, decrease % = 2800/8400 × 100 % = 100/3 = 33.33%

Example: Find the decrease value if 300 decreased by 30%

Solution:

Decrease 300 by 30%

= 30/100 × 300

= 90

Therefore, decrease value = 300 – 90 = 210

Example: Find the number , when decreased by 12 % becomes 198.

Solution:

Let the number be m.

Decrease = 12 % of m

= 12/100 × m = 3m /25

Therefore, decrease number = m – 3m/25 = (25m – 3m)/25 = 22m/25

According to the question, 22m/25 = 198

22m = 198 × 25

m = 4950/22
41

m = 225

Profit and Loss In terms of Percentage:

In calculating profit percent and loss percent we will learn about the basic
concepts of profit and loss. We will recall facts and formulas while calculating
profit percent and loss percent. Now we will apply the concept of percentage to
find profit/loss in selling and buying of goods in our day to day life.

Cost price (CP) The amount for which an item is bought is called its cost price.

Selling price (SP) The amount for which an item is sold is called its selling price.

Profit or gain When (SP) > (CP) then there is a gain.

Gain = (SP) - (CP)

Loss When (SP) < (CP) then there is a loss.

Loss = (CP) - (SP).

Notes:

The gain or loss is always reckoned on the cost price

Profit and loss formulas for calculating profit% and loss%:

I. Gain = (SP) - (CP)

II. Loss = (CP) - (SP)

III. Gain% = (gain / CP × 100)%

IV. Loss % = (loss/ CP × 100)%

V. To find SP when CP and gain% or loss% are given:


42

● SP = [(100 + gain %) / 100] × CP

● SP = {(100 - loss %) /100} × CP

VI. To find CP when SP and gain% or loss% are given:

● CP = {100/(100 + gain %)} × SP

● CP = {100 /(100 - loss %)} × SP

Worked-out problems on calculating profit percent and loss percent:

Example: Loide bought a DVD for N$ 750 and sold it for N$ 875. Find Loide’s gain
percent.

Solution:

CP = N$ 750 and SP = N$ 875

Since (SP) > (CP), Loide makes a gain.

Gain = N$ (875 - 750)

= N$ 125.00

Gain% = {(gain/CP) × 100} %

= {(125/750) × 100} %

= (50/3) %

= 16.66 % .
Example: Kamati purchased a table for N$ 1260 and due to some scratches on its
top he had to sell it for N$ 1197. Find his loss percent.

Solution:
43

CP N$1260 and SP = N$ 1197

Since (SP) < (CP), Kamati makes a loss.

Loss = N$ (1260 - 1197)

= N$ 63.00

Loss % = [(loss / CP) × 100] %

= [(63 / 1260) × 100] %

= 5%

In calculating profit percent and loss percent, sometimes after purchasing an


item, we have to pay some more money for things like transportation, repairing
charges, local taxes, These extra expenses are called overheads.
For calculating the total cost price, we add overheads to the purchase price.

NB: Some important formulae are as follows:

 (a + b)(a - b) = (a2 - b2)


 (a + b)2 = (a2 + b2 + 2ab)
 (a - b)2 = (a2 + b2 - 2ab)
 (a + b + c)2 = a2 + b2 + c2 + 2(ab + bc + ca)
 (a3 + b3) = (a + b)(a2 - ab + b2)
 (a3 - b3) = (a - b)(a2 + ab + b2)
 (a3 + b3 + c3 - 3abc) = (a + b + c)(a2 + b2 + c2 - ab - bc - ac)
 When a + b + c = 0, then a3 + b3 + c3 = 3abc.

1.4.1 Problem Set 1.4


1. Express 0. 4̅ as a vulgar fraction.
2. ̅̅̅̅ and 0. 5̅ as vulgar fractions.
Express 0. 45
3. Express 0. ̅̅̅̅̅
532 as a vulgar fraction.
4. ̅̅̅̅ as a vulgar fraction.
Express 0. 38
44

1.5 Ordering
General Objectives: Order quantities by magnitude
Specific Objectives: Order quantities by magnitude and demonstrate familiarity
with the symbols =, ≤, ≥, ≠, <, >

ORDERING OF NUMBER

Ascending Order
Numbers are said to be in ascending order when they are arranged from the
smallest to the largest number.

E.g. 5, 9, 13, 17 and 21 are arranged in ascending order.

Example

Arrange the following numbers in ascending order:


43, 52, 15, 26, 34

Solution:
4³ = 4 × 4 × 4 = 64
5² = 5 × 5 = 25

15 = 1

26 = 2 × 2 × 2 × 2 × 2 × 2 =64
34 = 3 × 3 × 3 × 3 = 81
The numbers arranged in ascending order are 1, 25, 64, 64, 81.
So, 15, 52, 43, 26, 34 are arranged in ascending order.
45

Descending Order

Numbers are said to be in descending order when they are arranged from the
largest to the smallest number.

E.g. 25, 21, 17, 13 and 9 are arranged in descending order.

Example

Arrange the following numbers in descending order:

42, 52, 24, 33

Solution:

42 = 4 × 4 = 16

52 = 5 × 5 = 25

24 = 2 × 2 × 2 × 2 = 16

33 = 3 × 3 × 3 = 27

The numbers arranged in descending order are 27, 25, 16, 16.
So, 33, 52, 42, 24 are arranged in descending order.

Greater or Less than and Equal to

In math, greater or less than and equal to help learners to understand how one
number is different from the other number. Whether one number is bigger or
smaller than the other number or both numbers are equal.

Important symbol or signs are used to identify the numbers and understand the
bigger number, smaller number and the numbers that are equal.

When one number is bigger than the other number; we use greater than sign >
46

When one number is smaller than the other number; we use less than sign <

When one number is equal to the other number; we use equal to sign =

● > Right hand open is always greater than sign.

● < Left hand open is always less than sign.

Bigger number > Smaller number

here it is greater than sign

Smaller number < Bigger number

here it is less than sign

Example:

● 5 is greater than 1

5 > 1

● 1 is less than 2

1 < 2

● 3 is equal to 3

3 = 3

1.5.1 Problem Set 1.5


1. Place 17, 5, 9 𝑎𝑛𝑑 8 in ascending order.
2. Place 3, 1, 19, 12, 9, 2 𝑎𝑛𝑑 7 in ascending order.
3. List the following numbers from highest to lowest value using inequality
signs
42 , 52 , 24 , 33
47

4. Sam must be at least 13 years or older to go to the park. Indicate this


expression using inequality sign.
5. In an exam, Alex has higher scores than Samuel. Samuel has equal scores as
Marry. Lina has the lowest score. Arrange these in ascending order.

1.6 Standard form


General Objectives: express very large and very small quantities in standard form
Specific Objectives: use the standard form 𝐴 × 10𝑛 where n is a positive or
negative integer and 1 ≤ 𝐴 ≤ 10
Standard form is a way of writing down very large or very small numbers easily.
10³ = 1000, so 4 × 10³ = 4000 . So 4000 can be written as 4 × 10³ . This idea can be
used to write even larger numbers down easily in standard form.
Small numbers can also be written in standard form. However, instead of the
index being positive (in the above example, the index was 3), it will be negative.
The rules when writing a number in standard form is that; first you write down a
number between 1 and 10, then you write × 10(to the power of a number).
Example
Write 81 900 000 000 000 in standard form: 81 900 000 000 000 = 8.19 × 1013
It’s 1013 because the decimal point has been moved 13 places to the left to get
the number to be 8.19
Example
Write 0.000 001 2 in standard form:
0.000 0012 = 1.2 ×10−6
It’s 10−6 because the decimal point has been moved 6 places to the right to get
the number to be 1.2
On a calculator, you usually enter a number in standard form as follows: Type in
the first number (the one between 1 and 10). Press EXP . Type in the power to
which the 10 is risen.
Manipulation in Standard Form
This is best explained with an example:
48

Example
The number p written in standard form is 8 ×105
The number q written in standard form is 5 × 10−2
Calculate p × q. Give your answer in standard form.
Multiply the two first bits of the numbers together and the two second bits
together:
8 × 5 × 105 × 10−2
= 40 × 103
The question asks for the answer in standard form, but this is not standard form
because the first part (the 40) should be a number between 1 and 10.
= 4 × 104
Example
Calculate p ÷ q
Give your answer in standard form.
This time, divide the two first bits of the standard forms. Divide the two second
bits. (8 ÷ 5) × (105 ÷ 10−2 ) = 1.6 × 107

Standard Equation of Polynomial:


The "Standard Form" for writing down a Polynomial is to put the terms with the
highest degree first (like the "2" in 𝑥 2 if there is one variable).
Example: Put this in standard Form : 5𝑥 2 -2 +𝑥 4 +𝑥 8

Solution: The highest degree is 8, so that goes first, then 4, 2 and then the
constant last:

𝑥 8 + 𝑥 4 + 5𝑥 2 -2 is the solution
49

1.6.1 Problem Set 1.6

1. Put this quadratic equation in standard form 3𝑥 2 − 7 + 4𝑥 3 + 𝑥 6


2. Put 𝑥 2 = 7 into the standard form
𝑥2 − 7 = 0

3. The number p written in standard form is 8 × 105 . The number q written in


standard form is 5 × 10−2 . Calculate p x q. Give your answer in standard
form.
4. Write 0.000 0012 in Standard form.
5. Write 81900 000 000 000 in Standard form.

1.7 Four basic operations

General Objectives: use the four basic operations in calculations


Specific Objectives: Apply the four basic operations for calculations with integers,
decimal fractions and vulgar (and mixed) fractions, including correct order of
operations and use of brackets.

BASIC OPERATIONS ACCORDING TO ORDER:

We will obey the rules for simplifying an expression using BODMAS rules -
involving integers for solving order of operations.

Follow the order of operation as:

1. Bracket → Solve inside the Brackets before Of, Multiply, Divide, Add or
Subtract.

For example:

7 × (15 + 5)

= 7 × 20
50

= 140

2. Of → Then, solve Of part (Powers, Roots, etc.,) before Multiply, Divide, Add or
Subtract.

For example:

6 + 3 of 7 - 5

=6+3×7-5

= 6 + 21 - 5

= 27 - 5

= 22

3. Division/Multiplication → then, calculate Multiply or Divide before Add or


Subtract. Start from left to right.

For example:

20 + 21 ÷ 3 × 2

= 20 + 7 × 2

= 20 + 14

= 34

4. Addition/Subtraction → at last Add or Subtract. Start from left to right.

17 + (8 - 5) × 5

= 17 + 3 × 5

= 17 + 15
51

= 32

Worked-out problems for solving BODMAS rules - involving integers:

Simplify using BODMAS rule:

(a) 25 - 48 ÷ 6 + 12 × 2

Solution:

25 - 48 ÷ 6 + 12 × 2

= 25 - 8 + 12 × 2, (Simplifying ‘division’ 48 ÷ 6 = 8)

= 25 - 8 + 24, (Simplifying ‘multiplication’ 12 × 2 = 24)

= 17 + 24, (Simplifying ‘subtraction’ 25 - 8 = 17)

= 41, (Simplifying ‘addition’ 17 + 24 = 41)

Answer: 41

(b) 78 - [5 + 3 of (25 - 2 × 10)]

Solution:

78 - [5 + 3 of (25 - 2 × 10)]

= 78 - [5 + 3 of (25 - 20)], (Simplifying ‘multiplication’ 2 × 10 = 20)

= 78 - [5 + 3 of 5], (Simplifying ‘subtraction’ 25 - 20 = 5)

= 78 - [5 + 3 × 5], (Simplifying ‘of’)

= 78 - [5 + 15], (Simplifying ‘multiplication’ 3 × 5 = 15)

= 78 - 20, (Simplifying ‘addition’ 5 + 15 = 20)

= 58, (Simplifying ‘subtraction’ 78 - 20 = 58)


52

Answer: 58

We will follow the rules for simplifying an expression using PEMDAS rules -
involving decimals for solving order of operations

Follow the order of operation as:

Example: Parenthesis → Find the things inside the Parenthesis part before
Exponent, Multiply, Divide, Add or Subtract.

For example:

7.04 × (4.07 + 6.72)

= 7.04 × 10.79

= 75.9616

Example: Exponent → Then do the Exponent part (Powers, Roots, etc.,) before
Multiply, Divide, Add or Subtract.

For example:

3.05 × (0.2)2 + 2

3.05 × 0.2 × 0.2 + 2

= 3.05 × 0.04 + 2

= 0.122 + 2

= 2.122

Example: Multiplication/ Division → Then do Multiply or Divide part before Add


or Subtract start from left to right.

For example:

5.75 × 3.5 + 6.4 ÷ 1.6


53

= 20.125 + 6.4 ÷ 1.6

= 20.125 + 4

= 24.125

Example: Addition/Subtraction → At last do Add or Subtract start from left to


right.

For example:

15.05 + (6.25 – 3.75) × 2.05

= 15.05 + 2.5 × 2.05

= 15.05 + 5.125

= 20.175

Worked-out problems for solving PEMDAS rules - involving decimals:

Simplify using PEMDAS rule:

(a) 16.5 – 16.5 ÷ 2.2 + 3.2 × 5.4

Solution:

16.5 – 16.5 ÷ 2.2 + 3.2 × 5.4

= 16.5 – 7.5 + 3.2 × 5.4, (Simplifying ‘division’ 16.5 ÷ 2.2 = 7.5)

= 16.5 – 7.5 + 17.28, (Simplifying ‘multiplication’ 3.2 × 5.4 = 17.28)

= 9 + 17.28, (Simplifying ‘subtraction’ 16.5 – 7.5 = 9)

= 26.28, (Simplifying ‘addition’ 9 + 17.28 = 26.28)

Answer: 26.28
54

(b) 10.75 - [3.25 + 3 of (19.45 – 2.65 × 6.75)]

Solution:

10.75 - [3.25 + 3 of (19.45 – 2.6 × 6.7)]

= 10.75 - [3.25 + 3 of (19.45 – 17.42)], (Simplifying 2.6 × 6.7 = 17.42)

= 10.75 - [3.25 + 3 of 2.03], (Simplifying 19.45 – 17.42 = 2.03)

= 1.75 - [3.25 + 3 × 2.03], (Simplifying ‘of’)

= 10.75 - [3.25 + 6.09], (Simplifying ‘multiplication’ 3 × 2.03 = 6.09)

= 10.75 – 9.34, (Simplifying ‘square brackets’ 3.25 + 6.09 = 9.34)

= 1.41, (Simplifying ‘subtraction’ 10.75 – 9.34 = 1.41)

Answer: 1.41

1.7.1 Problem Set 1.7


1. Simplify 4 + 32
2. Simplify 4 + (2 + 1)2
3. Simplify 4 + [−1(−2 − 1)]2
2 4
4. Simplify 4(− + )
3 3

1.8 Estimation
General Objectives: estimate numbers and quantities
Specific Objectives:
1. Make estimates of numbers and quantities
2. Give approximations to specified numbers of significant figures and decimal
places.
3. Round off to reasonable accuracy in the context of the given problem.
55

ROUNDING NUMBERS AND ESTIMATION

Rounding numbers is required when we deal with large numbers, for example,
suppose the population of a country is 5834237, it is difficult to remember the
seven digits and their order. But to get an idea of the size of the population of the
country, we may remember the two digits on the left and put zeros for other
digits. Thus, the population of the district can be conveniently remembered as
5800000.

We say that 5834237 has been rounded off to 5800000. Thus, 5800000 is a
rounded number for the number 5834237, rounded off to the nearest hundred
thousand. If we remember the population as 5830000, we have rounded off the
population to the nearest ten thousand.

Real life examples on rounding numbers:

(i) Kevin’s new neighbour asked him his age. Kevin said that he was fourteen years
old.

His actual age was fourteen years, two months and seven days.

(ii) David told Tjihero that he weighs about 50 kg. David’s exact weight was 52 kg.

(iii) Nelao paid N$245 for a pair of sports shoes. He told Johny: “ This pair of shoes
cost me nearly N$250”

In all this these statements, the numbers have been rounded off. A rounded
number is easy to remember and is a convenient figure for calculation.

While rounding numbers, the number 10 is very useful. We use it to think about
place value. The number 5 is also important.

5 is half of 10 and 5 is halfway 0 and 10.

These number lines show the halfway point between two numbers.

10 15 20 130 135 140 0 50 100

Example: Label the number line to show the halfway point between each pair of
numbers.
56

Look at the folded number line!

Each peak shows a number ending in 5. These numbers are halfway between the
two tens.

The tens are the base of each fold in the valleys.

The halfway number between 10 and 20 is 15.

The halfway number between 20 and 30 is 25.

If we place a counter on number 17, it would slide to 20 because 20 is the ten that
is nearest to 17.

A counter placed on 27 will slide to 30.

A counter placed on 36 will slide to 40.

A counter placed on 14 will slide to 10.

A counter placed on 33 will slide to 30.

Halfway number such as 5 and 15 are rounded to the nearest ten.

25 would be rounded to 30.


57

35 would be rounded to 40.

Numbers can be rounded to the nearest hundred, thousand, ten thousand and so
on.

The halfway numbers on all the number lines contain a 5. The place of the digit 5
varies.

The rules for rounding numbers are the same for rounding to the nearest ten,
hundred, and thousand ……

If the given number is less than the halfway number, then round down. The
rounded number will be less than the given number.

How do you round numbers?

Here we will discuss how to round numbers….

NB: If the given number is less than the halfway number, then round
down. The rounded number will be less than the given number
58

(i) 435 is less than the


halfway number, so it is
rounded down.
The rounded number is 400
which is less than the
original number 435. When
rounding, we replaced the
digits in the ones and tens
places by zeros. The digits in
the hundreds place remains
unchanged.
1Save
If the given number is
halfway or greater than the
halfway number, then it is
rounded up.

The rounded number will be


greater than the given
number.

(ii) Round 675 to the nearest


hundred.
675 is greater than the
halfway number, so it is
rounded up to 700. The
rounded number is greater
than the original number
675. When rounding, we
replaced the digits in the
ones and tens places by 1Save
zeros and increased the digit
in the hundreds place by 1.

A 3 digit number can be


rounded to the nearest TEN
or to the nearest HUNDRED.
59

(iii) Round 446 to the nearest ten.

Colour the digit in the tens place 446

Look at the digit to the right of the coloured digit. If it is greater than or equal to 5
then round up. If it is less than 5 then round down.

6>5

446 will be rounded up to 450.

(iv) Round 726 to the nearest hundred.

Colour the digit in the hundreds place 726

Look at the digit to the right. If It is greater than or equal to 5 then round up. If it
is less than 5 then round down.
2<5

726 will be rounded down to 700.

The above method is convenient and helps us to remember very large numbers
easily. It also simplifies calculation of large numbers.

Decimal fractions can also be rounded off to the nearest one place of decimal or
to two places of decimals or as required.

(i) 32.2 can be rounded off to 32

(ii) 46.8 can be rounded off to 47

(iii) 188.9 can be rounded off to 189

(iv) 59.1 can be rounded off to 59

(v) 170.2 can be rounded off to 170


60

Hilma bought a pair of shoes; the cost of a pair of shoes would probably read as
N$ 249.95. But she paid N$ 250, i.e., $ 249.95 has been rounded off to $ 250.

Pombili bought a pair of shirts; the cost of a pair of shirts would probably read as
N$ 143.22. But she paid N$ 143, i.e., N$ 143.22 has been rounded off to $ 143.

Nearest 1

While rounding off a decimal fraction to the nearest one, if the digit in the tenths
place is < 5 then the tenths place and the entire following place is replaced by 0. If
the tenths place is equal to or > 5 then the digit in the ones place is increased by 1
and all digits after the decimal are reduced to 0.

(i) 49.37 → 49.00

We see the digit in the tenths place is 3, round to the nearest whole number
which is smaller than the given decimal number. Since 3 < 5 then the decimal
number is rounded to 49.00

(ii) 96.872 → 97.00

Here, the digit in the tenths place is 8, round to the nearest whole number which
is greater than the given decimal number. Since 8 > 5 then the decimal number is
rounded to 97.00.

(iii) 67.437 → 67.000

We can see that the digit in the tenths place is 4, round to the nearest whole
number which is smaller than the given decimal number. Since 4 < 5 then the
decimal number is rounded to 67.000

(iv) 9.909 → 10.000

Here, the digit in the tenths place is 9, round to the nearest whole number which
is greater than the given decimal number. Since 9 > 5 then the decimal number is
rounded to 10.000

(v) 490.651 → 491.000


61

Here, the digit in the tenths place is 6, round to the nearest whole number which
is greater than the given decimal number. Since 6 > 5 then the decimal number is
rounded to 491.000

(vi) 2492.201 → 2492.000

We see the digit in the tenths place is 2, round to the nearest whole number
which is smaller than the given decimal number. Since 2 < 5 then the decimal
number is rounded to 2492.000

Note: Remember, to round off decimal numbers to the nearest ones (or units
place), look at the digit in the tenths place

We will discuss here how to round decimal numbers to the nearest hundredths or
correct to two decimal places.

To round off decimal numbers to the nearest hundredths, look at the digit in the
thousandths place.

1. When the digit in the thousands place is < 5, the thousandths place and
following digits are replaced by 0.

2. When the digit in the thousands place is = or > 5, the digit in the hundredths
places is increased by 1 and the following digits become 0.

Examples on rounding off to correct two places of decimal or rounding off to the
nearest hundredths:

(i) 189.434 → 189.430

We see the digit in the thousandths place is 4, then round it to the nearest
hundredths which is smaller than the given decimal number. Since 4 < 5 then the
decimal number is rounded to 189.430

(ii) 27.987 → 27.990

We see the digit in the thousandths place is 7, then round it to the nearest
62

hundredths which is greater than the given decimal number. Since 7 > 5 then the
decimal number is rounded to 27.990

By rounding off to the nearest 100, we will learn how to round the number to the
nearest hundred.

1. While rounding off to the nearest hundred, if the digit in the tens place is
between 0 – 4 i.e. < 5, then the tens place is replaced by ‘0’.

2. If the digit in the units place is equal to or >5, then the tens place is replaced by
‘0’ and the hundreds place is increased by 1.

Examples on rounding off to the nearest hundred (100):

(i) 142 → 100


We see the digit in the tens place is 2, we round to the nearest multiple
of hundred which is less than the number. Hence, 142 is nearer to 100 than 200.

(ii) 486 → 500


We see the digit in the tens place is 6, we round to the nearest multiple
of hundred which is greater than the number. Hence, 486 is nearer to 500 than
400.

(iii) 799 → 800


We see the digit in the tens place is 9, we round to the nearest multiple of
hundred which is greater than the number. Hence, 799 is nearer to 800 than 700.

Round off to nearest 1000

A large number may be rounded off to the nearest 1000, 10000 and more.

1. While rounding off to the nearest thousand, if the digit in the hundreds place is
between 0 – 4 i.e., < 5, then the hundreds place is replaced by ‘0’.

2. If the digit in the hundreds place is = to or > 5, then the hundreds place is
replaced by ‘0’ and the thousands place is increased by 1.

Examples on rounding off to the nearest thousand (1000):

(i) 3846 → 4000


63

We see the digit in the hundred’s place is 8, we round to the nearest multiple of
thousand which is greater than the number. Hence, 3846 is nearer to 4000 than
3000.

(ii) 8039 → 8000

We see the digit in the hundred’s place is 0, we round to the nearest multiple of
thousand which is smaller than the number. Hence, 8039 is nearer to 8000 than
9000.

1.8.1 Problem Set 1.8


1. Round off 365 to the nearest ten.
2. Round off 14, 769. 3352 to the nearest hundred.
3. Round off 16, 745. 2583 to the nearest thousandth.

1.9 Limits of Accuracy

General Objectives: know that approximated and rounded off data have specified
tolerances.
Specific Objectives:
1. Give appropriate upper and lower bounds for data given to specified
accuracy (e.g., measured lengths)
2. Obtain appropriate upper and lower bounds to solutions of simple
problems (e.g., the calculation of the perimeter or the area of a rectangle)
given data to specified accuracy.
Limits of accuracy - discrete measures
Number recordings are not always exact, and in some cases they may be rounded
up.
When a number has been recorded to certain accuracy for instance, the nearest
1cm or the closest 10, you can work out its highest and lowest possible values
according to the limits of accuracy provided.
64

These outcomes are often referred to as an upper or lower bound.


Solved Example:
Q: These numbers have been rounded to the nearest 10, write down the largest
and smallest values they could be. Express your answers as inequalities in terms
of 𝑥:
a. 50 b. 80 c. 110
Solution: a.45 ≤ 𝑥 < 55 b.75 ≤ 𝑥 < 85 c.105 ≤ 𝑥 < 115
Example: These lengths have been rounded to the nearest tenth; write the
upper and lower limits. Express your answers as inequalities in terms of 𝑥:
a. 12.5cm b. 21.7cm c . 52.1cm
Solution: 12.45 ≤ 𝑥 < 12.55 21.65 ≤ 𝑥 < 21.75 52.05 ≤ 𝑥 < 52.15
Example:


65

1.9.1 Problem Set 1.9


1. Write down the limits of accuracy for 18kg which is given to 2 significant
figures.
2. Write down the limits of accuracy for 3.24m which is given to 2 decimal
places.
3. A car is 2.6m long, measured correctly to 1 decimal place. List the upper
and lower bounds for this measurement.
4. For example, a room is measured as 6.4 x 4.3 meters, measured to 1
decimal place. Calculate the Limits of Accuracy of the area of the room.
5. Peter is in training for a 400 meter race. He states that he can run 400
meters in 44 seconds. Both of these measurements are given to two
significant figures. Find his maximum speed.
66

1.10 Ratio, proportion and rate


General Objectives: Demonstrate an understanding of the elementary ideas and
notation of ratio, direct and inverse proportion and common measures of rate.
Specific Objectives:
1. Apply the ideas and notation of ratio, proportion and rate to practical
situations.
2. Divide a quantity in a given ratio.
3. Use scales in practical situations
4. Calculate average speed.
5. Increase and decrease a quantity by a given ratio.
6. Express direct and inverse variation in algebraic terms and use this form of
expression to find unknown quantities.
RATIO AND PROPORTION
A ratio is a comparison of two or more numbers that are usually of the same type
or measurement. If the numbers have different units, it is important to convert
the units to be the same before doing any calculations. We write the numbers in a
ratio with a colon (:) between them.
For example, if there are 8 learners who travel by bus and 12 learners who travel
by taxi, then we say we have a ratio of 8 learners travelling by bus to 12 learners
travelling by taxi. We can write this as 8 : 12. We can also simplify this ratio to
2 : 3, by dividing both parts by 4.

We will discuss this unit in the following sub headings:

● Ratio and terms of ratio

● Properties of ratio

● Ratio in the simplest form

● Simplification of ratio
67

● Comparison of ratio

● Dividing the given quantity in the given ratio

● Proportion

● Continued proportion

● Examples on ratio and proportion

Ratio
The ratio of two quantities 'a' and 'b' of the same kind and in the same units is a
fraction a/b which shows that how many times one quantity is of the other and is
written as a : b and is read as 'a is to b' where b ≠ 0.

Terms of the ratio

In the ratio a : b, the quantities a and b are called terms of the ratio. Here, 'a' is
called the first term or the antecedent and `b' is called the second term or
consequent.

Example:

In the ratio 5 : 9, 5 is called the antecedent and 9 is called the consequent.

Properties of ratio

If the first term and the second term of a ratio are multiplied or divided by the
same non-zero number, the ratio does not change.
68

● 𝑎/𝑏 = 𝑥𝑎/𝑥𝑏, (𝑥 ≠ 0) 𝑆𝑜, 𝑎 ∶ 𝑏 = 𝑥𝑎 ∶ 𝑥𝑏

● 𝑎/𝑏 = (𝑎/𝑥)/(𝑏/𝑥), (𝑥 ≠ 0) 𝑆𝑜, 𝑎 ∶ 𝑏 = 𝑎/𝑥 ∶ 𝑏/𝑥

Ratio in the simplest form

A ratio 𝑎 ∶ 𝑏 is said to be in the simplest form if 𝑎 and 𝑏 have no common factor


other than 1.

Example:

Express 15 : 10 in the simplest form.

Solution:

15
10

15 ÷5
=
10 ÷5

3
= (In this we cancelled the common factor 5)
2

Thus, we have expressed the ratio 15/10 in the simplest form, i.e., 3/2 and the
terms 3 and 2 have common factor only number 1.

Note:

● In ratio, quantities being compared must be of the same kind, otherwise the
comparison becomes meaningless.

For example; comparing 20 pens and 10 apples is meaningless.

● They must be expressed in the same units.

● In a ratio, order of the terms is very important. The ratio a: b is different from

b : a.
69

● The ratio has no units.

For example; Dozen = 12, Gross = 144, Score = 20

Decade = 10, Century = 100, Millennium = 1000

Example:

Express the following ratios in the simplest form.

(a) 64 cm to 4.8 m

(b) 36 minutes to 36 seconds

(c) 30 dozen to 2 hundred

Solution:

64 𝑐𝑚
(a) Required ratio =
4.8 𝑚

64 𝑐𝑚
= (4.8
×100) 𝑐𝑚

64 𝑐𝑚
=
480 𝑐𝑚

64
=
480

2
=
15

= 2 : 15

36 𝑚𝑖𝑛𝑢𝑡𝑒𝑠
(b) Required ratio =
36 𝑠𝑒𝑐𝑜𝑛𝑑𝑠

36 ×60 𝑠𝑒𝑐𝑜𝑛𝑑𝑠
=
36 𝑠𝑒𝑐𝑜𝑛𝑑𝑠
70

60
=
1

= 60 ∶ 1

30 𝑑𝑜𝑧𝑒𝑛
(c) Required ratio =
2 ℎ𝑢𝑛𝑑𝑟𝑒𝑑

30 ×12
=
2 ×100

9
=
5

=9∶5

Simplification of ratio

If the terms of the ratio are expressed in fraction form; then find the Least
Common Multiple of the denominators of these fractions. Now, multiply each
fraction by the L.C.M, then ratio is simplified.

Example:

Simplify the following ratios.

5 3 4
(a) ∶ ∶
2 8 9

1 2
(b)2 ∶3
7 5

Solution:

(a) The L.C.M. of 2, 8, 9 = 2 × 2 × 2 × 3 × 3

=8×9

= 72

Now, multiplying each fraction by the L.C.M.


71

5 3 4
× 72 = 180 × 72 = 27 × 72 = 32
2 8 9

So, the ratio becomes 180 : 27 : 32


1 2
(b) 2 ∶3
7 5

15 17
= :
7 5

15 5
= ×
7 17

75
=
119

So, the ratio becomes 75 : 119

Comparison of ratios
Ratios can be compared as fractions. Convert them into equivalent ratios as we
convert the given fractions into equivalent fractions and then compare.

Example:

Which ratio is greater?

1 1 4 3
2 ∶3 , 2.5 : 3.5 , ∶
3 2 5 2

Solution:

Simplifying the given 3 ratios

1 1 7 7 7 7 7 2 14 2
2 ∶3 = ∶ = ÷ = × = =
3 2 3 2 3 2 3 7 21 3

25 5
2.5 : 3.5 = =
35 7

4 3 4 2 8
: = × =
5 2 5 3 15
72

𝟐 𝟓 𝟖
, ,
𝟑 𝟕 𝟏𝟓

L.C.M. of 3, 7, 15 = 105

2 2×35 70
= = (divide 105 by 3 to get 35)
3 3×35 105

5 5×15 75
= = (divide 105 by 7 to get 15)
7 7×15 105

8 8×7 56
= = (divide 105 by 15 to get 7)
15 15×7 105

75 70 56
𝑇ℎ𝑒𝑟𝑒𝑓𝑜𝑟𝑒, > >
105 105 105

5 2 8
so, > >
7 3 15

𝟏 𝟏 𝟒 𝟑
Therefore, 2.5 : 3.5 > 𝟐 ∶𝟑 > ∶
𝟑 𝟐 𝟓 𝟐

Dividing the given quantity in the given ratio

If 'p’ is the given quantity to be divided in the ratio 𝑎 ∶ 𝑏, then add the terms of
the a ratio, i.e., 𝑎 + 𝑏, then the 1ˢᵗ part = {𝑎/(𝑎 + 𝑏)} × 𝑝 and 2ⁿᵈ part
{𝑏/(𝑎 + 𝑏)} × 𝑝

Example:

1 1 3
Divide N$290 among A, B, C in the ratio 1 ,1 and
2 4 8

Solution:

𝟑 𝟓 𝟑
Given ratios = : :
𝟐 𝟒 𝟖

The L.C.M. of 2, 4, 8 is 8.
73

𝟑 𝟓 𝟑
So we have × 8 : ×8 ∶ × 8 = 12 ∶ 10 : 3 so, sum of 12+10+3=25
𝟐 𝟒 𝟖

𝟏𝟐
Therefore, Share of A = × 290 = N$139.20
𝟐𝟓

𝟏𝟎
Share of B = × 290 = N$116.00
𝟐𝟓

𝟑
Share of C = × 290 = N$34.80
𝟐𝟓

Check: 139.20 + 116 + 34.80 = N$ 290

Proportion
We have already learnt that statement of equality of ratios is called proportion, if
four quantities a, b, c, d are in proportion, then a : b = c : d or a : b : : c : d (: : is the
symbol used to denote proportion).

⇒ a/b = c/d

⇒a×d=b×c

⇒ ad = bc

Here a, d are called the extreme terms in which a is called the first term and d is
called the fourth term and b, c are called the mean terms in which b is called
the second term and c is called the third term.

Thus, we say, if product of mean terms = the product of extreme terms, then the
terms are said to be in proportion.

Also, if a : b :: c : d, then d is called the fourth proportional of a, b, c.

Continued Proportion
The three quantities a, b, c are said to be in continued proportion if a : b :: b : c

⇒ a/b = b/c
74

⇒ a × c = b²

⇒ b² = ac

⇒ b = √ac

Here, b is called the mean proportional of a and c. The square of middle term is
equal to the product of 1ˢᵗ term and 3ʳᵈ term.

Also, if a : b :: b : c, then c is called the third proportional of a, b.

Example:

Determine if the following are in proportion.

(a) 6, 12, 24

2 1 4 5
(b) 1 ,6 , ,
3 4 9 3

Solution:

(a) Here, product of first term and third term = 6 × 24 = 144 and square of middle
term = (12) ² = 12 × 12 = 144

2 1 4 5
(b) 1 , 6 , ,
3 4 9 3

2 1 4 5
Here, a = 1 b=6 c= d=
3 4 9 3

2 1 4 5
a:b=1 :6 c:d= :
3 4 9 3

5 25 4 5
= ∶ = ÷
3 4 9 3

5 25 4 3
= ÷ = ×
3 4 9 5
75

5 4 12
= × =
3 25 45

4 4
= =
15 15

Since, a : b = c : d

2 1 4 5
Therefore, 1 , 6 , , are in proportion.
3 4 9 3

Solved Examples:

Example: Arrange the following ratios in descending order.

2 : 3, 3 : 4, 5 : 6, 1 : 5

Solution:

2 3 5 1
Given ratios are , , ,
3 4 6 5

The L.C.M. of 3, 4, 6, 5 is 2 × 2 × 3 × 5 = 60

2 2 ×20 40
Now, = = (divide 60 by 3 to get 20)
3 3 ×20 60

3 3 ×15 45
= = ( divide 60 by 4 to get 15)
4 4 ×15 60

5 5 ×10 50
= = (divide 60 by 6 to get 10)
6 6 ×10 60

1 1 ×12 12
= = (divide 60 by 5 to get 12)
5 5 ×12 60

50 45 40 12
Clearly, > > >
60 60 60 60

5 3 2 1
Therefore, > > >
6 4 3 5

So, 5 : 6 > 3 : 4 > 2 : 3 > 1 : 5


76

Example: Two numbers are in the ratio 3 : 4. If the sum of numbers is 63, find the
numbers.

Solution:

Sum of the terms of the ratio = 3 + 4 = 7

Sum of numbers = 63

3
Therefore, first number = × 63 = 27
7

4
Second number = × 63 = 36
7

Therefore, the two numbers are 27 and 36.

Example: If 𝑥 ∶ 𝑦 = 1 : 2, find the value of (2𝑥 + 3𝑦) ∶ (𝑥 + 4𝑦)

Solution:

𝑥 : 𝑦 = 1 : 2 means 𝑥/𝑦 = 1/2

Now, (2𝑥 + 3𝑦) : (𝑥 + 4𝑦) = (2𝑥 + 3𝑦)/(𝑥 + 4𝑦) [Divide numerator and
denominator by 𝒚.]

= [(2𝑥 + 3𝑦)/𝑦]/[(𝑥 + 4𝑦)/2] = [2(𝑥/𝑦) + 3]/[(𝑥/𝑦) + 4], put 𝑥/𝑦 = 1/2

We get = [2 (1/2) + 3)/(1/2 + 4) = (1 + 3)/[(1 + 8)/2] = 4/(9/2) = 4/1 × 2/9 = 8/9

Therefore the value of (2𝑥 + 3𝑦) : (𝑥 + 4𝑦) = 8 : 9

More solved problems on ratio and proportion are explained here with full
description.

Example: A bag contains N$510 in the form of 50 p, 25 p and 20 p coins in the


ratio 2 : 3 : 4. Find the number of coins of each type.
77

Solution:
Let the number of 50 p, 25 p and 20 p coins be 2𝑥, 3𝑥 and 4𝑥.

2𝑥 × 50 3𝑥 × 25 4𝑥 × 20
Then + + = 510
100 100 100

𝑥 3𝑥 4𝑥
+ + = 510
1 4 5

20𝑥 + 15𝑥 + 16𝑥


= 510
20
51𝑥
⇒ = 510
20

510 × 20
𝑥=
51

𝑥 = 200

2𝑥 = 2 × 200 = 400

3𝑥 = 3 × 200 = 600

4𝑥 = 4 × 200 = 800.

Therefore, number of 50 p coins, 25 p coins and 20 p coins are 400, 600, 800
respectively.

Example: If 2A = 3B = 4C, find A : B : C

Solution:

Let 2A = 3B = 4C = 𝑥
𝑥 𝑥 𝑥
So, A = B = C =
2 3 4
78

The L.C.M of 2, 3 and 4 is 12


𝑥 𝑥 𝑥
Therefore, A : B : C = × 12 : × 12 : × 12
2 3 4

= 6𝑥 : 4𝑥 : 3𝑥

=6:4:3

Therefore, A : B : C = 6 : 4 : 3

More worked out problems on ratio and proportion are explained here step-by-
step.

Example: Mother divided the money among Ron, Sam and Maria in the ratio 2 : 3
: 5. If Maria got N$150, find the total amount and the money received by Ron and
Sam.

Solution:

Let the money received by Ron, Sam and Maria be 2𝑥, 3𝑥, 5𝑥 respectively.

Given that Maria has got N$ 150.

Therefore, 5𝑥 = 150

150
or, 𝑥 =
5

or, 𝑥 = 30

So, Ron got = 2𝑥

= N$ 2 × 30 = N$60

Sam got = 3𝑥

= N$3 × 30 = N$90
79

Maria got = 5𝑥

=5×30= N$150

Therefore, the total amount N$(150 + 60 + 90) = N$300


1
Example: Divide N$370 into three parts such that second part is of the third part
4
and the ratio between the first and the third part is 3 : 5. Find each part.

Solution:

Let the first and the third parts be 3𝑥 and 5𝑥.

1
Second part = of third part.
4

1
= × 5𝑥
4

5𝑥
=
4
5𝑥
Therefore, 3𝑥 + + 5𝑥 = 370
4
12𝑥 +5𝑥 + 20𝑥
= 370
4

37𝑥
= 370
4

370 × 4
𝑥=
37

1480
𝑥=
37

𝑥 = 40

Therefore, first part = 3𝑥

= 3 × 40

= N$120.00
80

5𝑥
Second part =
4

5 ×40
=
4

= N$50.00

Third part = 5𝑥

= 5 × 40

= N$ 200.00

More worked out problems on ratio and proportion using step-by-step


explanation.

Example: Set up all possible proportions from the numbers 8, 12, 20, 30.

Solution:

We note that 8 × 30 = 240 and 12 × 20 = 240

Thus, 8 × 30 = 12 × 20 ………..(I)

Hence, 8 : 12 = 20 : 30 ……….. (i)

We also note that, 8 × 30 = 20 × 12

Hence, 8 : 20 = 12 : 30 ……….. (ii)

(I) can also be written as 12 × 20 = 8 × 30

Hence, 12 : 8 = 30 : 20 ……….. (iii)

Lastly (I) can also be written as


81

12 : 30 = 8 : 20 ……….. (iv)

Thus, the required proportions are 8 : 12 = 20 : 30

8 : 20 = 12 : 30 12 : 8 = 30 : 20 12 : 30 = 8 : 20

DIRECT AND INVERSE VARIATION:

Direct Variation:

If two variables A and B are so related that when A increases ( or decreases ) in a


given ratio, B also increases ( or, decreases ) in the same ratio, then A is said to
vary directly as B ( or, A is said to vary as B ). This is symbolically written as, A ∝ B
(read as, ‘A varies as B’ ). Suppose a train moving at a uniform speed travels 𝑑 km.
in 𝑡 minutes. Now, consider the following table:

𝑑 (km) 24 12 48 36

𝑡 (min) 30 15 60 45

Inverse Variation:

A variable quantity 𝐴 is said to vary inversely as another variable quantity 𝐵,


1
when 𝐴 varies as the reciprocal of 𝐵 i.e., when 𝐴 varies as .
𝐵

1 1
Thus, if 𝐴 varies inversely as 𝐵, we write A ∝ or, A = m ∙( ) or, AB = 𝑚 where 𝑚
𝐵 𝐵
(≠ 0) is the constant of variation. Hence, if one variable varies inversely as
another, then the product of the corresponding values of the variables is
constant.

Conversely, if 𝐴𝐵 = 𝑘 where 𝐴 and 𝐵 are variables and 𝑘 is a constant, then A ∝


1
Hence, if the product of the corresponding values of two variables is constant,
𝐵
then one quantity varies inversely as another.

Again, if 𝐴 varies inversely as 𝐵 then AB = constant; but AB = constant implies that


when 𝐴 increases in a given ratio, 𝐵 decreases in the same ratio and vice – versa.
82

Thus, if two variables are so related that an increase (or decrease) in the value of
one variable in a certain ratio corresponds to a decrease (or increase) in the same
ratio in the value of the other variable then one variable varies inversely as
another.

Let 𝑎 𝑚. And 𝑏 𝑚. Be the length and breadth 𝑟 area 160 m². Now, consider the
following table:

𝑎 𝑚. 20 16 40

𝑏 𝑚. 8 10 4

Step-by-step explanation on worked-out examples on variation:

Example: (i) If A varies inversely as B and A = 2 when B = 10, find A when B = 4.

(ii) If 𝑥 ∝ 𝑦² and 𝑥 = 8 when 𝑦 = 4, find 𝑦 when 𝑥 = 32.

Solution: (i) Since A vary inversely as B

1 1
Therefore A ∝ or, A = k ∙ ………………. (1), where k = constant of variation.
𝐵 𝐵

Given A = 2 when B = 10.

Putting these values in (1), we get,

1
2=k∙
10
(Note: a dot · Sign Means Multiplication)
or, k = 20.
1
Therefore, the law of variation is: A = 20 ∙ ……………... (2)
𝐵

1
When B = 4, then from (2) we get, A = 20 ∙ =5
4

Therefore, A = 5 when B = 4.
83

(ii) Since, 𝑥 ∝ 𝑦²

Therefore, 𝑥 = m ∙ 𝑦² ……………… (1)

where m = constant of variation.

Given 𝑥 = 8 when 𝑦 = 4.

Putting these values in (1), we get,

8 = m ∙ 4² = 16m

8
or, m =
16

1
or, m =
2

1
Therefore the law of variation is: 𝑥 = ∙ 𝑦² ………….. (2) When 𝑥 = 32, then from (2)
2
we get,

1
32 = ∙ 𝑦²
2

or, 𝑦² = 64

or, 𝑦 = ± 8.

Hence, 𝒚 = 8 or, - 8 when 𝒙 = 32.

Example: 𝑥 varies directly as the square of 𝑦 and inversely as the cube root of 𝑧
and 𝑥 = 2, when 𝑦 = 4, 𝑧 = 8. What is the value of 𝑦 when 𝑥 = 3, and 𝑧 = 27?

Solution:

By the condition of the problem, we have,

1
𝑥 ∝ 𝑦² ∙ 3
√𝑧
84

1
Therefore 𝑥 = k ∙ 𝑦² ∙ 3 ……(1)
√𝑧

where k = constant, of variation.

Given 𝑥 = 2 when 𝑦 = 4, 𝑧 = 8.

Putting these values in (1), we get,

1 1
2 = k ∙ 4² ∙ 3 = k ∙ 16 ∙ = 8k
√8 2

2 1
or, k = =
8 4

1 1
Therefore the law of variation is: 𝑥 = ∙ 𝑦² ∙ 3 .... (2)
4 √𝑧

When 𝑥 = 3, 𝑧 = 27, then from (2) we get,

1 1 1 1
3= ∙ 𝑦² ∙ 3 = ∙ 𝑦² ∙
4 √27 4 3

or, 𝑦² = 36

or, 𝑦 = ± 6

Therefore, the required value of y is 6 or - 6.

Example: Fill in the gaps:

(i) If A ∝ B² then B ∝ …..


1
(ii) If P ∝ , then 𝑄 ∝ ……
√𝑄

(iii) If m ∝ ∛n, then n ∝ ……


85

Solution:

(i) Since A ∝ B²

Therefore, A = kB² [k = constant of variation]

1
or, B² =( )A
𝑘

1
or, B = ± ( ) √A
√𝑘

1
Therefore B ∝ √A since ± = constant.
√𝑘

1
(ii) Since p ∝
√𝑄

1
Therefore p = k ∙ [k = constant of variation]
√𝑄

𝐾
Since, √𝑄 =
𝑃

𝐾²
or, 𝑄 =
𝑃²

1
Therefore, Q ∝ , as k² = constant.
𝑃²

(iii) Since, m ∝ ∛n

Therefore m = k ∙ ∛n [k = constant of variation]

or, m³ = k³ ∙ n

1
or, n = ( ) ∙ m³
𝐾³

1
Therefore n ∝ m³ as = constant.
𝐾³
86

1.10.1 Problem Set 1.10


1. Which of these pairs of ratios are equal?
a) 3:4 and 75:10
b) 2:3 and 10:20
c) 5:1 and 100:20
d) 10:1 and 40:5
2. A fruit and nut company has the following standards requirement: In a
packet of dried fruit and nuts, there must be two hundred grams of fruit for
every 50g of nuts.
a) Write this as a simple ratio
b) What will the amount of fruit be if there are 500g of nuts?
c) What will the amount of fruit be if there are 25g of nuts?

3. The ratio of female learners to male learners in a class is 3:2. If there are 30
female learners in the class, work out:
a) The number of male learners
b) The total number of learners in the class

4. If two pencils cost N$1.50 all together, how many pencils can you buy with
N$9.00?

1.11 Use of an electronic calculator

General Objectives: know how and when to use an electronic calculator


Specific Objectives:
1. Use an electronic calculator efficiently
2. Apply appropriate checks of accuracy
Sometimes the computations are somewhat time-consuming, like long division, or
involve square roots. For such computations, one can use an electronic calculator.
87

1.11.1 Problem Set 1.11


1. At a fund-raising event, 43 participants donated N$60 each, 21 participants
donated N$80 each and 16 participants donated N$100 each. What was the
average (arithmetic mean) donation per participant, in dollars? (Use
electronic calculator to solve this).
2. Compute (−15)3
3. Find the length, to the nearest 0.01, of the hypotenuse of a right angled
triangle with legs of length 21 and 54; that is, use the Pythagorean theorem
and calculator for this purpose.
4. Calculate sin (50ᣞ).
5
5. Calculate the integral ∫2 (𝑥 3 − 5𝑥 − 3)𝑑𝑥

1.12 Money and Finance


General Objectives: do calculations using money and understand issues involving
personal and household finances
Specific Objectives:
1. Solve problems involving money and convert from one currency to another
2. Use given data to solve problems on personal and household finance
involving earnings, simple interest, compound interest (knowledge of the
formula is not required, can be provided in examination), discount, profit
and loss, tax and budgeting.
3. Extract data from tables and charts.
Some of the useful terms that will be used in this section are as follows:
To find the interest for any given period of time, multiply the principal by
the rate by the time.
Formula I = P x R x T

To find the amount, add the interest (I) to the principal (P)
Formula: A = P + I
88

Rate ( R ) or rate percent is the fractional part in hundredths. The base ( B )


is the whole quantity of which some percent is to be found.

To find the amount of discount, multiply the list price, or base, by the rate
of discount.
Formula: P = B x R

To find the net price, subtract the amount of the discount from the original
or list price.
Formula D = B-P

Interests

Interest is of two types, simple interest and compound interest.

Simple interest: When interest is calculated on principal amount, that


interest is called Simple interest.

Compound interest: After a definite period of time, the interest accrued is


added to the previous principal to get the new principal. For the next
definite period of time, if the interest is calculated on the new principal,
then that interest is called Compound interest.

Phase of compound interest: The period of time in each stage after which
the compound interest is due, is called the phase of compound interest.
Usually, the phases of compound interest are calculated for 3 months, 6
months and 1 year.

Amount when subjected to compound interest: The sum of the principal


and compound interest for a specific period of time is known as amount.
89

By calculating compound interest using simple interest only ,we can


calculate simple interest on a certain Principal (N$ P), at a certain Rate (R
%) per annum for a given period of time (T years), then the simple interest
is given by the formula:

S.I.=P×R×T

When money is borrowed on simple interest then the interest is calculated


uniformly on the original sum (principal) throughout the period of a loan.
But in everyday life, the interest charged or paid is rarely simple interest.

However, in post offices, banks, insurance corporations and other


companies which lend money and accept deposits, the method of
calculating interest is quite different.

Under this method, the borrower and the lender agree to fix a certain unit
of time, say one year or a half-year or one quarter of a year (i.e., 3
months), to settle the previous account.

In such cases, the interest accrued during the first unit of time is added to
the original principal and the amount obtained is taken as the principal for
the second unit of time. The amount of this principal at the end of the
second unit of time becomes the principal for the third unit of time, and so
on.

After a certain specified period, the difference between the amount and
the money borrowed is called the compound interest (abbreviated as CI).

The fixed unit of time is known as the conversion period.

To Find Compound Interest (CI) when Interest is Compounded Annually

In such cases where interest is compounded yearly, the interest accrued


during the first year is added to the principal and the amount so obtained
becomes the principal for the second year. The amount at the end of the
second year becomes the principal for the third year, and so on.

Solved problems to find Compound Interest when Interest is


Compounded Annually:
90

Example: Find the compound interest on N$5000 for 3 years at 8% per


annum, compounded annually.

Solution:

Principal for the first year =N$5000

8
Interest for the first year = N$5000 × = N$400.00
100

Amount at the end of the first year N$(5000 + 400) = N$5400.00

Principal for the second year = N$5400.00

8
Interest for the second year = N$5400 × = N$432.00
100

Amount at the end of the second year = N$(5400 + 432) = N$5832.00

Principal for the third year = N$5832.00

8
Interest for the third year = N$5832 × = N$466.56
100

Amount at the end of the third year = N$(5832 + 466.56) = N$6298.56

Therefore, compound interest = N$(6298.56 - 5000) = N$1298.56

Example: Find the compound interest on N$25000 for 3 years at 6% per


annum, compounded annually.

Solution:

Principal for the first year = N$25000.00

6
Interest for the first year = N$25000 × = N$1500.00
100

Amount at the end of the first year N$(25000 + 1500) = N$26500.00


91

Principal for the second year = N$26500.00

6
Interest for the second year = N$26500 × = N$1590.00
100

Amount at the end of the second year = N$(26500 + 1590) = N$28090.00

Principal for the third year = N$28090.00

6
Interest for the third year = N$28090 × = N$1685.40
100

Amount at the end of the third year = N$(28090 + 1685.40) =N$29775.40

Therefore, compound interest = N$(29775.40 - 25000) = N$4775.40

To Find Compound Interest (CI) when Interest is Compounded Half-


Yearly
𝑅
In such cases, if the rate of interest is R% per annum then it is clearly ( )%
2
per half-year.

The amount after the first half-year becomes the principal for the next half-
year, and so on.
The method for calculating compound interest in such cases is shown in
the example given below.

Solved problems to find Compound Interest when Interest is


Compounded Half-Yearly

Example: Find the compound interest on N$5000 for 1 year at 8% per


annum, compounded half-yearly.

Solution:

Rate of interest = 8% per annum

= 4% per half-year.
92

Time = 1 year = 2 half-years

Original principal = N$5000.00

4
Interest for the first half-year = N$5000 × = N$200.00
100

Amount at the end of the first half-year = N$(5000 + 200) = N$5200.00

Principal for the second half-year = N$5200.00

4
Interest for the second half-year = N$5200 × = N$208.00
100

Amount at the end of the second half-year = N$(5200 + 208) = N$5408.00

Therefore, compound interest = N$(5408 - 5000) = N$408.00

Example: Find the compound interest on N$10000 for 1 year at 10% per
annum, compounded half-yearly.

Solution:

Rate of interest = 10% per annum

= 5% per half-year.

Time = 1 year = 2 half-years

Original principal = N$10000.00

5
Interest for the first half-year = N$10000 × = N$500.00
100

Amount at the end of the first half-year = N$(10000 + 500) = N$10500.00

Principal for the second half-year = N$10500.00


93

5
Interest for the second half-year = N$10500 × = N$525.00
100

Amount at the end of the second half-year =N$(10500 + 525)= N$11025.00

Therefore, compound interest = N$(11025 - 10000) = N$1025.00

And therefore, the formula for calculating the compound and simple
interest is: P=Principal

𝑹 n R= Rate% per anum


Compound Interest (CI) = P (𝟏 + )
𝟏𝟎𝟎 n=years

Simple Interest (SI) = 𝑷𝒓𝒊𝒏𝒄𝒊𝒑𝒂𝒍 (𝑷) × 𝑹𝒂𝒕𝒆 (𝑹) × 𝑻𝒊𝒎𝒆 (𝑻)

Amount (A) = Principal (P) + Interest (I)

Principal (P) = Amount (A) – Interest (I)

Interest (I) = Amount (A) – Principal (P)

Example: Richard deposits N$ 5400 and got back an amount of N$ 6000


after a year. Find the simple interest he got.

Solution:

Principal (P) = N$ 5400.00

Amount (A) = N$ 6000.00

Simple Interest (SI) = Amount (A) – Principal (P)

= 6000 - 5400
94

= 600
Therefore, Richard got an interest of N$ 600.00
Marked Price:

In big shops and departmental stores, every item is tagged with a card and
its price is written on it. This is called the marked price of that item,
abbreviated as MP.
For books, the printed price is the marked price.

List Price:

Items which are manufactured in a factory are marked with a price


according to the list supplied by the factory, at which the retailer is
supposed to sell them. This price is known as the list price of the item.

Discount:

In order to increase the sale or clear the old stock, sometimes the
shopkeepers offer a certain percentage of rebate on the marked price. This
rebate is known as discount.
Notes:

The discount is always reckoned on the marked price.

Selling price = (marked price) - (discount)

As:
'selling price' is the amount you actually pay for the item when you
purchase.

'marked price' is the general price of the item without any discount.

'discount' is a percentage of the marked price.

Worked-out problems related on discount, selling price and marked price


95

based on the above formula are discussed below with a detailed


explanation:

Example: The marked price of a ceiling fan is N$ 1250 and the shopkeeper
allows a discount of 6% on it. Find the selling price of the fan.

Solution:

Marked price = N$ 1250 and discount = 6%.

Discount = 6% of Marked Price

= (6% of N$ 1250)

6
= N$1250 ×
100

= N$ 75.00

Selling price = (Marked Price) - (discount)

= N$ (1250 - 75)

= N$ 1175.00

Hence, the selling price of the fan is N$ 1175.00

Example: A trader marks his goods at 40% above the cost price and allows
a discount of 25%. What is his gain percent?

Solution:

Let the cost price be N$ 100

Then, marked price = N$ 140

Discount = 25% of Marked Price

= (25% of N$ 140)
96

25
= N$140 ×
100

= N$ 35.00

Selling price = (marked price) - (discount)

= N$ (140 - 35)

= N$ 105.00

Gain% = (105 - 100) % = 5%.

Hence, the trader gains 5%.

Example: A dealer purchased a washing machine for N$ 7660. He allows a


discount of 12% on its marked price and still gains 10%. Find the marked
price of the machine.

Solution:

Cost price of the machine = N$ 7660, Gain% = 10%.

(100 + gain%)
Therefore, selling price = × CP
100

(100 + 10)
= N$ × 7660
100

110
= N$ × 7660
100

= N$ 8426.00

Let the marked price be N$ 𝑥.

Then, the discount = 12% of N$𝑥


97

12
= N$ 𝑥 ×
100

3𝑥
= N$
25

Therefore, SP = (Marked Price) - (discount)

= N$ 𝑥 − (3𝑥)/25 = (25𝑥 − 3𝑥) / 25

= N$ 22𝑥/25

But, the SP = N$ 8426.00

Therefore, 22𝑥/25 = 8426

⇒ 𝑥 = (8426 × 25/22)

⇒ 𝑥 = 9575

Hence, the marked price of the washing machine is N$ 9575.00

Successive Discounts:

If two or more discounts are allowed one after the other then such
discounts are known as successive discounts or discounts in series.

Suppose a discount of 25% is given on an item. Then on the reduced price a


discount of 15% is given. In such a case, we say that successive discounts of
25% and 15% are given.

Example: Find the single discount equivalent to two successive discounts of


20% and 10%.

Solution:
98

Let the marked price of an item be N$ 100

Then, first discount on it = N$ 20

Price after first discount = N$ (100 - 20) = N$ 80

Second discount on it = 10% of N$ 80

10
= N$ 80 × = N$ 8
100

Price after second discount = N$ (80 - 8) = N$ 72

Net selling price = N$ 72.00

Single discount equivalent to given successive discounts = (100 - 72)% =


28%.

Changing local currency to foreign currency


When you go abroad you need to change your money to the currency of
the country you visit.
A table of exchange rates is shown on the following page.
99

Namibia 16.34 N$

To convert N$ to foreign currency, Divide it by the exchange rate.


FOREIGN CURRENCY = AMOUNT in N$ ÷ EXCHANGE RATE
Example:
Anna is going to England on holiday and changes N$20000 to pounds (£).
How many Pounds does she get?
The exchange rate for one Pound (£) is N$16.34
So, N$ 20000 ÷ 16.34 = £ 1223.99
Anna gets £ 1223.99

CHARTS AND PLOTTING DATA:

In a pie chart, the various observations or components are represented by


the sectors of a circle and the whole circle represents the sum of the values
of all components.

The central angle for a component is given by:

Central angle for a component =( Value of the component/Sum of the


values of all components )× 360°
100

How to make a pie chart or graph?

Construction of a pie chart or graph from the given data.

Steps of pie chart construction:

1. Calculate the central angle for each component, given by

Central angle of component = (Value of the component/Total value )× 360°

2. Draw a circle of convenient radius.


3. Within this circle, draw a horizontal radius.
4. Starting with the horizontal radius, draw radii making central angles
corresponding to the values of the respective components, till all the
components are exhausted. These radii divide the whole circle into various
sectors.
5. Shade each sector with different design.

This will be the required pie chart for the given data.
Example on how to construct a pie chart:

Example: Mr. Peter, with a yearly salary of N$ 10800 plans his budget for a
year as given below:

Item Food Education Rent Savings Miscellaneous


Amount (in N$) 3150 1950 2100 2400 1200

Represent the above data by a pie chart.

Solution:

Total amount earned by Mr. Peter in a year = N$ 10800.00


Central angle of component = Value of the component/Total value × 360°

Calculation of central angles :


101

Item Amount (in $) Central Angle


3150
Food 3150 (10800 × 360) ° = 105°
1950
Education 1950 (10800 × 360) ° = 65°
2100
Rent 2100 (10800 × 360) °= 70°
2400
Savings 2400 (10800 × 360) °= 80°
1200
Miscellaneous 1200 (10800 × 360) °= 40°

Construction of a pie chart

Items

Education Rent Rent


65° 70°
Food
Misc
40° Savings
Food
105° Misc
Savings
80° Education

Steps of construction:

1. Draw a circle of any convenient radius.

2. Draw a horizontal radius of this circle.

3. Draw sectors starting from the horizontal radius with central angles of
105 degree, 65 degree, 70 degree, 80 degree and 40 degree respectively.

4. Shade the sectors differently using different colors and label them.

Thus, we obtain the required pie chart, as shown in the above figure.
102

1.12.1 Problem Set 1.12


1. A manufacturer gains 25% after allowing a trade discount of 15% of the list
price of an article. The cost of manufacturing the article is increased by 25%
and the catalog price is increased by 20% only. Find the new profit percent
keeping the same rate of discount.
2. Find the simple interest on N$900 for 2 years at 6%.
3. Find the interest and amount of N$400 for 3 years, 3 months and 10 days at
6%.
4. A real estate agent sold a piece of property for $50,000. His commission is
7.5%. How much does he receive?
5. Change 25% to a decimal.
6. Change 1.5% to a decimal.
7. A used car listed at N$ 925.00 was sold at a discount of 10%. Find the
discount.
8. A file cabinet was listed at N$62.50. Find the net price if the discount was
12%.
9. A damaged chair that cost N$110 was sold at a loss of 10%. Find the loss
and the selling price.
10. A bracelet was sold at a profit of $9000. The rate of profit was 30%. What
was the cost of the bracelet?
103

Chapter 2
2.1 SI units of measures

General Objectives: use units of measure in practical situations


Specific Objectives:
1. Use SI units of mass, length, area, volume and capacity in practical
situations.
2. Express quantities in larger or smaller units.
3. Calculate time in terms of the 24-hour and 12-hour clock (notation for time
e.g., 14:00 or 2:00 pm)
4. Read clocks, dials and timetables
2.1.1 SI units
Scientists all over the world use the same system of units so they can
communicate information clearly. This system of measurement is called the
International System of Units (SI). Metric measurement is based on the number
ten and makes calculations with the system relatively easy.
The unit for mass is gram (g).
The unit for length is meter (m).
The unit for volume is liter (L).
MEASUREMENT
Measurement of any physical quantity involves comparison with a certain basic,
arbitrarily chosen, internationally accepted reference standard called unit. The
result of measurement of a physical quantity is expressed by a number (or
numerical measure) accompanied by a unit. Although the number of physical
quantities appears to be very large, we need only a limited number of units for
expressing all the physical quantities, since they are interrelated with one
another. The units for the fundamental or base quantities are called fundamental
or base units. The units of all other physical quantities can be expressed as
combinations of the base units. Such units obtained for the derived quantities are
104

called derived units. A complete set of these units, both the base units and
derived units, is known as the system of units.
SI UNIT OF LENGTH:
The SI unit of length is meters (m) and the calculation can be done by various
units.

Name Symbol Equivalence


Kilometer Km 1000 m
hectometer hm 100 m
decameter dam 10 m
meter m 1m
decimeter dm 0.1 m
centimeter cm 0.01 m
millimeter mm 0.001 m

Note: Tips to convert an amount from one unit to another:

 If the original unit is less than the one we want to get, the amount will be
divided by 10 as many times as the number of rows that have to be
“climbed” in the table above.
 If the original unit is larger than the one we want to get, the amount will be
multiplied by 10 as many times as the number of rows that have to be
“gone down” in the table above.

Example: If you want to convert 1400 meters into decameters: One meter is
less than a decameter therefore we have to divide 1400 by 10, once (because
we have to go up once from the meter to decameter) .Therefore 1400÷10
decameters. That is, 1400 meters gives you 140 decameters.
105

Measurement of Masses:
The SI unit of mass is Kilograms (kg). The other measuring units are listed in the
table below:

Name Symbol Equivalence


kilogram Kg 1000 g
hectogram hg 100 g
decagram dag 10 g
gram g 1g
decigram dg 0.1 g
centigram cg 0.01 g
milligram mg 0.001 g

Note: Tips to convert an amount from one unit to another:

 If the original unit is less than the one we want to get, the amount will be
divided by 10 as many times as the rows that have to be "climbed" in the table
above.
 If the original unit is larger than the one we want to get, the amount will be
multiplied by 10 as many times as the rows that have to be "gone down" in the
table above.

Example: convert 15 hectogram to decigram:


Solution: 15 hg = 15 × 1000 dg = 15000 dg(We are multiplying here by 1000 or
15 x 10 x 10 x 10 because Decigram is 3 rows down the table from hectogram).

Example: convert 15 hectogram to kilogram:


Solution: 15 hg = (15/10 ) kg = 1.5 kg(Here we are dividing the value by 10
because Kilogram is 1 row up the table from hectogram).
106

Measurement of Capacity:

Capacity is measured in Liter (L). The other measuring units are listed in the table
below:
Name Symbol Equivalence
kiloliter Kl 1000 l
hectoliter hl 100 l
decaliter dal 10 l
liter l 1l
deciliter dl 0.1 l
centiliter cl 0.01 l
milliliter ml 0.001 l

Note: Tips to convert an amount from one unit to another:

 If the original unit is less than the one we want to get, the amount will be
divided by 10 as many times as the rows that have to be "climbed" in the table
above.
 If the original unit is larger than the one we want to get, the amount will be
multiplied by 10 as many times as the rows that have to be "gone down" in the
table above.

Example: If you want to convert 400 milliliters to liters:


If we go from milliliters to liters we have to go up three rows, then we must divide
400 by (10 x 10 x 10),(which is the same as dividing by 1000). Therefore:
400/1000 liters.
So, in 400 milliliters there are 0.4 liters.
107

Measurement of Volume:

The Volume is measured in Cubic Meter (m³). The Other units are listed below:
Name Symbol Equivalence
cubic kilometer km³ 1 000 000 000 m³
cubic hectometer hm³ 1 000 000 m³
cubic decameter dam³ 1000 m³
cubic meter m³ 1 m³
cubic decimeter dm³ 0.001 m³
cubic centimeter cm³ 0.000 001 m³
cubic millimeter mm³ 0.000 000 001 m³

Note: Tips to convert an amount from one unit to another:

 If the original unit is less than the one we want to get, the amount will be
divided by 1000 as many times as the rows that have to be "climbed" in the
table above.
 If the original unit is larger than the one we want to get, the amount will be
multiplied by 1000 as many times as the rows that have to be "gone down" in
the table above.

Note: We divide the value by 1000 because of the unit of cube (³) involved in it.
So multiplying 10 three times we get 1000.

Example: If you want to convert 6000000 cubic centimeters into cubic


decimeters, you have to climb only one row, then it must divided once by1000:

So 6000000/1000 cubic decimeters.


Therefore 6000000 cubic centimeters are 6000 cubic decimeters

NOTE: The below table depicts the Standard SI units with the Standard units .
108

1. We can have multiple Values as we are using grams as the standard instead of
Kilograms.

2. One Important relation

1 Milliliter = 1 cubic Centimeter

The following Table depicts the representation of Different SI units.

Base quantity Name Symbol


Length meter m
Mass kilogram kg
Time second s
Volume cubic meter m3

Density kilogram per cubic meter kg/m3


specific volume cubic meter per kilogram m3/kg

2.1.2 Time Format


There are two main ways to represent time i.e., 24 hour clock or AM/PM. In 24
hour clock, the time is shown as to how many hours and minutes since midnight.
In AM/PM (12 hour format), the day is split into:
a) AM hours: hours running from midnight to noon
b) PM hours: hours running from noon to midnight
In terms of days and years, the following conversions are helpful:
1 Week = 7 Days 1 Hour = 60 Minutes
1 Day = 24 Hours 12 Months = 1 Year
Summary : There are two main ways to show the time: "24 Hour Clock" or
"AM/PM":
24 Hour Clock: The time is shown as how many hours and minutes since midnight.
109

AM/PM (or "12 Hour Clock"): the day is split into:

 12 Hours running from Midnight to Noon (the AM hours), and


 The other 12 Hours running from Noon to Midnight (the PM hours).

The below picture represents both Formats:

Converting from 12 hour time to 24 hour time


The following simple steps will help you change a 12 hour time to a 24 hour
time.

 If the hour is exactly 12pm, then simply remove the 'pm' label.
 If the hour is 12am, then change it to 00.
 The minutes and seconds never change when changing between 24 hour
and 12 hour time.
 Add 12 to any hour after Noon (and subtract 12 for the first hour of the
day):

For the first hour of the day (12 Midnight to 12:59 AM), subtract 12 Hours

Example: 12 Midnight =00:00, 12:35 AM = 00:35


 From 1:00 AM to 12:59 PM, there’s no change
Example: 11:20 AM = 11:20, 12:30 PM = 12:30
 From 1:00 PM to 11:59 PM, add 12 Hours
Example: 4:45 PM = 16:45, 11:50 PM = 23:50
110

More Examples
1:45am = 01:45 1:45pm = 13:45
4:20am = 04:20 4:20pm = 16:20
11:32am = 11:32 11:32pm = 23:32
12:07am = 00:07 12:07pm = 12:07

Converting from 24 hour time to 12 hour time


The following simple steps will help you change a 24 hour time to a 12 hour
time with 'am' and 'pm'.

 If the hour is exactly 12, then simply label it as a pm time.


 If the hour is 00, then change it to 12 and label it as an am time.
 The minutes and seconds never change when changing between 24 hour
and 12 hour time.
 For the first hour of the day (0:00 to 0:59), add 12 Hours, make it "AM"
Example: 00:10 = 12:10 AM, 00:40 = 12:40 AM
 From 1:00 to 11:59, just make it "AM"
Example: 1:15 = 1:15 AM, 11:25 = 11:25 AM
 From 12:00 to 12:59, just make it "PM"
Example: 12:10 = 12:10 PM, 12:55 = 12:55 PM
 From 13:00 to 23:59, subtract 12 Hours, make it "PM"
Example: 14:55 = 2:55 PM, 23:30 = 11:30 PM

Examples
14:36 = 2:36pm 02:12 = 2:12am
09:24 = 9:24am 17:48 = 5:48pm
00:45 = 12:45am 12:15 = 12:15pm
20:36 = 8:36pm 23:56 = 11:56pm
111

Note: "12 AM" and "12 PM" can cause confusion, so we prefer "12 Midnight" and
"12 Noon".

Units of Time Conversion Chart:

Units of time conversion chart are discussed here in hour, minute, second, day,
week, month, and year.

The standard units of time are :

1 hour = 60 minutes

1 minute = 60 seconds

1 hour = 60 minutes = 3600 seconds (60 × 60)

1 day = 24 hours

1 week = 7 days

1 year = 365 days

1 year = 12 months

1year = 52 weeks

These are the units of time conversion table.

For example:

1. How many minutes are there in a year?

Solution:

We know that:
112

1 year = 365 days.

1 day = 24 hours.
1 hour = 60 minutes.
So one year = (365 × 24 × 60) minutes.

= (8760 × 60) minutes.

= 525600 minutes.
2.1.3 Problem Set 2.1

1. From the list shown below, fill in the blanks using appropriate choices:
Density
Length
Mass
Time
Volume

a) g/mL
b) s
c) km
d) L
e) g
f) g/cm3
g) cm3
h) mm
i) mg

2. Convert the following measurements:

(a) 1000 mg = _____ g (b) 198g = _____ Kg (c) 8 mm = _____ cm


(d) 27.5 mg = _____ g (e) 923 cm = _____ m (f) 355 mL = _____ L
113

3. Interconvert the following time formats and fill in the blanks:


a) 14:25
b) 10:20 AM
c) 18:05
d) 12 Midnight
e) 11:50 PM
4. Convert the following and fill in the blanks for various units of time:
a) 20ns µs
b) 10ms s
c) 5h min
d) 3 weeks days
e) 7 Days hours
f) 36 hours seconds
114

Chapter 3
3.1 Perimeters, areas and volumes

General Objectives: do calculation involving the perimeter and area of simple


shapes, the surface area and volume of simple solid bodies.
Specific Objectives:
1. Carry out calculations involving
a) The perimeter and area of a rectangle and triangle
b) The circumference and area of a circle
c) The perimeter and area of a parallelogram and a trapezium
d) The volume of a cuboid, prism and cylinder
e) The surface area of a cuboid and a cylinder
2. Solve problems involving the arc length and sector area as fractions of the
circumference and area of a circle, the surface area and volume of a
sphere, pyramid and cone (given formulae for the sphere, pyramid and
cone)

3.1.1 Basic Geometrical Shapes and Formulae


Shape Features Geometrical Representation
Rectangle  Opposite sides are of
equal length
 All angles are of measure
115

=90ᣞ
 Perimeter= 2(h+b)
 Area= bh

Parallelogr
Parallelogram  Opposite sides are of
am equal length
 Opposite angles are
equal but not of 90ᣞ
 Perimeter= 2(s+b)
 Area= bh

Rhombus  All sides are of equal


length
 Opposite angles are
equal but not of 90ᣞ
 Perimeter= 4S
 Area= ½ (d1xd2)

Square  All sides are of equal


length
 All angles are of 90ᣞ
 Perimeter= 4S
 Area= S2 or ½ d2

Trapezoid  All sides are of different


lengths
 All angles are of different
116

measures
 Perimeter= S1+S2+S3+S4
 Area= ½ h (b1+b2)

Triangle  Sum of all interior angles


= 180ᣞ . Perimeter=Sum
of sides
 An angle and its opposite
side are proportional
 Area= ½ bh

Equilateral  All sides are of equal


Triangle
length
 All angles are of 60ᣞ
 Perimeter = 3S
√3 2
 Area = 𝑆
4

Circle  AB=D=Diameter
 AC=BC=Radius=r
 Circumference = 2𝜋𝑟
 Area =𝜋𝑟 2
117

Cylinder  Volume =𝜋𝑟 2 h


 Surface Area of Cylinder
For hollow cylinder =
2𝜋𝑟ℎ
For solid cylinder =
2𝜋𝑟ℎ+2𝜋𝑟 2

Cube  All sides are of equal


length, a=b=c
 Volume=a3
 Surface Area = 6a2

Rectangul  Volume= abc


ar Solid
 Surface Area=
2ab+2bc+2ac

4
Sphere  Volume= 𝜋𝑟 3
3

 Surface Area= 4𝜋𝑟 2


118

Right  Lateral Surface Area of a


Cone
right cone is L.S.A =𝜋𝑟𝑙,
where 𝑙 is the slant
height of the cone.
 Total Surface Area =
𝜋𝑟𝑙 + 𝜋𝑟 2

Pyramid  Lateral Surface area of a


1
pyramid is = 𝜌𝑙
2

 Total Surface Area=


1
𝜌𝑙+B
2

Square
A Square is a closed figure Quadrilateral having the following Properties:

1. The Diagonals of a square bisect each other and meet at 90°


2. The diagonals of a square bisect its angles.
3. Opposite sides of a square are both Parallel and equal in length.
4. All four angles of a square are equal. (Each is 360°/4 = 90°, so every angle of
a square is a right angle.)
5. All four sides of a square are equal.
6. The diagonals of a square are equal.
119

Let ‘a’ be the length of each side of a square. Then,

• Perimeter = 4a units; Where a is the side of square


• Area = a 2 = (Perimeter /4) 2 sq. units
• Area = ½ (Diagonal) 2 sq. Units
• Side of the square = √ (Area) unit
• Diagonal = √ a unit

Worked Out Examples:


Example1. Find the area and perimeter in square centimeters, of a square
whose side is:
(i) 2.4 dm (ii) 20 mm
Solution : (i) We have,
1 dm = 10 cm
Side of the square = 2.4 dm = (2.4 x 10) cm = 24 cm
Area of the square = (side) 2
= 24 2
= 576 cm 2
Perimeter of the square = 4 x side
= 4 x 24
= 96 cm

(ii) We have,
1 mm = 0.1 cm
Side of the square = 20 mm = (20 x 0.1) cm = 2 cm
Area of the square = (side) 2
=22
120

= 4 cm 2
Perimeter of the square = 4 x side
=4x2
= 8cm

Example2. Find the area of a square park whose perimeter is 320 m.

Solution: Area = (Perimeter/ 4) 2 sq. units


Area = (320/ 4) 2
= (80) 2
∴ Area of a square park = 6400m 2
Example3. The side of a square is 4 cm. Find the area of the triangles formed
by joining all of its diagonals.

Solution: Side = 4 cm
Area of square = (side) 2 = 16 cm 2
Area of triangle formed by joining two of its diagonals
= ¼ of 16 = 4 cm 2

Rectangle

Definition:
It is a 4-sided flat shape with straight sides where all interior angles are right
angles (90°). Also the opposite sides are parallel and of equal length.
Example: A square is a special type of rectangle (with equal sides).
121

Formulas:
The area of a rectangle is given by multiplying the width by the height. As a
formula: Area=w × h
Where, w is the width and h is the height of Rectangle
The perimeter is the total distance around the outside, which can be found by
adding together the length of each side. In the case of a rectangle, opposite
sides are equal in length, so the perimeter is twice its width plus twice its
height. Or as a formula:
Perimeter=2(w+h).
Where: w is the width of the rectangle and h is the height of the rectangle.

Worked Out Examples:

Example1. Find the perimeter and area of the rectangle of length 17 cm and
breadth 13 cm.

Solution:

Given: length = 17 cm, breadth = 13 cm

Perimeter of rectangle = 2 (length + breadth)

= 2 (17 + 13) cm

= 2 × 30 cm

= 60 cm

We know that the area of rectangle = length × breadth

= (17 × 13) cm2


= 221 cm2
122

Example2. Find the breadth of the rectangular plot of land whose area is 660 m2
and whose length is 33 m. Find its perimeter.

Solution:

We know that the breadth of the rectangular plot =Area/length


=660m2/33m

= 20 m

Therefore, the perimeter of the rectangular plot = 2 (length + breadth)

= 2(33 + 20) m

= 2 × 53 m = 106 m

Example3. Find the area of the rectangle if its perimeter is 48 cm and its breadth
is 6 cm.

Solution:

P = 2 (l + b)

Here, P = 48 cm; b = 6 cm

Therefore, 48 = 2 (l + 6)

⇒ 48/2 = l + 6

⇒ 24 = l + 6

⇒ 24 - 6 = l

⇒ 18 = l

Therefore, length = 18 cm

Now, area of rectangle = l × b = 18 × 6 cm2 = 108cm2.


123

Example4. Find the breadth and perimeter of the rectangle if its area is 96 cm2
and the length is 12 cm.

Solution:

Given, A = 96 cm2 and l = 12 cm

A=l×b

Therefore, 96 = 12 × b

⇒ 96/12 = b

⇒ b = 8 cm

Now, P = 2 (l + b)

= 2 (12 + 8)
= 2 × 20

= 40 cm

Parallelogram

A parallelogram is a 4-sided shape formed by two pairs of


Parallel lines. Opposite sides are equal in length and
opposite angles are equal in measure. To find the area of a
parallelogram, multiply the base by the height. The
formula is:
Area of Parallelogram = base× height
Like any polygon, the perimeter is the total distance around the outside, which
can be found by adding together the length of each side. In the case of a
parallelogram, each pair of opposite sides is the same length, so the perimeter is
twice the base plus twice the side length. Or as a formula:

Perimeter = 2(w+h). where, w and h are two adjacent sides of


Parallelogram
124

Proof of the above Area and Perimeter

ABCD is a parallelogram with base (b) and altitude (h).

Area of parallelogram = 2 × Area of ∆ABC

= 2 × 1/2 × base × height sq. units

= 2 × 1/2 × AB × CE sq. units

= b × h sq. units

= base × height sq. units

Perimeter of parallelogram = 2(AB + BC)

= 2 × (Sum of adjacent sides)

Worked Out Examples :

Exercise1. A flooring tile has a shape of a parallelogram whose base is 28cm and
the corresponding height is 20cm .how many such tiles are required to cover a
floor of an area 2800m2

Solution:
Area of each parallelogram tile
= Base of the parallelogram × corresponding height of the parallelogram
= 28 cm × 20 cm= 560 cm2

Area of the floor = 2800 × (100 cm)2 = 2800 × 10000 cm2

Area of the floor = Number of tiles × Area of each parallelogram tile.


125

Number of tiles = Area of the floor/ Area of each parallelogram tile


= 2800 × 10000 cm2 /560 cm2 = 50000 tiles.

Exercise2. The area of the parallelogram ABCD is 54 m2 and its perimeter is 34 m.


What are the dimensions of the parallelogram?

Solution:
Step 1: The area of the parallelogram ABCD = base × height = BC × AE
= 54 m2 [Given, area of ABCD = 54 m2]

Step 2: From the figure above, we can see that the height of the parallelogram is
AE = 6 m

Step 3: The base length of the parallelogram is BC = 54 · AE = 546 = 9 m[Substitute


AE = 6.]

Step 4: The perimeter of the parallelogram = AB + BC + AB + BC [From the figure,


since CD = AB and AD = BC.]

Step 5: [Substitute the value of the perimeter of the parallelogram.] (2 × AB) + (2 ×


BC) = 34

Step 6: [Distributive property.]2 × (AB + BC) = 34

Step 7: [Divide each side by 2.]AB + BC = 17

Step 8: [Substitute base length BC =9 m.]AB + 9 = 17

Step 9: [Subtract 9 from both sides.]AB = 17 - 9 = 8 m

Step 10: The dimensions of the parallelogram are l = 9 m and b = 8 m.


126

Exercise3. Find the perimeter of a parallelogram whose slant height is 24, height
is 22 and breadth is 26, all units are in measured in mm.

Solution: Given that:


a = slant height of the parallelogram
b= breadth of the parallelogram
Perimeter of a parallelogram = 2 × (24+26) = 2 × 50 = 100 mm

Exercise4. Find the perimeter of the parallelogram ABCD.

Solution:
Perimeter of the parallelogram ABCD = AB + BC + CD + AD
AB = DC = 4 cm and AD = BC = 5 cm
Perimeter = 4 + 5 + 4 + 5 = 18 cm

Triangle

Here we will discuss about the area and perimeter of the triangle.

 If a, b, c are the sides of the triangle, then the perimeter of triangle = (a + b


+ c) units.
 Area of the triangle = √(s(s - a) (s - h) (s - c)) .
 The semi-perimeter of the triangle, s = (a + b + c)/2.
 In a triangle if b is the base and h is the height of the triangle then.
 Area of triangle = 1/2 × base × height

Similarly,
127

1/2 × AC × BD 1/2 × BC × AD

f) Base of the triangle = (2 Area)/height.


g) Height of the triangle = (2 Area)/base.
h) Area of right angled triangle(A) = 1/2 × Base× Height
i) If a represents the side of an equilateral triangle, then its area = (a²√3)/4.

WORKED OUT EXAMPLES

Exercise1. Find the area and height of an equilateral triangle of side 12 cm. (√3 =
1.73).

Solution:

√3
Area of the triangle = × a² square units
4
√3
= × 12 × 12
4

= 36√3 cm²

= 36 × 1.732 cm²

= 62.35 cm²
√3
Height of the triangle = a units
2
√3
= × 12 cm
2

= 1.73 × 6 cm
128

= 10.38 cm

Exercise2. Find the area of a right-angled triangle whose hypotenuse is 15 cm and


one of the sides is 12 cm.

Solution:

AB² = AC² - BC²

= 15² - 12²

= 225 - 144

= √81

Therefore, AB = 9
1
Therefore, area of the triangle = × base × height
2

1
= × 12 × 9
2

= 54 cm²

Exercise3. The base and height of the triangle are in the ratio 3 : 2. If the area of
the triangle is 243 cm² find the base and height of the triangle.

Solution:
Let the common ratio be 𝑥

Then height of triangle = 2 𝑥

And the base of triangle = 3 𝑥

Area of triangle = 243 cm²


1 1
Area of triangle = × b × h, 243 = ×3𝑥 ×2𝑥
2 2

⇒ 3 𝑥² = 243
129

243
⇒ 𝑥² =
3

⇒ 𝑥 = √81

⇒ 𝑥 = √(9 × 9)

⇒𝑥=9

Therefore, height of triangle = 2 × 9 = 18 cm

Base of triangle = 3𝑥 = 3 × 9 = 27 cm

Exercise4. Find the area of a triangle whose sides are 41 cm, 28 cm, 15 cm. Also,
find the length of the altitude corresponding to the largest side of the triangle.

Solution:

𝑎 +𝑏 +𝑐
Semi-perimeter of a triangle =
2
41 + 28 +15
=
2
84
=
2

= 42 cm

Therefore, area of a triangle = √(s(s − a) (s − b) (s − c))

= √(42 (42 − 41) (42 − 28) (42 − 15)) cm²

= √(42 × 1 × 27 × 14) cm²

= √(3 × 3 × 3 × 3 × 2 × 2 × 7 × 7) cm²

= 3 × 3 × 2 × 7 cm²

= 126 cm²
130

1
Now, area of a triangle = ×b×h
2

2𝐴
Therefore, h =
𝑏

2 × 126
=
41
252
=
41

= 6.1 cm

Circle

Circumference of circle: The distance around the circular region is called its
circumference. The ratio of circumference of any circle to its diameter is constant.
Circumference/Diameter = Pie(π)

i.e., c/d = π or c = πd

We know that diameter is twice the radius, i.e., d = 2r

C = π × 2r

⇒ C = 2πr
22
Therefore approximate value of π = or 3.14
7

Area of circle: The measure of the region enclosed inside the circle is called its
area.

r
Area

Area of circle = πr²


131

Where, r is the radius of the circle.

Worked Out Examples

Exercise1. Find the circumference and area of circle with radius 7 cm.

Solution:

Circumference of circle = 2πr


22
=2× ×7
7

= 44 cm

Area of circle = πr²


22
= × 7 × 7 cm²
7

= 154 cm²

Exercise2. A race track is in the form of a ring whose inner circumference is 220 m
and outer circumference is 308 m. Find the width of the track.

Solution:
Let r₁ and r₂ be the outer and inner radii of ring.

Then 2πr₁ = 308


22
2× r₁ = 308
7
308 ×7
⇒ r₁ =
2 ×22

⇒ r₁ = 49 m

2πr₂ = 220
22
⇒2× × r₂ = 220
7
132

220 ×7
⇒ r₂ =
2 ×22

⇒ r₂ = 35 m

Therefore, width of the track = (49 - 35) m = 14 m

Exercise3. The area of a circle is 616 cm². Find its circumference.

Solution:

We know that the area of circle = πr²


22
⇒ × r² = 616
7
616 ×7
⇒ r² =
22

⇒ r² = 28 × 7

⇒ r = √(28 × 7) = √196

⇒ r = √(2 × 2 × 7 × 7)

⇒r=2×7

⇒ r = 14 cm

22
Therefore, circumference of circle = 2πr = 2 × × 14 = 88 cm
7

Exercise4. Find the area of the circle if its circumference is 132 cm.

Solution:

We know that the circumference of circle = 2πr

Area of circle = πr²


133

Circumference = 2πr = 132


22
⇒2× × r = 132
7
7 ×132
⇒r=
2 ×22

⇒ r = 21 cm

Therefore, area of circle = πr²


22
= × 21 × 21
7

= 1386 cm²
Trapezium
Recall that a trapezium is a quadrilateral with one pair of parallel sides. The
lengths of the parallel sides are the bases. The perpendicular distance between
the parallel sides is the height, or altitude, of the trapezium.

To find the area of the trapezium, let’s turn it into a parallelogram. To do this,
make a copy of the trapezium and then rotate the copy 180°. Now, this is a
parallelogram with height h and base b1+b2. Let’s find the area of this shape.

A=h × (b1+b2)
Because the area of this parallelogram is made up of two congruent trapeziums,
the area of one trapezium would be :
134

𝟏
A= × h × (b1+b2)
𝟐

The formula for the area of a trapezium could also be written as the average of
the bases times the height.
Like any polygon, the perimeter is the total distance around the outside, which
can be found by adding together the length of each side. Or as a formula:

Perimeter = a+ b+ c+ d, where: a, b, c, d are the lengths of each side.

Worked Out Examples

Exercise1. Find the area of the trapezium below.

1
Solution: A= × (11) × (14+8)
2
1
A= × (11) × (22)
2
A=121 units2

Exercise 2. Find the area of the trapezium below.

1
A= x (9) × (15+23)
2
1
A= x (9) × (38)
2
A=171 units2
135

Exercise3. Find the perimeter of the trapezium.

Solution: Even though we are not told the length of the second base, we can find
it using special right-angled triangles. Both triangles at the ends of this trapezium
are isosceles right-angled triangles, so the hypotenuses are 4√2(By applying sine
formula in the smaller traingle) and the other legs are of length 4.
P= 8+ 4√2 + 16 + 4√2

P= 24 + 8√2 =35.3 units

Exercise4: The area of a trapezium shaped field is 480 m2 , the distance between
two parallel sides is 15 m and one of the parallel side is 20 m. Find the other
parallel side.

Solution: One of the parallel sides of the trapezium is a = 20 m, let another


parallel side be b, height h = 15 m. The given area of trapezium = 480 m2.
1
Area of a trapezium = h (a + b)
2

1
So, 480 = × 15 × (20 + b)
2
480
= (20 + b)
7.5

64 =20 + b

64 – 20 = b

44 =b

So, b= 44m

Hence the other parallel side of the trapezium is 44 m.


136

Surface Area of a Cube

A cube is a three-dimensional figure with six equal square faces.


The surface area of a cube is the sum of the area of the six squares that cover it.

The figure below shows a cube. The dotted lines indicate edges hidden from your
view.

If s is the length of one of its sides, then the area of one face of the cube is s2

Since a cube has six faces, the surface area of a cube is six times the area of one
face.

Surface area of a cube = 6 x s2

Exercise:

Example1: Find the surface area of a cube with a side of length 3 cm

Solution: Given that s = 3

Surface area of a cube = 6s2 = 6 x (3)2 = 54 cm2

Exercise2: If each edge of a cube is increased by 50% then what is the percentage
increase in the surface area of the cube?

Solution:
Initial Surface Area of Cube = 6a2
137

50 3𝑎
Final Edge of Cube = a + ×a=
100 2
3𝑎 3𝑎 54𝑎² 27𝑎²
Final Surface Area = 6 x × = =
2 2 4 2
27𝑎² 2 15𝑎²
Increase in Surface Area = - 6a =
2 2
Therefore, Percentage = Increase/Initial × 100
15𝑎² ÷2 15𝑎² 1 15
× 100 = × × 100 = × 100 =125%
6𝑎² 2 6𝑎² 12

Volume of Cube:
In Geometry, a cube is a three-dimensional solid object bounded by
six Square faces, Facets or sides, with three meeting at each vertex.
The cube is the only regular Hexahedron and is one of the five Platonic Solids. It
has 6 faces, 12 edges, and 8 vertices.

Given the length of one side, call it a, the volume of a cube can be found by using
the following formula:
Vcube = a3 = a × a × a

a
a

Example: Find the volume of a cube if the length of one side is 2 cm

Solution: Vcube = 23

Vcube = 2 × 2 × 2

Vcube = 8 cm3
138

Cuboid

A cuboid is a solid bounded by six faces that are rectangular in shape. It has six
flat faces and all angles are right angle.

height (h)

Windth (w)
length (l)

Volume = Length (l) × Width (w) × Height (h)


The total surface area (TSA) of a cuboid is the sum of the areas of its 6 faces,
which is given by:

TSA = 2 (lw + wh + hl)


Example: The length, breadth and height of a cuboid are 16cm, 14cm and 10cm
respectively. Find the total surface area of the cuboid?
Solution: The total surface area of a cuboid is given by:
TSA = 2 (l× b + b× h + h× l)
Given that:
l = 16cm
b = 14cm
h = 10cm

Putting the values in the equation we will get:


TSA = 2 (16× 14 + 14× 10 + 10× 16)
TSA = 2(224 + 140 + 160)
TSA = 2 × 524
TSA = 1048 cm2

Example: Find the volume of a cuboid of dimensions 14 cm × 12 cm × 10 cm.

Solution: Volume of cuboid = length × breadth × height.

Here, length = 14 cm, breadth = 12 cm and height = 10 cm.


139

Volume of cuboid = 14 × 12 × 10

= 1680 cm³
Therefore, volume of cuboid = 1680 cm³

Summary on Surface Area of Cuboid:

To calculate the surface area of the cuboid we need to first calculate the area of
each face and then add up all the areas to get the total surface area.

Total area of top and bottom surfaces is lw + lw = 2lw


Total area of front and back surfaces is lh + lh = 2lh
Total area of the two side surfaces is wh + wh = 2wh

Surface area of cuboid = 2lw + 2lh + 2wh = 2(lw + lh + wh)

Example: Find the surface area of the following cuboid.

3cm 6cm

5cm
140

Solution:

Total area of top and bottom surfaces is 2 × 5 × 6 = 60 cm2


Total area of front and back surfaces is 2 × 5 × 3 = 30 cm2
Total area of the two side surfaces is 2 × 6 × 3 = 36 cm2

Surface area of cuboid = 60 + 30 + 36 = 126 cm2

Or l = 6 cm, w = 5 cm and h = 3 cm
Surface area of cuboid = 2(lw + lh + wh) = 2 (6 × 5 + 6 × 3 + 5 × 3) = 126 cm

Cylinder
A Cylinder is a solid geometrical figure with two straight parallel sides and a
circular or oval cross section. It has a flat base and a flat top. The base is the same
as the top, and also in-between. It has one curved side.

Height (h)

Cylinder

Surface Area of a Cylinder: Surface Area = 2 × π × r × (r+h) or 2πrh + 2πr²


It came from:
1. Surface Area of One End = π × r2

2. Surface Area of Side = 2 × π × r × h

Volume = π × r2 × h

It came from:

1. Area of the base: π × r2


141

2. Height: h
3. Volume = Area × Height = π × r2 × h

Example: The diameter of the base of a cylinder is 12 cm and the height is 8 cm.
Find the surface area of the solid cylinder. Use π=3.14

Solution: Radius = 6 cm
Surface area = 2πr (r + h)
= 2 × π × 6 × (6+8)
= 527.52 cm2

Example: Calculate the Volume of a cylinder if r = 2 cm and h = 5 cm?


Solution: Volume = π × r2× h
= 3.14 × 22× 5
= 3.14 × 4 × 5 = 3.14 × 20
= 62.8 cm3
Example: Find the height of a cuboid whose volume is 300 cm³ and the base area
is 30cm² ?
Solution:
V=l×w×h
Since the base area is defined as: l × w, the height is therefore:
v
h=
𝑏𝑎𝑠𝑒 𝑎𝑟𝑒𝑎

300
h=
30
h = 10 cm

Area of a Sector

We can work out the Area of a Sector by comparing its angle to the angle of a full
circle.

Note: we are using radian for the angles.


142

This is the reasoning:


A circle has an angle of 2π and an Area of: πr2
θ
A Sector with an angle of θ (instead of 2π) has an Area of: × πr2

θ
This can be simplified as: × r²
2

θ
Area of Sector = × r² (when θ is in radians)
2
π
Area of Sector = θ × × r2 (when θ is in degrees)
360

Example:

Given that the radius of the circle is 5 cm, calculate the area of the shaded sector.
(Take π = 3.142).

60°
Ɵ

π
Solution: Area of Sector = θ × × r2 (when θ is in degrees)
360
143

π
Area of Sector = 60 × × 52 (when θ is in degrees)=13.09 cm2
360

Example:

The area of a sector with a radius of 6 cm is 35.4 cm2. Calculate the angle of the
sector. (Take π = 3.142).
π
Solution: Area of Sector = θ × × r2 (when θ is in degrees)
360
π
We have to find the angle by applying the following formula, A=θ× × r2
360
𝟑𝟔𝟎 × 𝐀
We get, Central Angle = 112.66° θ= = 112.66°
𝛑 𝐫²
Example: A pizza has a diameter of 14cm.

a). Calculate the area of a slice of pizza when the chef made all the slices with an
angle of 45°.

b). How many slices do you think this pizza has?

Solution:

a.
π
Area of Sector = θ × × r2 (when θ is in degrees)
360
π
A = 45° × × 142
360
π
A = 45° × × 196
360

A = 76.97 cm2

b. If all slices were cut with the same angle, then all slices must have the same
size.

So, "How many 45° will give 360°?"


144

45° + 45° + 45° + 45° + 45° + 45° + 45° + 45° = 360°

Or 8 × 45° = 360°

The pizza has 8 slices

Volume of Sphere
A sphere is a perfectly round geometrical object in three-dimensional space that
is the surface of a completely round ball.
Given the radius, the volume of a sphere can be found by using the following
formula:

𝟒
Volume of Sphere = × 𝝅 × r3
𝟑
Surface Area of Sphere = 4 × 𝝅 × r2

Use, 𝜋 = 3.142

Example1: Find Vsphere if r = 3 cm


Solution:
4
Vsphere = × 𝜋 × r3
3

4 4
= × 3.142 × 33 = × 3.142 × 27
3 3

4
= × 84.83
3

= 113.11 cm3
145

Example2: The diameter of the sphere is 11.9 cm, Find the Surface area of
Sphere.
Solution:
Radius of Sphere= 11.9÷2 = 5.95cm
Surface Area = 4 ×3.142× 5.952 =444.93 cm2

Example3. A spherical ball has a surface area of 2464 cm2. Find the radius of the
ball, correct to 2 decimal places, using π = 3.142

Solution:
SA = 4 × π × r2
In order to find r, we need to isolate it from the equation above:
𝑆𝐴
r2 =

2464
r2 =
4×π
2
r =196.05
r = √196.05
r = 14.00 cm²

Example4. Find the surface area of the sphere whose radius is 18 cm. [π = 3.14]
Solution:
r = 18 cm
The surface area of a sphere is given by:
SA = 4 × π × r2
SA = 4 × 3.14 × 182
SA = 4 × 3.14 × 342
SA = 4069.44 cm2

The surface area of the sphere is 4069.44 cm2.


146

Cone
A cone is a three-dimensional geometric shape that tapers smoothly from a flat
base (frequently, though not necessarily, circular) to a point called the apex or
vertex.
1
Curved Surface Area of a Cone = × l × 2πr = πrl
2
Total Surface Area of a Cone = πrl + πr²
𝟏
Volume of a Cone = πr2h
𝟑

Worked Out Example:


Example1: Find the curved surface area of a right circular cone whose slant height
is 10 cm and base radius is 7 cm.
22
Solution : Curved surface area = πrl = × 7 × 10 cm2 = 220 cm2
7
Example2: The height of a cone is 16 cm and its base radius is 12 cm. Find the
curved surface area and the total surface area of the cone (Use π = 3.14).
Solution: Here, h = 16 cm and r = 12 cm. So, from l2 = h2 + r2 , we have l = 20 cm
So, curved surface area = πrl= 3.14 × 12 × 20 cm² = 753.6 cm2
Further, total surface area = πrl + πr2 = 753.6 + (3.14 × 12 × 12) cm2
= (753.6 + 452.16) cm2
= 1205.76 cm2
147

Example3: The height and the slant height of a cone are 21 cm and 28 cm
22
respectively. Find the volume of the cone. Use π =
7

Solution : From l2= r2+ h2 , we have r =7√7 cm for l=28cm and h= 21 cm


1
So, volume of the cone = πr2 h = 7546 cm3
3

Prism
A Prism is a solid geometric figure whose two ends are similar, equal, and parallel
rectilinear figures, and whose sides are parallelograms.

A prism is a polyhedron with two parallel, congruent faces called bases that are
polygons.

Triangular Prism
𝟏
V= x B x h
𝟐 Rectangular Prism
or
𝟏
V= x l x h x b V =B × h or V = l x h x b
𝟐

The volume V of a prism is the area of the base B times the height h.
148
Worked example
Exercise: Find the volume of the prism shown.

Solution: The formula for the volume of a prism is V=B×h, where B is the base
area and h is the height.
The base of the prism is a rectangle. The length of the rectangle is 9 cm and the
width is 7 cm.
The area A of a rectangle with length l and width w is A=l × w.
So, the base area is 9×7 or 63 cm2.
The height of the prism is 13 cm.
Substitute 63 for B and 13 for h in V=B × h.
V=(63) × (13)
Multiply.
V=819 cm³
Therefore, the volume of the prism is 819cm³.
149

Exercise: Find the volume of the prism shown.

Solution: With a triangular prism, the bases are the parallel sides (where the
triangles are).
𝟏 𝟏
B = × b × h = x (9.0 × 4.0 cm)
𝟐 𝟐

B = 18 cm2
This prism has a height of 8.1 cm.
Volume = B × h = 18 cm2 × 8.1 cm
Volume = 145.8 cm3
Exercise: Find the volume of the trapezoidal prism shown below.

Solution: The base of this prism is a trapezium.


𝟏 𝟏
B = × (b1 + b2) × h = × (4+10) ×2 cm
𝟐 𝟐

B = 14 cm2
150

This prism has a height of 7.0 cm.


Volume = B × h = 14 cm2 × 7.0cm
Volume = 98 cm3

3.1.2 Problem Set 3.1


1. A rectangle is 16 cm long and 10 cm wide. If the length is reduced by k cm
and its width is increased also by k cm so as to make it a square then find
out the change in area.
2. What happens to the area of a circle if radius is increased by 20%?
3. A rectangular lot 50 feet by 100 feet is surrounded on all sides by a
concrete walk 5 feet wide. Find the number of square feet in the surface of
the walk.
4. Find the lateral surface area of a right cone if the radius is 4 cm and the
slant height is 5 cm.

5. A conical tent has a radius of 3m and a perpendicular height of 4m. Find:


a) The curved surface area of the cone using 𝜋 =3.142
b) The slant height of the tent

6. In a circle whose radius is 4 cm, find the arc length intercepted by each of
these angles. Take 𝜋 ≈ 3
𝜋
a)
4
𝜋
b)
6
c) 2𝜋
7. Find:
a) Circumference of a circle with radius 6 centimeters
b) Radius of a circle with circumference 31 meters
151

Chapter 4

4.1 Geometrical Terms and Relationships


General Objectives: know and use geometrical terms and the vocabulary of
simple plane figures and simple solids.
Specific Objectives:
1. Use and interpret the geometrical terms: point, line, parallel, intersecting,
bearing, right angle, acute, obtuse and reflex angles, perpendicular,
similarity, congruence
2. Use and interpret vocabulary of triangles, quadrilaterals, circles, polygons
and simple solid figures, including nets.
3. Use the relationships between areas of similar triangles with corresponding
results for similar figures and extension to volumes and surface areas of
similar solids.
Angles
When two lines meet at a point, they form an angle. The size of the angle
depends only upon the amount of opening between the lines. Angle B is larger
than angle A.

4.1.1 Types of angles


a) Acute angle : An acute angle is less than 90ᣞ
152

b) Right angle : A right angle is equal to 90ᣞ

c) Obtuse angle: An obtuse angle lies between 90ᣞ to 180ᣞ

d) Reflex angle: A reflex angle is greater than 180ᣞ

Some of the important terms are:


Congruent Angles Angles that have the same measure
are called congruent angles
Angle bisector Ray that divides an angle into two
congruent angles.
Adjacent Angles Two angles that lie in the same
plane and share a common side and
a common vertex.

Vertical Angles Two non-adjacent angles formed by


intersecting lines.
153

Linear Pair Pair of adjacent angles whose non-


common sides are opposite rays.

Perpendicular Lines Lines that intersect to form four


right angles.

COMPLEMENTS AND SUPPLEMENTS


Two angles whose sum is 90° are complementary. For example, a 60° angle is the
complement of a 30° angle. Angles having a sum equal to 180° are
supplementary. For example, a 100° angle is the supplement of an 80° angle.
Below are the figures showing complementary and supplementary angles.

(a) Segment: A part of line with two end points is called a line-segment.

A line segment is denoted by AB and its length is denoted by AB.

(b) Ray: A part of a line with one end-point is called a ray.

We can denote a line-segment AB, a ray AB and length AB and line AB by the
same symbol AB.

(c) Collinear points: If three or more points lie on the same line, then they are
called collinear points, otherwise they are called non-collinear points.
154

(d) Angle: An angle is formed by two rays originating from the same end point.

The rays making an angle are called the arms of the angle and the end-points are
called the vertex of the angle.

(e) Straight angle: The measure of a straight angle is 180°.

Example: x and y are complementary angles. Given x = 35˚, find the value of y.
Solution:
x + y = 90˚
35˚ + y = 90˚
y = 90˚ – 35˚ = 55˚
Note: The two angles do not need to be together or adjacent. They just need to
add up to 90 degrees. If the two complementary angles are adjacent then they
will form a right angle.
(f) Linear pair of angles: If the sum of two adjacent angles is 180°, then their non-
common lines are in the same straight line and two adjacent angles form a linear
pair of angles.
It is a pair of Adjacent angles formed by intersecting lines. Angles 1 and 2 below
are a linear pair. So are angles 2 and 4, angles 3 and 4, and angles 1 and 3. Linear
pairs of angle are Supplementary

Linear pair of
angles
155

(g) Vertically opposite angles: When two lines AB and CD intersect at a point O,
the vertically opposite angles are formed.

Here are two pairs of vertically opposite angles. One pair is ∠AOD and ∠BOC , the second
pair is ∠AOC and ∠BOD. The vertically opposite angles are always equal.
So, ∠AOD = ∠BOC and ∠AOC = ∠ BOD

Intersecting lines and non-intersecting lines: Two lines are intersecting if they
have one point in common. We have observed in the above figure that lines AB
and CD are intersecting lines, intersecting at O, their point of intersection.

Parallel lines: If two lines do not meet at a point if extended to both directions,
such lines are called parallel lines. Lines PQ and RS are parallel lines.

Note: The length of the common perpendiculars at different points on these


parallel lines is the same. This equal length is called the distance between two
parallel lines.

Example : In the figure below, lines PQ and RS intersect each other at point O. If
∠POR : ∠ ROQ = 5 : 7, find all the angles
156

Solution : ∠ POR +∠ ROQ = 180° (Linear pair of angles)


But ∠POR : ∠ ROQ = 5 : 7 (Given)
Therefore, ∠ POR = (5/12) × 180° = 75°
Similarly, ∠ ROQ = (7/12) × 180° = 105°
Now, ∠ POS = ∠ROQ = 105° (Vertically opposite angles) and
∠ SOQ = ∠POR = 75° (Vertically opposite angles)
Example : In the figure below, ray OS stands on a line POQ. Ray OR and ray OT are
angle bisectors of ∠ POS and ∠ SOQ, respectively. If ∠ POS = x, find ∠ ROT.

Solution : Ray OS stands on the line POQ.


Therefore, ∠ POS + ∠ SOQ = 180°
But, ∠ POS = x
Therefore, x + ∠ SOQ = 180°
So, ∠ SOQ = 180°– x
Now, ray OR bisects ∠ POS, therefore, ∠ ROS = 1/2 × ∠ POS = 1/2 × x = x/2
Similarly, ∠ SOT = 1/2 × ∠ SOQ = 1/2 × (180° – x) = 90-x/2
∠ ROT = ∠ ROS + ∠ SOT = x/2+90°-x/2 = 90°
157

Example : In the figure below, OP, OQ, OR and OS are four rays. Prove that ∠ POQ
+ ∠ QOR + ∠ SOR + ∠ POS = 360°

Solution: In the first figure, you need to produce any of the rays OP, OQ, OR or OS
backwards to a point. Let us produce ray OQ backwards to a point T so that TOQ is
a line (see second figure.). Now, ray OP stands on line TOQ
Therefore, ∠ TOP + ∠ POQ = 180° (1) (Linear pair axiom)
Similarly, ray OS stands on line TOQ.
Therefore, ∠ TOS + ∠ SOQ = 180° (2)
But ∠ SOQ = ∠ SOR + ∠ QOR
So, (2) becomes ∠ TOS + ∠ SOR + ∠ QO R = 180° (3)
Now, adding (1) and (3), you get
∠ TOP + ∠ POQ + ∠ TOS + ∠ SOR + ∠ QOR = 360° (4)
But ∠ TOP + ∠ TOS = ∠ POS,
Therefore, (4) becomes ∠ POQ + ∠ QOR + ∠ SOR + ∠ POS = 360°

Parallel Lines and a Transversal

As we know that a line which intersects two or more lines at distinct points is
called a transversal. Line l intersects lines m and n at points P and Q respectively.
Therefore, line l is a transversal for lines m and n. Observe that four angles are
formed at each of the points P and Q. Let us name these angles as ∠ 1, ∠ 2, . . .,∠8
as shown in below Fig.. ∠ 1, ∠ 2, ∠ 7 and ∠ 8 are called exterior angles, while ∠ 3,
∠ 4, ∠ 5 and ∠ 6 are called interior angles.
158

(a) Corresponding angles : (i) ∠ 1 and ∠ 5 (ii) ∠ 2 and ∠ 6


(iii) ∠ 4 and ∠ 8 (iv) ∠ 3 and ∠ 7
(b) Alternate interior angles : (i) ∠ 4 and ∠ 6 (ii) ∠ 3 and ∠ 5
(c) Alternate exterior angles: (i) ∠ 1 and ∠ 7 (ii) ∠ 2 and ∠ 8
(d) Interior angles on the same side of the transversal:
(i) ∠ 4 and ∠ 5 (ii) ∠ 3 and ∠ 6
Note: Interior angles on the same side of the transversal are also referred to as
consecutive interior angles or allied angles or co-interior angles.
Axiom 1 : If a transversal intersects two parallel lines, then each pair of
corresponding angles is equal. Axiom 1 is also referred to as the corresponding
angles axiom. Now, let us discuss the converse of this axiom which is as follows:
If a transversal intersects two lines such that a pair of corresponding angles is
equal, then the two lines are parallel.
Does this statement hold true? It can be verified as follows: Draw a line AD and
mark points B and C on it. At B and C, construct ∠ ABQ and ∠ BCS equal to each
other as shown in Fig.(i)
159

Produce QB and SC on the other side of AD to form two lines PQ and RS [see
Fig(ii)]. You may observe that the two lines do not intersect each other. You may
also draw common perpendiculars to the two lines PQ and RS at different points
and measure their lengths. You will find it the same everywhere. So, you may
conclude that the lines are parallel. Therefore, the converse of corresponding
angles axiom is also true. So, we have the following axiom
Axiom 2: If a transversal intersects two lines such that a pair of corresponding
angles is equal, then the two lines are parallel to each other.

Can we use corresponding angles axiom to find out the relation between the
alternate interior angles when a transversal intersects two parallel lines?
In the above figure, transversal PS intersects parallel lines AB and CD at points Q
and R respectively.
Is ∠ BQR = ∠ QRC and ∠ AQR = ∠ QRD? You know that ∠ PQA = ∠ QRC (1)
(Corresponding angles axiom)
Is ∠ PQA = ∠BQR ? (Vertically opposite angle) (2)
So, from (1) and (2), you may conclude that ∠ BQR = ∠ QRC.
Similarly, ∠ AQR = ∠ QRD.
Theorem: If a transversal intersects two parallel lines, then each pair of alternate
interior angles is equal.
160

Now, using the converse of the corresponding angles axiom, can we show the two
lines parallel if a pair of alternate interior angles is equal? In the figure above, the
transversal PS intersects lines AB and CD at points Q and R respectively such that
∠ BQR = ∠ QRC.
Is AB || CD? ∠ BQR = ∠ PQA (Vertically Opposite angle) (1)
But, ∠ BQR = ∠ QRC (Given) (2)
So, from (1) and (2), you may conclude that ∠ PQA = ∠ QRC
But they are corresponding angles. So, AB || CD (Converse of corresponding
angles axiom)
Theorem: Lines Parallel to the same line are Parallel to each other.
If two lines are parallel to the same line, will they be parallel to each other? Let us
check it.

See the figure below, in which line m || line l and line n || line l.

Note: This sign II means parallel

Let us draw a line t transversal for the lines, l, m and n.


It is given that line m || line l and line n || line l.
161

So, ∠ 2 = ∠3 ; But ∠ 2 and ∠ 3 are corresponding angles and they are equal.
Therefore, you can say that Line m || Line n (Converse of corresponding angles
axiom)
Therefore, ∠ 1 = ∠ 2 and ∠ 1 = ∠ 3 (Corresponding angles axiom)
Example 5: In a Fig. below, if PQ || RS, ∠ MXQ = 135° and ∠ MYR = 40°, find
∠ XMY.

Solution: Here, we need to draw a line AB parallel to line PQ, through point M as
shown in Second Fig. Now, AB || PQ and PQ || RS.
Therefore, AB || RS
Now, ∠ QXM + ∠ XMB = 180° (AB || PQ, Interior angles on the same side of the
transversal XM)
But ∠ QXM = 135° So, 135° + ∠ XMB = 180°
Therefore, ∠ XMB = 45° …………………………..(1)
Now, ∠ BMY = ∠ MYR (AB || RS, Alternate angles) Therefore, ∠ BMY = 40° (2)
Adding (1) and (2), you get ∠ XMB + ∠ BMY = 45° + 40°
That is, ∠ XMY = 85°
Example 6: If a transversal intersects two lines such that the bisectors of a pair of
corresponding angles are parallel, then prove that the two lines are parallel.
162

Solution: In the figure above, a transversal AD intersects two lines PQ and RS at


points B and C respectively. Ray BE is the bisector of ∠ ABQ and ray CG is the
bisector of ∠ BCS; and BE || CG.
We are to prove that PQ || RS. It is given that ray BE is the bisector of ∠ ABQ.
Therefore, ∠ ABE = (½) ∠ ABQ (1)
Similarly, ray CG is the bisector of ∠ BCS.
Therefore, ∠ BCG = (1/2)∠ BCS (2)
But BE || CG and AD is the transversal.
Therefore, ∠ ABE = ∠ BCG (Corresponding angles axiom) (3)
Substituting (1) and (2) in (3), you get
(1/2)∠ ABQ = (1/2)∠ BCS That is, ∠ ABQ = ∠ BCS
But, they are the corresponding angles formed by transversal AD with PQ and RS;
and are equal. Therefore, PQ || RS (Converse of corresponding angles axiom)
Example 7: In the figure below, AB || CD and CD || EF. Also EA ⊥ AB. If ∠ BEF =
55°, find the values of x, y and z.
163

Solution: y + 55° = 180° (Interior angles on the same side of the of the transversal
ED)
Therefore, y = 180°– 55° = 125°
Again x = y (AB || CD, Corresponding angles axiom)
Therefore x = 125°
Now, since AB || CD and CD || EF, therefore, AB || EF.
So, ∠ EAB + ∠ FEA = 180° (Interior angles on the same side of the transversal EA)
Therefore, 90° + z + 55° = 180° which gives z = 35°

Angle Sum Property of a Triangle


We have studied that the sum of all the angles of a triangle is 180°. We can prove
this statement using the axioms and theorems related to parallel lines
Theorem : The sum of the angles of a triangle is 180°

Proof: Let us see what is given in the statement above, that is, the hypothesis and
what we need to prove. We are given a triangle PQR and ∠ 1, ∠ 2 and ∠ 3 are the
angles of ∆ PQR (see above Fig. ).
We need to prove that ∠ 1 + ∠ 2 + ∠ 3 = 180°.
Let us draw a line XPY parallel to QR through the opposite vertex P, so that we can
use the properties related to parallel lines. Now, XPY is a line.
164

Therefore, ∠ 4 + ∠ 1 + ∠ 5 = 180° (1)


But XPY || QR while PQ, and PR are transversals. So, ∠ 4 = ∠ 2 and ∠ 5 = ∠ 3
(Pairs of alternate angles) Substituting ∠ 4 and ∠ 5 in (1),
We get ∠ 2 + ∠ 1 + ∠ 3 = 180° That is, ∠ 1 + ∠ 2 + ∠ 3 = 180°

Side QR is produced to point S, ∠ PRS is called an exterior angle of ∆PQR.


∠ 3 + ∠ 4 = 180°(Straight angle)(1)
Also, see that ∠ 1 + ∠ 2 + ∠ 3 = 180° (2)
From (1) and (2), you can see that ∠ 4 = ∠ 1 + ∠ 2.
Theorem: If a side of a triangle is produced, then the exterior angle formed is
equal to the sum of the two interior opposite angles.
Proof: It is obvious from the above theorem that an exterior angle of a triangle is
greater than either of its interior apposite angles and is equal to the sum of two
interior opposite angles.
Solved Examples:
Example 1: In the figure below, if QT ⊥ PR, ∠ TQR = 40° and ∠ SPR = 30°, find
𝑥 and y.
165

Solution: In ∆ TQR, 90° + 40° + 𝑥 = 180° (Angle sum property of a triangle)


Therefore, 𝑥 = 50° Now, y = ∠ SPR + 𝑥 (Theorem 2)
Therefore, y = 30° + 50° = 80°
Example 2 : In the figure on the next page the sides AB and AC of ∆ABC are
produced to points E and D respectively. If bisectors BO and CO of ∠ CBE and ∠
BCD respectively meet at point O, then prove that
∠ BOC = 90°– 1/2 ∠BAC.

Solution: Ray BO is the bisector of ∠ CBE.


Therefore, ∠ CBO = 1/2 ∠ CBE
= 1/2 (180° – y)
= 90° – y/2 (1)
Similarly, ray CO is the bisector of ∠ BCD.
166

Therefore, ∠ BCO = 1/2 ∠ BCD


= 1/2 (180° – z)
= 90° – z/2 (2)
In ∆ BOC, ∠ BOC + ∠ BCO + ∠ CBO = 180° (3)
Substituting (1) and (2) in (3), you get ∠ BOC + 90°–z/2 + 90° – y/2 = 180°
So, ∠ BOC = z/2 + y/2
or, ∠ BOC = 1/2 (y + z) (4)
But, x + y + z = 180° (Angle sum property of a triangle)
Therefore, y + z = 180° – x
Therefore, (4) becomes ∠ BOC = 1/2 (180° – x)
= 90° – x/2
= 90° – 1/2 ∠ BAC

4.1.2 Problem Set 4.1


1. Angle BAC is the right angle. Can you find the measure of angle ABC?

2. Select the statement that best describes the image below.

3. Which statement is not true?


a) Parallel lines are the same length apart and never touch.
b) Line segments have two endpoints.
c) Perpendicular lines form 45° angles.
167

d) Intersecting lines have one common point


4. In the image below:

a) Name the two lines


b) Tell whether these two lines are parallel, intersecting, or
perpendicular.

4.2 Geometrical Constructions


General Objectives: Measure lines and angles and construct simple geometrical
figures using straight edges, compasses, protractors and set squares.
Specific Objectives:
1. Measure lines and angles
2. Construct a triangle given the three sides using a straight edge and compass
only.
3. Construct other simple geometrical figures from given data using
protractors and set squares as necessary.
4. Construct angle bisectors and perpendicular bisectors using straight edges
and compasses only.
5. Read and make scale drawings.
168

Measure Lines and Angles using a compass and protractor

1) Draw 70° 2) Draw 140°

Angle : 70° Angle: 140°

Measure the angles below using a protractor:


1) 2)

Angle: 30°
Angle : 45°
169

3) 4)

55° 115°

Set Square
There are two types of set squares and they are named according to the angles
present on each.

Set squares are useful for drawing parallel lines and perpendicular lines.

Parallel Lines

Lines that lie in the same plane and do not meet one another are said to be
parallel lines.

In the accompanying diagram, the AB is parallel to the CD. This is indicated by


170

the similar arrows.

Drawing Parallel Lines

A ruler and set square can be used to draw parallel lines as described below.

Step 1: Position an edge of the set square against a ruler and draw a line along
one of the other edges.
Step 2: Slide the set square into a new position while keeping the ruler fixed
exactly at the same position.
Step 3: Draw a line along the same edge that was used in Step 1.
171

AB is parallel to CD and PQ is Parallel to RS

Example: Use a ruler and set square to draw a line that is parallel to a given line,
AB, and passes through a given point, P.

Solution:

Step 1: Position an edge of the set square along the given line, AB.
Step 2: Place a ruler against one of the other edges.
Step 3: Slide the set square along the ruler until the edge used in Step 1 passes
through the given point P.
Step 4: Draw the line CD through P.

The line CD passes through the given point, P, and is parallel to the given line AB.

Perpendicular Lines

Lines that are at right angles to each other are said to be perpendicular lines.

Note that a vertical line is perpendicular to the horizontal line, whereas


perpendicular lines can be drawn in any position. Bricklayers use a plumb-bob to
set out vertical lines and a spirit level to set out horizontal lines.
172

In the accompanying diagram, the line PQ is at right angles to the line AB. The
right angle is indicated by a square. We say that PQ is perpendicular to AB.

Drawing Perpendicular Lines

A set square can be used to draw a perpendicular at a point on a given line as


described below.

Step 1: Set an edge of the set square on the given line so that the other edge is
just in contact with the point.
Step 2: Draw a line that passes through the given point with the help of the set
square.

Example

Use a set square to draw a perpendicular to a given line, AB, through a point, P,
not on the line.
173

Solution:

Step 1: Set an edge of the set square on the given line so that the other edge is
just in contact with the point.
Step 2: Draw a line that passes through the given point with the help of the set
square.

4.2.1 Problem Set 4.2


1. Construct a triangle with three sides of BC=5cm, AC=7 cm and AB= 8 cm.
2. Draw the angle bisector of the angle given in Figure

3. Draw the perpendicular bisector of a straight line

4. Draw a square using only a ruler and a compass.


5. Sketch figure using the steps give below:
a) Draw a line AB
b) Draw a ray CD
c) Draw a ray EF so that it is perpendicular to AB.
174

d) Draw a line segment GH so that it is parallel to CD.

4.3 Symmetry
General Objectives: Recognize properties of simple plane figures directly related
to their symmetries.
Specific Objectives:
1. Recognize line and rotational symmetry (including order of rotational
symmetry) in two dimensions.
2. Recognize properties of triangles, quadrilaterals and circles directly related
to their symmetries.
3. Use the following symmetry properties of circles:
a) Equal chords are equidistant from the center.
b) The perpendicular bisector of a chord passes through the center of a
circle.
c) Tangents from an external point are equal in length.
Reflective Symmetry
An object has a reflective symmetry if it can be reflected in a particular line and
looks the same as the original. The line the object is reflected across is called a
line of symmetry or a mirror line.
Rotational Symmetry
An object has a rotational symmetry if it looks the same after some rotation by a
partial turn.

Line of symmetry in Polygons and other Figures


Basically, a line of symmetry is a line that divides a figure into two mirror images.
The figure is mapped onto itself by a reflection in that line.
175

A set of points has a line of symmetry if and only if there is a line, l, such that the
reflection through l of each point in the set is also a point in the set.

Certain figures can be mapped onto themselves by a reflection in their lines of


symmetry. Some figures have one or more lines of symmetry, while other figures
have no lines of symmetry.
(Remember, if you fold the figure on a line of symmetry, the folded sides
coincide.)
Lines of Symmetry: One line of symmetry Two lines of symmetry Four lines of symmetry

No line of
symmetry

Beware, not all lines that divide a figure into two congruent halves are lines of
symmetry.
The diagonal of a rectangle divides the rectangle into two triangles that are
congruent (same size and shape). But the diagonal line is NOT a line of symmetry.
Folding along the diagonal does not coincide with the other side.

While the diagonal of a rectangle is not a line of symmetry, the rectangle does
have a vertical and a horizontal line of symmetry, as seen above.
176

The diagonal of a parallelogram also divides the figure into two triangles that are
congruent. But the diagonal line is NOT a line of symmetry. When folding along
the diagonal, the two halves (triangles) do not coincide.

No line of symmetry

The parallelogram, however, has NO lines of symmetry. Even if we try to be clever


and draw the line parallel to a set of sides, the folding does not coincide with the
other side.
Some special circumstances:
In regular polygons (where all sides are congruent and all angles are congruent),
the number of lines of symmetry equals the number of sides.(Start by drawing the
lines through the vertices).

Regular Pentagon
177

When working with a circle, any line through the center of the circle is a line of
symmetry. There are an infinite number of lines of symmetry for a circle.

Circle

There are various Symmetry Properties of Circles. Some of them are mentioned
below.
Equal cords are equidistant from centre

Perpendicular bisector of a chord passes through the Centre


178

Tangent from External Point

90°

90°

For a circle, from an external point P, two tangents PX and PY can be drawn.
These tangents make an angle of 90° with the radius of a circle.
PX = PY
< XOP = < YOP
And < XPO = < YPO

4.3.1 Problem Set 4.3


1. ⧍ABC is an isosceles triangle with AC=BC. What is the equa on of the line
of symmetry of this triangle?

2. Look for lines of symmetry in the figures given below:


179

a)

b)

3. Find the order of rotational symmetry for the figures below:


a)

b)
180

c)

4. Fill out the table below:


Shape Order of Number of lines
rota onal of symmetry
Symmetry
Equilateral
triangle
Square
Regular
Pentagon
Regular
Hexagon
Regular
Heptagon
Regular
Octagon
Regular
Nonagon
Regular
Decagon
Regular
Dodecagon
181

5. Describe the symmetry properties of each of the following triangles:


a)

b)

c)

4.4 Angle Properties


General Objectives: Calculate unknown angles using the geometrical properties
of intersecting and parallel lines and of simple plane figure.
Specific Objectives: Calculate unknown angles using the following geometrical
properties (reasons may be required but no formal proofs)
1. Angles at a point
2. Angles on a straight line and intersecting straight lines
3. Angle formed within parallel lines.
4. Angle properties of triangles and quadrilaterals.
5. Angle properties of regular polygons.
182

6. Angle in a semi-circle
7. Angle between tangent and radius
8. Angle properties of irregular polygons.
9. Angle at a center of a circle is twice the angle at the circumference.
10. Angles in the same segment are equal.
11. Angles in opposite segments are supplementary.

Angle Properties
Angles around a Point:

Angles around a point will always add up to 360 degrees.


The angles above all add to 360° : 53° + 80° + 140° + 87° = 360°
Example : What is angle "c"?

To find angle c we take the sum of the known angles and subtract that from 360°
183

Sum of known angles = 110° + 75° + 50° + 63°


= 298°
Angle c = 360° − 298°
= 62°
Adjacent angles on straight lines
Angles on one side of a straight line will always add to 180 degrees.

If a line is split into two and you know one angle, you can always find the other
one, they add to 180 degrees.
30° + 150° = 180°
Angles formed at Intersecting Lines
Two parallel lines intersected by a transversal form corresponding pairs of angles
that are congruent.

Adjacent angles share a common vertex and a common ray.


Two intersecting lines form pairs of adjacent angles that are supplementary. Also,
two intersecting lines form pairs of congruent angles, called vertical angles.
184

Corresponding pair:

Adjacent Pair:

Vertical Pair:

Angle formed within Parallel lines


When parallel lines get crossed by another line (which is called a Transversal), you
can see that many angles are the same, as in this example:
185

These angles can be made into pairs of angles which have special names.
Some of those special pairs of angles can be used to test if lines really are parallel:

Example :

Corresponding Angles are equal a=e


or
Alternate Interior Angles are equal c=f
or
Alternate Exterior Angles are equal b=g
or
Consecutive Interior Angles add up to 180° d + f = 180°
186

Therefore the two lines are parallel.


Angles in triangles
In a triangle, the three interior angles always add up to 180°.
This is a proof that the angles in a triangle add to 180°

The top line (that touches the top of the triangle) is running parallel to the base
of the triangle.
So: Angles A are equal, Angles B are equal.
And you can easily see that A + C + B does a complete rotation from one side of
the straight line to the other, or 180°
Example: Find the Missing Angle "C"

Triangle Missing Angle


Start With: A + B + C = 180°
Fill in what we know: 38° + 85° + C = 180°
Rearrange C = 180° - 38° - 85°
Calculate: C = 57°
A + B + C = 180°
187

Polygon
A polygon is a plane shape (two-dimensional) with straight sides. Examples
include triangles, quadrilaterals, pentagons, hexagons and so on.

A "Regular Polygon" has:


All sides equal and All angles equal.
Otherwise it is irregular.
So what can we know more about regular polygons? First of all, we can work out
angles.
The Exterior Angle is the angle between any side of a shape, and a line extended
from the next side.

Exterior Angle
All the Exterior Angles of a polygon add up to 360°, so:

360°
Each exterior angle must be , (where n is the number of sides)
n
188

Example : What is the exterior angle of a regular octagon?

Solution: An octagon has 8 sides, so:


360° 360°
Exterior angle = = = 45°
n 8

Interior Angles
The Interior Angle and Exterior Angle are measured from the same line, so they
add up to 180°.
189

Interior Angle = 180° − Exterior Angle


360°
We know the Exterior angle = , so:
n
𝟑𝟔𝟎°
Interior Angle = 180° −
𝐧
This can be rearranged like this:
360°
Interior Angle = 180° −
n
n ×180° n ×180°
= −
n n
180°
= (n-2) ×
n
𝟏𝟖𝟎°
So , Interior Angle = (n − 2) ×
𝐧

Example : What is the interior angle of a regular octagon?

A regular octagon has 8 sides, so:


360°
Exterior Angle = = 45°
8
Interior Angle = 180° − 45° = 135°
180°
So, Interior Angle = (n − 2) ×
n
190

180° 180°
= (8 − 2) × =6× = 135°
8 8
Irregular Polygon
Definition: Any polygon that is not a regular polygon. A polygon whose sides are
not of the same length or whose interior angles do not all have the same
measure.
An irregular polygon can have sides of any length and each interior angle can be
of any measure. They can be convex or concave, but all concave polygons are
irregular since the interior angles cannot all be the same. If you draw a polygon at
random, it would probably be irregular.
The formula for finding the sum of the interior angles of a polygon is the same,
whether the polygon is regular or irregular. So you would use the formula:
(n-2) x 180, where n is the number of sides in the polygon.
Example : Find the size of each interior angle of a Regular Decagon (10 sides)

Solution: Sum of Interior Angle


= (n − 2) × 180°
= (10 − 2)×180° = 8×180° = 1440°
And it is a Regular Decagon so:
1440°
Each interior angle = = 144°
10
191

Angle in a Semicircle
An angle inscribed in a semicircle is always a right angle:

(The end points are either end of a circle's diameter, the apex point can be
anywhere on the circumference.)
Because:
The inscribed angle 90° is half of the central angle 180°

Example : What is the size of Angle BAC?

The Angle in the Semicircle Theorem tells us that Angle ACB = 90°
Now use angles of a triangle that add up to 180° to find Angle BAC:
Angle BAC + 55° + 90° = 180°
BAC + 145°=180
BAC=180°-145°
Angle BAC = 35°
192

Angle between tangent and radius of a circle


A tangent to a circle is perpendicular to the radius at the point of tangency.

The angle at the centre of a circle


One of the rules of geometry is that the angle subtended at the centre of a circle
is double the size of the angle subtended at the edge from the same two points.
You will often see the word 'subtended' used in maths questions. Do not be put
off by it. 'Subtended' is just a technical way of saying 'made' or 'hanging down'.
Have a look at the diagrams below showing this rule:
193

Example : Work out the size of the angle subtended by the arc at the centre of
each circle.

Solution: The angle at the centre is always double the angle at the circumference.
So the first angle = 100°, and the second angle = 80°.

Angles in the same segment are equal


Angles subtended (made) by the same arc at the circumference are equal.

Exercise: What is the size of angle x?

Solution: x=40o
194

Angles in the opposite segment are supplementary


The alternate segment theorem states that an angle between a tangent and a
chord through the point of contact is equal to the angle in the alternate segment.

51°

74°

74° 51°

Problem Set 4.4

1. calculate the unknown angle in each of the following diagrams.

a)

b)
195

c)

d)

e)

f)
196

2. a)

b)

c)

d)
197

e)

f)

3. The diagram below shows two straight lines which cross each other. Calculate a,
b and c.

4. The diagram below shows a regular hexagon. The angles marked at the
center are all the same.

(a) What is the size of each angle marked at the center of the hexagon?
198

(b) What would these angles be if the polygon was a decagon (10
sides)?

(c) If the center angles were 30°, how many sides would the polygon
have?

5. Find the unknown angle in each triangle.

80°
a)

50° ?

b)
35° 120°

94° 23°
c)

?
81°
d)

62°
199

e)
?
38°

17°

f) 91°

? 41°

6. Find the exterior angles marked on these triangles.


a)

41°

61° ?

b)
18°
?

124°
200

c) ?

47°

? d)

25°

108°

7. Find three unknown exterior angles marked a, b and c on the triangle below:

a
61°

b
65°

8. Find the unknown angle or angles marked in each of the following diagrams.
a)

52°

a 121°
201

b)
a
152°

111°
c)

140°

a 130°
b
d)

130°

a c d

e)

b
c
202

f)
c
b d

a
110°

9. Given the labelled diagram with diameter AB, find x.


A

3x+ 5
x −1
60°

10. Given circle with center indicated. Find x.

4.5 Locus
General Objec ves: determine the locus (path) of a point under certain
condi ons.
Specific Objec ves:
203

Use the following loci and the method of intersecting loci for sets of points in two
dimensions:
1. Which are at a given distance from a given point.
2. Which are at a given distance from a given straight line.
3. Which are equidistant from two given points,
4. Which are equidistant from two given intersecting straight lines.
Locus
A shape created by the set of points whose position satisfies a given set of rules is
known as a locus.
For example:
A Circle is "the locus of points on a plane that are a certain distance from a central
point".
For a given line segment and its endpoints, the locus is the set of points that is the
same distance from both endpoints. From the definition of a midpoint, the
midpoint is equidistant from both endpoints. In addition, each point on a
perpendicular bisector of this segment has the same distance between each
endpoint (which is not the same distance as the other points on the perpendicular
bisector). So, the perpendicular bisector of a line segment is its locus.
To understand the concept of locus, imagine that the plane is composed of an
infinite number of points packed closely together. Then, select only those points
whose location meets certain rules. The selected points then form a shape,
perhaps a line or curve.
Locus Theorems
Theorem 1
The locus of points at a fixed distance 𝑑, from point 𝑃 is a circle with the given
point 𝑃 as its center and 𝑑 as its radius.
204

Theorem 2
The locus of points at a fixed distance , from a line, 𝑙 , is a pair of parallel lines 𝑑
distance from 𝑙 and on either side of 𝑙.

Theorem 3
The locus of points equidistant from two points, P and Q, is the perpendicular
bisector of the line segment determined by the two points.

Theorem 4
The locus of points equidistant from two parallel lines, 𝑙1 and 𝑙2 is a line parallel
to both 𝑙1 and 𝑙2 and midway between them.
205

Theorem 5
The locus of points equidistant from two intersecting lines, 𝑙1 and 𝑙2 is a pair of
bisectors that bisect the angles formed by 𝑙1 and 𝑙2 .

Examples of Locus of a Moving Point:

Example: A point moving in such a manner that three times of distance from the
𝑥-axis is greater by 7 than 4 times of its distance form the 𝑦-axis; find the
equation of its locus.

Solution: Let P (𝑥, 𝑦) be any position of the moving point on its locus. Then the
distance of P from the 𝑥-axis is y and its distance from the 𝑦 −axis is 𝑥.

By problem, 3𝑦 – 4𝑥 = 7,

This is the required equation to the locus of the moving point.

Example: Find the equation to the locus of a moving point which is always
equidistant from the points (2, -1) and (3, 2). What curve does the locus
represent?
206

Solution:

Let A (2, -1) and B (3, 2) be the given points and (𝑥, 𝑦) be the

co-ordinates of a point P on the required locus. Then,

PA2 = (𝑥 − 2)2 + (𝑦 + 1)2 and PB2 = (𝑥 − 3)2 + (𝑦 − 2)2

By problem, PA = PB or, PA2 = PB2

or, (𝑥 − 2)2 + (𝑦 + 1)2 = (𝑥 − 3)2 + (𝑦 − 2)2

or, 𝑥 2 - 4𝑥 + 4 + 𝑦2 + 2𝑦 + 1 = 𝑥 2 – 6𝑥 + 9 + 𝑦2 – 4𝑦 + 4

or, 2𝑥 + 6𝑦 = 8

or, 𝑥 + 3𝑦 = 4 ……… (1)

This is the required equation to the locus of the moving point

Clearly, equation (1) is a first degree equation in 𝑥 and 𝑦; hence, the locus of P is a
straight line whose equation is 𝑥 + 3𝑦 = 4.

Example: A and B are two given points whose co-ordinates are (-5, 3) and (2, 4)
respectively. A point P moves in such a manner that PA : PB = 3 : 2. Find the
equation to the locus traced out by P. what curve does it represent?

Solution: Let (h, k) be the co-ordinates of any position of the moving point on its
locus. By question,
𝑃𝐴 3
=
𝑃𝐵 2

or, 3 ∙ PB = 2 ∙ PA

or, 9 ∙ PB2 = 4 ∙ PA2

Or, 9[(h - 2)2 + (k - 4)2] = 4[(h + 5)2 + (k - 3)2]


207

or, 9 [h2 - 4h + 4 + k2 - 8k + 16] = 4[h2 + 10h + 25 + k2 - 6k + 9]

Or, 5h2 + 5k2 – 76h – 48k + 44 = 0

Therefore , the required equation to the locus traces out by P is

5𝑥 2 + 5𝑦2 – 76𝑥 – 48𝑦 + 44 = 0 ……….. (1)

We see that the equation (1) is a second degree equation in 𝑥, 𝑦 and its
coefficients of 𝑥 2 and 𝑦2 are equal and coefficients of 𝑥𝑦 is zero.

Therefore, equation (1) represents a circle.

Therefore, the locus of P represents the equation of a circle.

Example: Find the locus of a moving point which forms a triangle of area 21
square units with the point (2, -7) and (-4, 3).

Solution: Let the given point be A (2, -7) and B (-4, 3) and the moving point P
(say), which forms a triangle of area 21 square units with A and B, have co-
ordinates (x, y). Thus, by question area of the triangle PAB is 21 square units.
Hence, we have,

Therefore, the required equation to the locus of the moving point is 5𝑥 + 3𝑦 = 10


or, 5𝑥 + 3𝑦 + 21 = 0.

½ | (6 – 4𝑦 - 7𝑥) – ( 28 + 3𝑥 + 2𝑦) | = 21

or, |6 – 28 - 4𝑦 – 2𝑦 - 7𝑥 – 3𝑥 | = 42

or, 10𝑥 + 6𝑦 + 22 = ±42

Therefore, either, 10𝑥 + 6𝑦 + 22 = 42 i.e., 5𝑥 + 3𝑦 = 10


208

Problem Set 4.5


1. A point moving in such a manner that three times of distance from the x-
axis is greater by 7 than 4 times of its distance from the y-axis; find the
equation of its locus.
2. Find the equation to the locus of a moving point which is always equidistant
from the points (2, -1) and (3, 2). What curve does the locus represent?
3. A and B are two given point whose co-ordinates are (-5, 3) and (2, 4)
respectively. A point P moves in such a manner that PA: PB = 3 : 2. Find the
equation to the locus traced out by P. what curve does it represent?
4. Find the locus of a moving point which forms a triangle of area 21 square
units with the point (2, -7) and (-4, 3).
5. The sum of the distance of a moving point from the points (c, 0) and (-c, 0)
is always 2a units. Find the equation to the locus of the moving point.
209

Chapter 5

5.1 Algebraic Representation and Formulae


General Objectives: know how to express basic arithmetic processes algebraically
and how to substitute letters in formulae with numbers and how to transform
simple formulae
Specific Objectives:
1. Use letters to express generalized numbers
2. Express basic processes algebraically
3. Substitute numbers for words and letters in formulae
4. Transform simple formulae
5. Construct linear equations from given situations
6. Transform formulae involving roots, powers, fractions and factorization
5.1.1 Algebra
The methods or rules used in algebra are actually the extension of the methods or
rules used in arithmetic. In algebra, we use letters and symbols with or without
numbers to represent quantities for example, in algebra we have terms like
𝑥 2 , 5𝑎𝑏, −3𝑎2 𝑥𝑦 etc. Consider a term (−3𝑎2 𝑥𝑦) used in algebra. It has the
following parts:
Coefficient: The number part of this equation i.e., (−3) is called coefficient.
Variable: The letters or symbols used in an algebraic expression are called
variables ‘a’, ‘𝑥’ and ‘𝑦’ are three distinct variables.
Value of a Variable: A variable may have one or more values in a given situation.
If 𝑥 2 = 4, then x may have two values (2) and (-2). If a condition that 𝑥 is positive
number is applied, then the value of 𝑥 is only one that is (2). Value of a variable
can be changed according to the situation. That’s why they are called variables.
210

Let’s understand it in simple terms:


You know the missing number?

− 2 = 4

It is 6, right? Because 6 − 2 = 4. Easy stuff.


Well, in Algebra we don't use blank boxes, we use a letter (usually an 𝑥 or 𝑦, but
any letter is fine to use). So we write:

x − 2 = 4

It is really that simple. The letter (in this case an 𝑥) just means "we don't know
this yet", and is often called the unknown or the variable.
And when we solved it we write:

x = 6

Algebra is just like a puzzle where we start with something like "𝑥 − 2 = 4" and we
want to end up with something like "𝑥 = 6".

5.1.2 Four Basic Operations


1. Addition 𝒂 + 𝒃
The operation sign is +, and is called the plus sign. For example, if 𝒂
represents 3, and 𝒃 represents 4, then 𝒂 + 𝒃 represents 7.

2. Subtraction 𝒂 − 𝒃
The operation sign is −, and is called the minus sign. For example, if 𝒂
represents 8, and 𝒃 represents 2, then 𝒂 − 𝒃 represents 6.

3. Multiplication 𝒂 . 𝒃
The multiplication sign in algebra is a centered dot. We don’t use the
multiplication cross x because we don’t want to confuse it with letter 𝒙 .
For example, if 𝒂 represents 2, and 𝒃 represents 5, then 𝒂. 𝒃 = 𝟐 . 𝟓 = 𝟏𝟎
which means “2 times 5 equals 10”.
211

However, we often omit the multiplication dot and simply write 𝒂𝒃. In
other words, when there is no operation sign between two letters, or
between a letter and a number, it always means multiplication. 𝟐𝒙 means
“2 times 𝒙”.

4. Division 𝒂/𝒃
In algebra, we use the horizontal division bar. If 𝒂 represents 10, and 𝒃
𝒂 𝟏𝟎
represents 2, then = = 𝟓, means “10 divided by 2 is 5”.
𝒃 𝟐

Rules for carrying out basic operations


P.E.M.D.A.S

a. Perform the operations inside a parenthesis(small bracket) first


b. Then exponents
c. Then multiplication and division, from left to right
d. Then addition and subtraction, from left to right
e. You can also create a little phrase to memorize, as the sequence:
Please Excuse My Dear Aunt Sally

Examples
A. Evaluate the following arithmetic expression: 𝟑 + 𝟒 × 𝟐

Solution
Student 1 Student 2
3+4x2 3+4x2
=7 x 2 =3 + 8
=14 =11
212

Student 1 performed the operation of addition first, then multiplication; whereas


student 2 performed multiplication first, then addition.
We need a set of rules in order to avoid this kind of confusion.
Rule 1: First perform any calculations inside parenthesis (small bracket).
Rule 2: Next perform all multiplications and divisions, working from left to right.
Rule 3: Lastly, perform all additions and subtractions, working from left to right.
B. Simplify the expressions:
a) 2𝑥 + 3𝑥 − 3 − 4𝑥 − 12
b) 3𝑦 + 6𝑥 − 𝑦 + 3𝑥 + 2

a) Solution
+2𝑥, +3𝑥, −4𝑥 are like terms and when grouped give 𝒙
−3, −12 are like terms and when grouped give -15
Hence, 2𝑥 + 3𝑥 − 3 − 4𝑥 − 12 = 𝑥 − 15
b) Solution
+3𝑦, −𝑦 are like terms and when grouped give 2y
+6𝑥, +3𝑥 are like terms and when grouped give 9𝒙
Hence, 3𝑦 + 6𝑥 − 𝑦 + 3𝑥 + 2 = 2𝑦 + 9𝑥 + 2
C. Evaluate 3 + 6 x (5 + 4) ÷ 3 - 7 using the order of operations.
Solution

Step 1: 3 + 6 x (5 + 4) ÷ 3 - 7 = 3 + 6 x 9 ÷ 3 - 7 Parenthesis (small


bracket)
Step 2: 3 + 6 x 9 ÷ 3 - 7 = 3 + 54 ÷ 3 - 7 Multiplication
Step 3: 3 + 54 ÷ 3 - 7 = 3 + 18 - 7 Division
Step 4: 3 + 18 - 7 = 21 - 7 Addition
Step 5: 21 - 7 = 14 Subtraction
213

𝟑𝟔−𝟔
D. Evaluate the arithmetic expression
𝟏𝟐+𝟑

Solution
As the above expression involves a fraction bar therefore, we have to divide the
numerator by the denominator. All calculations must be performed before
dividing.
36 − 6 (36 − 6)
=
12 + 3 (12 + 3)
Evaluating this expression we get:
36 − 6 (36 − 6) 30
= = =2
12 + 3 (12 + 3) 15

𝟐𝟐
E. Solve the expression using the order of operations: 𝟑 × (𝟓 + 𝟖) − +𝟑
𝟒

Solution
I. Parenthesis(small bracket) first: 5 + 8 = 13
22
3 × 13 − + 3
4
2
II. Exponent next: square the 2 or 2 = 4
4
3 × 13 − + 3
4

III. Multiplication and Division next (3 x 13) (4 / 4) left to right:


39 − 1 + 3

IV. Addition and Subtraction next left to right:


39 − 1 + 3 = 41

Note that we first subtracted 1 from 39 (left to right) and then added 3 for
the correct answer, 41.
214

5.1.3 Use of Brackets


For removing the brackets from an algebraic expression, the following rules are
very important:
Rules
i. When a bracket is preceded by the sign “+”, the bracket can be removed
without making any change in the expression ,e.g., 𝑎 + (𝑏 − 𝑐) → 𝑎 + 𝑏 − 𝑐.
ii. When a bracket is preceded by the sign “-”, the bracket can be removed
and the sign of every term within the bracket changes, e.g.,
𝑎 − (𝑏 − 𝑐 + 𝑑) → 𝑎 − 𝑏 + 𝑐 − 𝑑.
iii. For any algebraic expression, different types of brackets are removed in the
following order:
a) " − " Bar
b) ( ) Parentheses or small bracket
c) { } Braces or medium bracket
d) [ ] Square or big bracket
Examples
A. Simplify the following expressions:
a) (2𝑎 − 3𝑏) − (3𝑎 + 4𝑏) − (𝑏 − 2𝑎)
b) (𝑥 − 3) − (𝑦 − 4)
c) 𝑎 − [𝑏 − (𝑐 − 𝑑 + 𝑒)]
d) 𝑎 − [2𝑎 − {3𝑎 + (2𝑎 − 𝑎 ̅̅̅̅̅̅̅
+ 1 )}]

a) Solution
(2𝑎 − 3𝑏) − (𝟑𝒂 + 𝟒𝒃) − (𝒃 − 𝟐𝒂)
By removing brackets in the expression, both brackets are preceded by a minus
sign so we’ll follow Rule No. 2

= 2𝑎 − 3𝑏 − 3𝑎 − 4𝑏 − 𝑏 + 2𝑎

= 𝑎 − 8𝑏
215

b) Solution
(𝑥 − 3) − (𝒚 − 𝟒) = 𝑥 − 3 − 𝑦 + 4
(𝑥 − 3) − (𝑦 − 4) = 𝑥 − 𝑦 + 1

The sign preceding (𝑥 − 3) is understood to be + . Therefore the signs within


those parentheses do not change. But the sign preceding (𝑦 − 4)is minus.
Therefore, 𝑦 changes to −𝑦 , and −4 changes to +4.
Finally, it is a format in algebra to write the literal terms, 𝑥 − 𝑦, to the left of
the numerical term.

c) Solution
We will remove all the grouping symbols. We will do it by removing the
brackets first. Then we will do it again removing the parentheses first. So,
upon removing the brackets:
𝑎 − [𝑏 − (𝑐 − 𝑑 + 𝑒)] = 𝑎 − 𝑏 + (𝑐 − 𝑑 + 𝑒)
Within the brackets, there are two terms. The first term is b. The second term
is −(𝑐 − 𝑑 + 𝑒). Since the brackets are preceded by −, the sign of each of the
two terms changes. The signs within the term (𝑐 − 𝑑 + 𝑒) do not change.
Finally, we remove the parentheses, which are preceded by + :
𝑎−𝑏+𝑐−𝑑+𝑒
d) Solution
Simplify 𝑎 − [2𝑎 − {3𝑎 + (2𝑎 − ̅̅̅̅̅̅̅
𝑎 + 1 )}]
= 𝑎 − [2𝑎 − {3𝑎 + (2𝑎 − 𝑎 − 1 )}] (Removing Bar)
= 𝑎 − [2𝑎 − {3𝑎 + 2𝑎 − 𝑎 − 1}] (Removing Small
Bracket)
= 𝑎 − [2𝑎 − 3𝑎 − 2𝑎 + 𝑎 + 1] (Removing Medium
Bracket)
= 𝑎 − 2𝑎 + 3𝑎 + 2𝑎 − 𝑎 − 1] (Removing Big Bracket)
= 3𝑎 − 1

5.1.4 Evaluation of Algebraic Expressions


If in any algebraic expression, variables are replaced by the given numerical
values, the result obtained is the value of the expression for a given value of the
variables, e.g.to find the value of 5𝑥 + 1 at 𝑥 = 2. We put 2 in place of 𝑥. Thus,
5𝑥 + 1 = 5(2) + 1 = 10 + 1 = 11
216

Examples
A. Evaluate the expression 𝟐𝒙𝟑 − 𝒙𝟐 + 𝒚 for 𝒙 = 𝟑 and 𝒚 = −𝟐
Solution
Evaluate 𝟐𝒙𝟑 − 𝒙𝟐 + 𝒚 for 𝒙 = 𝟑 Make sure the equation is clear and
and 𝒚 = −𝟐 you know which variable is which. It's
a good idea to write the expression
down and what each variable is. Leave
yourself enough room to work out the
problem line by line, with each step
right below the previous one.
𝟐(𝟑)𝟑 − (𝟑)𝟐 + (−𝟐) Replace each variable in the
expression with its value. In this
example, this means each 𝑥 becomes a
3 and each 𝑦 becomes a -2. It's a good
idea to use parentheses to keep track
of this.
𝟐(𝟐𝟕) − 𝟗 + (−𝟐) Perform operations with exponents.
𝟓𝟒 − 𝟗 + (−𝟐) Perform operations with multiplication
and division.
𝟒𝟑 Perform operations with addition and
subtraction.

B. Evaluate 𝒙𝟒 + 𝟑𝒙𝟑 − 𝒙𝟐 + 𝟔 𝒇𝒐𝒓 𝒙 = −𝟑


Solution
(–3)4 + 3(–3)3 – (–3)2 + 6
= 81 + 3(–27) – (9) + 6
= 81 – 81 – 9 + 6
= –3
C. Evaluate 𝟑𝒙𝟐 − 𝟏𝟐𝒙 + 𝟒 𝒇𝒐𝒓 𝒙 = −𝟐
Solution
3(–2)2 – 12(–2) + 4 = 3(4) + 24 + 4 = 12 + 24 + 4 = 40
217

5.1.5 Problem Set 5.1


√𝑤
1. Evaluate 3𝑥 2 − 2𝑦 3 + |𝑥𝑦| for 𝑥 = 6, 𝑦 = −3 𝑎𝑛𝑑 𝑤 = 81

2. Evaluate 4𝑥 2/3 − 5 = 11

5.1.6 Formulae
For algebra there are many formulae with the help of which results are obtained
without going through the operations of multiplication and division. These
formulae have special significance in algebra because of their usefulness.
Formula I
(𝑎 + 𝑏)2 = 𝑎2 + 2𝑎𝑏 + 𝑏 2
We can prove this by two different methods.
Method I
(𝑎 + 𝑏)2 = (𝑎 + 𝑏)(𝑎 + 𝑏)
= 𝑎(𝑎 + 𝑏) + 𝑏(𝑎 + 𝑏)
= 𝑎2 + 𝑎𝑏 + 𝑏𝑎 + 𝑏 2
= 𝑎2 + 𝑎𝑏 + 𝑎𝑏 + 𝑏 2
= 𝑎2 + 2𝑎𝑏 + 𝑏 2
Method II
218

Suppose AB = a units, BC = b units,


⟹AC = AB + BC = (a + b) units

AF = AG + GF = (a + b) units
Area of square ACDF = AC x AF = (a + b) x (a + b)
= (𝒂 + 𝒃)𝟐 𝒔𝒒. 𝒖𝒏𝒊𝒕𝒔 … … … … . . 𝐈

Area of square ACDF = Area of square ABHG + Area of rectangle BCIH + Area of
Square HIDE + Area of rectangle GHEF
= (AB x AG) + (BH x BC) +( HI x HE) + (GH x GF)
= (a x a) + (a x b) + (b x b) + (a x b)
= 𝒂𝟐 + 𝟐𝒂𝒃 + 𝒃𝟐 𝒔𝒒. 𝒖𝒏𝒊𝒕𝒔 … … … … . . 𝐈𝐈

Therefore, from I and II

(𝑎 + 𝑏)2 = 𝑎2 + 2𝑎𝑏 + 𝑏 2

Formula II
(𝑎 − 𝑏)2 = 𝑎2 − 2𝑎𝑏 + 𝑏 2
Method I
(𝑎 − 𝑏)2 = (𝑎 − 𝑏)(𝑎 − 𝑏)
= 𝑎(𝑎 − 𝑏) − 𝑏(𝑎 − 𝑏)
= 𝑎2 − 𝑎𝑏 − 𝑏𝑎 + 𝑏 2
= 𝑎2 − 𝑎𝑏 − 𝑎𝑏 + 𝑏 2
= 𝑎2 − 2𝑎𝑏 + 𝑏 2
219

Method II

Suppose AC = AF = A units
AB = AG = b units
⟹BC = AC - AB = (a - b) units
GF = AF - AG = (a - b) units
HI = BC = (a - b) units and HE = GF = (a - b) units
Since, Area of the square HIDE = HI x HE
= (a - b) x (a - b)
= (𝒂 − 𝒃)𝟐 𝒔𝒒. 𝒖𝒏𝒊𝒕𝒔 … … … … . . 𝐈
Area of square HIDE = Area of square ACDF - Area of rectangle ACIG - Area of
rectangle GHEF
= (AC x AF) – (AC x AG) – (GH x GF)
= a.a - a.b - (a-b).b
= a2 - ab - b(a-b)
= a2 - ab - ab + b2
= 𝒂𝟐 − 𝟐𝒂𝒃 + 𝒃𝟐 𝒔𝒒. 𝒖𝒏𝒊𝒕𝒔 … … … … . . 𝐈𝐈

Hence, From I and II


(𝑎 − 𝑏)2 = 𝑎2 − 2𝑎𝑏 + 𝑏 2
220

Deductions from Formulas I and II


I. (𝑎 + 𝑏)2 = 𝑎2 + 2𝑎𝑏 + 𝑏 2
II. (𝑎 − 𝑏)2 = 𝑎2 − 2𝑎𝑏 + 𝑏 2
III. 4𝑎𝑏 = (𝑎 + 𝑏)2 − (𝑎 − 𝑏)2

I. (𝑎 + 𝑏)2 = 𝑎2 + 2𝑎𝑏 + 𝑏 2
= 𝑎2 − 2𝑎𝑏 + 2𝑎𝑏 + 2𝑎𝑏 + 𝑏 2
= 𝑎2 − 2𝑎𝑏 + 4𝑎𝑏 + 𝑏 2
= (𝑎 − 𝑏)2 + 4𝑎𝑏 … (𝑖)

II. From (𝑖) (𝑎 + 𝑏)2 − 4𝑎𝑏 = (𝑎 − 𝑏)2 + 4𝑎𝑏 − 4𝑎𝑏


= (𝑎 − 𝑏)2

III. From (𝑖)


(𝑎 + 𝑏)2 − (𝑎 − 𝑏)2 = 4𝑎𝑏
𝑎2 + 2𝑎𝑏 + 𝑏 2 − (𝑎2 − 2𝑎𝑏 + 𝑏 2 )
𝑎2 + 2𝑎𝑏 + 𝑏 2 − 𝑎2 + 2𝑎𝑏 − 𝑏 2
= 𝑎2 − 𝑎2 + 𝑏 2 − 𝑏 2 + 2𝑎𝑏 + 2𝑎𝑏
= 4𝑎𝑏

Formula III
𝑎2 − 𝑏 2 = (𝑎 + 𝑏)(𝑎 − 𝑏)
Here (𝑎 + 𝑏)(𝑎 − 𝑏) = 𝑎2 − 𝑎𝑏 + 𝑎𝑏 − 𝑏 2 = 𝑎2 − 𝑏 2

Method I
We begin by giving a geometric proof of an algebraic identity which involves
cutting and translating something not permitted in Euclidean geometry.
221

𝑎2 𝑏2

Method II

"If a straight line be cut into equal and unequal segments, the rectangle contained
by the unequal segments of the whole together with the square on the straight
line between the points of the section is equal to the square on the half.''
Proof of Theorem: Rectangle ABFE is equal to rectangle BDHF. Rectangle BCGF is
equal to rectangle GHKJ. If square FGJI is added to these two rectangles (which
222

together form rectangle ACGE which is ''contained by the unequal segments of


the whole'') what we end up with is precisely rectangle BDKI, which is constructed
''on the half.'' Thus we have the equality (𝑎 + 𝑏)(𝑎 − 𝑏) + 𝑏 2 = 𝑎2 which is
equivalent to the equality above but does not contain the difficult-to-interpret
subtraction of areas.
Formula IV
(𝑥 + 𝑎)(𝑥 − 𝑏) = 𝑥 2 + (𝑎 − 𝑏)𝑥 − 𝑎𝑏
Proof
(𝑥 + 𝑎)(𝑥 − 𝑏) = 𝑥(𝑥 − 𝑏) + 𝑎(𝑥 − 𝑏)
= 𝑥 2 − 𝑏𝑥 + 𝑎𝑥 − 𝑎𝑏
= 𝑥 2 + (𝑎 − 𝑏)𝑥 − 𝑎𝑏

𝒃 𝑏𝑥 𝑎𝑏

𝒙
𝒙−𝒃 𝑥 2 − 𝑏𝑥 𝑎𝑥 − 𝑎𝑏

𝒙 𝒂
(𝑥 + 𝑎)(𝑥 − 𝑏) = 𝑥 2 − 𝑏𝑥 + 𝑎𝑥 − 𝑎𝑏

Examples
A. Find the square of 𝒂 + 𝟓
(𝑎 + 5)2 = (𝑎)2 + 2(𝑎)(5) + (5)2
= 𝑎2 + 10𝑎 + 25

B. Find the square of 𝟐𝒂 + 𝟑𝒃


(2𝑎 + 3𝑏)2 = (2𝑎)2 + 2(2𝑎)(3𝑏) + (3𝑏)2
= 4𝑎2 + 12𝑎𝑏 + 9𝑏 2

C. Simplify (𝟐𝒙 − 𝟑𝒚)𝟐 − (𝒙 − 𝟐𝒚)𝟐


223

= [(2𝑥)2 − 2(2𝑥)(3𝑦) + (3𝑦)2 ] − [(𝑥)2 − 2(𝑥)(2𝑦) + (2𝑦)2 ]


= (4𝑥 2 − 12𝑥𝑦 + 9𝑦 2 ) − (𝑥 2 − 4𝑥𝑦 + 4𝑦 2 )
= 4𝑥 2 − 12𝑥𝑦 + 9𝑦 2 − 𝑥 2 + 4𝑥𝑦 − 4𝑦 2
= 3𝑥 2 − 8𝑥𝑦 + 5𝑦 2
D. If 𝒂 + 𝒃 = 𝟔, and 𝒂𝒃 = 𝟓, find the value of 𝒂𝟐 + 𝒃𝟐 .
Here 𝑎 + 𝑏 = 6, 𝑎𝑏 = 5 → (𝑎 + 𝑏)2 = 𝑎2 + 𝑏 2 + 2𝑎𝑏
(6)2 = 𝑎2 + 𝑏 2 + 2 × 5
36 = 𝑎2 + 𝑏 2 + 10
𝑎2 + 𝑏 2 = 36 − 10 = 20
E. Find the value of 𝟏𝟎𝟑 × 𝟗𝟕 with the help of a formula.
Here we express 103 as 𝑎 + 𝑏 and 97 as 𝑎 − 𝑏. The product 103 × 97 can
be written as
(100 + 3)(100 − 3) which is:
= (100)2 − (3)2
= 10000 − 9 = 9991
F. Find the product (𝒙 + 𝟑)(𝒙 + 𝟓)
(𝑥 + 3)(𝑥 + 5) = 𝑥 2 + (3 + 5)𝑥 + 15 = 𝑥 2 + 8𝑥 + 15

5.2 Algebraic Manipulation


General Objectives: manipulate algebraic expressions and extract factors
Specific Objectives:
1. Multiply a monomial by a polynomial
2. Use brackets and extract common factors
3. Expand products of algebraic expressions
4. Factorize where possible expressions of the form 𝑎𝑥 + 𝑎𝑦 ; 𝑎𝑥 + 𝑏𝑥 +
𝑘𝑎𝑦 + 𝑘𝑏𝑦 ; 𝑎2 𝑥 2 − 𝑏 2 𝑦 2 ; 𝑎2 + 2𝑎𝑏 + 𝑏 2 ; 𝑎𝑥 2 + 𝑏𝑥 + 𝑐
5. Manipulate algebraic fractions e.g.,
𝑥 𝑥 − 4 2𝑥 2 3𝑎 5𝑎𝑏 1 2
+ , − , 𝑥 , −
3 2 3 𝑥−3 4 3 𝑥−2 𝑥−3
224

𝑥 2 −2𝑥
6. Factorize and simplify expressions such as
𝑥 2 −5𝑥+6
2
7. Write Quadratic expressions in the form 𝑎(𝑥 + 𝑝) +q
5.2.1 Algebraic Expressions
All expressions that connect variables, constants etc. by algebraic operations of
addition, subtraction, multiplication and division are called algebraic expressions
for example, 2𝑥𝑦 + 𝑦, 𝑦 3 + 𝑦 9 , 𝑧 3 − 5𝑎𝑏
a) Monomial
An algebraic expression that has only one term is called monomial. For
example, 7, 𝑥 2 , 2𝑦 3 , −4𝑥𝑦 3 etc.

b) Binomial
An algebraic expression having only two terms is called binomial. For
example, 𝑥 2 + 𝑦 2 , 2𝑥 4 + 12 𝑎𝑛𝑑 3𝑥 3 − 9

c) Trinomial
An algebraic expression with three terms is called trinomial. For example,
3𝑥 2 + 2𝑦 + 2, 3𝑥 2 + 15𝑥 − 1 𝑎𝑛𝑑 7𝑥 2 − 2𝑥𝑦 + 2𝑥 2 𝑦 6

d) Polynomial
An algebraic expression with more than three terms is called polynomial. In
general, binomials and trinomials are also included or called polynomial.
For example: 3𝑥 2 + 53, 5𝑥 6 + 7, 15𝑥 2 − 2𝑥𝑦𝑧 + 7𝑥𝑦 2 + 1

5.2.2 Factors
Numbers have factors:

And expressions like 𝑥 2 + 4𝑥 + 3 also have factors:


225

Factoring
Factorization is the opposite process of expanding brackets. For example,
expanding brackets would require 2(𝑥 + 1) to be written as 2𝑥 + 2. Factorization
would be to start with 2𝑥 + 2 and to end up with 2(𝑥 + 1).

The two expressions 2(𝑥 + 1) and 2𝑥 + 2 are equivalent; they have the same
value for all values of 𝑥. Factoring is like "splitting" an expression into a
multiplication of simpler expressions.
Identities to Remember
 𝑎2 − 𝑏 2 = (𝑎 + 𝑏)(𝑎 − 𝑏)
 𝑎2 + 2𝑎𝑏 + 𝑏 2 = (𝑎 + 𝑏)(𝑎 + 𝑏)
 𝑎2 − 2𝑎𝑏 + 𝑏 2 = (𝑎 − 𝑏)(𝑎 − 𝑏)
 𝑎3 + 𝑏 3 = (𝑎 + 𝑏)( 𝑎2 − 𝑎𝑏 + 𝑏 2 )
 𝑎3 − 𝑏 3 = (𝑎 − 𝑏)( 𝑎2 + 𝑎𝑏 + 𝑏 2 )
 𝑎3 + 3𝑎2 𝑏 + 3𝑎𝑏 2 + 𝑏 3 = (𝑎 + 𝑏)3
 𝑎3 − 3𝑎2 𝑏 + 3𝑎𝑏 2 − 𝑏 3 = (𝑎 − 𝑏)3
Common Factors
Factorizing based on common factors relies on there being factors common to all
the terms. For example, 2𝑥 − 6𝑥 2 can be factorized as follows:
2𝑥 − 6𝑥 2 = 2𝑥(1 − 3𝑥)
Difference of Two Squares
We have seen that
(𝑎𝑥 + 𝑏)(𝑎𝑥 − 𝑏) can be expanded to 𝑎2 𝑥 2 − 𝑏 2
Therefore,
𝑎2 𝑥 2 − 𝑏 2 can be factorized as (𝑎𝑥 + 𝑏)(𝑎𝑥 − 𝑏)
226

For example,

𝑥 2 − 16 can be written as 𝑥 2 − 42 which is a difference of two squares.

Therefore, the factors of 𝑥 2 − 16 are (𝑥 − 4) and (𝑥 + 4). In order to find a


difference of two squares, we look for expressions:

 Consisting of two terms


 With terms that have different signs (one negative, one positive)
 With each term a perfect square.

For example: 𝑎2 − 1, 4𝑥 2 − 𝑦 2 , −49 + 𝑝4

Examples
A. Factor 𝟐𝒚 + 𝟔
Solution
Both 2𝑦 and 6 have a common factor of 2:
 2𝑦 is 2 × 𝑦
 6 is 2 × 3
The whole expression can be factorized into:
2𝑦 + 6 = 2(𝑦 + 3)
So 2𝑦 + 6 has been “factored into” 𝟐 and (𝒚 + 𝟑)
B. Factor 𝟒𝒙𝟐 − 𝟗
Solution
It might be considered as “the difference of squares”.
227

Because 4𝑥 2 is (2𝑥)2 and 9 is (3)2


So, we have
4𝑥 2 − 9 = (2𝑥)2 − (3)2
And that can be produced by the difference of squares formula:
(𝑎 + 𝑏)(𝑎 − 𝑏) = 𝑎2 − 𝑏 2
Where 𝑎 is 2𝑥 and 𝑏 is 3.
(2𝑥 + 3)(2𝑥 − 3) = (2𝑥)2 − (3)2 = 4𝑥 2 − 9
So the factors of 4𝑥 2 − 9 are (2𝑥 + 3) and (2𝑥 − 3)

Factorizing using a switch around in brackets


C. Factorize 𝟓(𝒂 − 𝟐) − 𝒃(𝟐 − 𝒂)
Solution
Use a “switch around” strategy to find the common factor.
Notice that (2 − 𝑎) = −(𝑎 − 2)
5(𝑎 − 2) − 𝑏(2 − 𝑎) = 5(𝑎 − 2) − [−𝑏(𝑎 − 2)]
= 5(𝑎 − 2) + 𝑏(𝑎 − 2)
= (𝑎 − 2)(5 + 𝑏)
D. Factorize 𝟑𝒂(𝒂𝟐 − 𝟒) − 𝟕(𝒂𝟐 − 𝟒)
Solution
Take out the common factor (𝑎2 − 4)

3𝑎(𝑎2 − 4) − 7(𝑎2 − 4) = (𝑎2 − 4)(3𝑎 − 7)

Factorize the difference of two squares (𝑎2 − 4)

(𝑎2 − 4)(3𝑎 − 7) = (𝑎 − 2)(𝑎 + 2)(3𝑎 − 7)


228

5.2.3 Problem Set 5.2


1. Evaluate the following expressions
a. (𝑥 2 − 𝑥)3𝑦
b. (𝑥 + 𝑎)(2𝑥 + 3𝑦 − 5)

2. Factorize 24𝑥 2 𝑦 + 34𝑥𝑦 + 12𝑦


3. Factorize −2𝑥 3 − 6𝑥 2 + 56𝑥

5.3 Polynomials
General Objectives: manipulate polynomials
Specific Objectives:
1. Carry out operations of addition, subtraction and multiplication of
polynomials.
2. Carry out division of a polynomial by a binomial expression and identify the
quotient and the remainder.
Addition of polynomials:
Let's find the sum of the following two polynomials:
(3𝑦5 − 2𝑦 + 𝑦4 + 2𝑦3 + 5) and (2𝑦5 + 3𝑦3 + 2+ 7𝑦)
Step: Arrange the Polynomial in standard form. Standard form of a
polynomial just means that the term with the highest degree comes first and each
of the following terms follows.
= (3𝑦5 +2𝑦5 + 𝑦4 + 2𝑦3 + 3𝑦3 − 2𝑦+7𝑦 + 5+2)
= (5𝑦5 +𝑦4 +5𝑦3 +5𝑦+7)
Difference of polynomials:
Let's find the difference of the same two polynomials
(3𝑦5 − 2𝑦 + 𝑦4 + 2𝑦3 + 5) and (2𝑦5 + 3𝑦3 + 2+ 7𝑦)
= (3𝑦5 − 2𝑦 + 𝑦4 + 2𝑦3 + 5) - (2𝑦5 + 3𝑦3 + 2+ 7𝑦)
= (3𝑦5 -2𝑦5 + 𝑦4 + 2𝑦3 - 3𝑦3 − 2𝑦-7𝑦 + 5-2)
229

= (𝑦5 + 𝑦4 - 𝑦3 − 9𝑦 + 3)
Multiplication of polynomials:
Let's find the product of polynomials: (3𝑥 – 4𝑦) (5𝑥 – 2𝑦)
Step 1: Distribute each term of the first polynomial to every term of the second
polynomial. In this case, we need to distribute 3𝑥 and –4𝑦.

Step 2: Combine like terms.

Division of a polynomial by a binomial expression:

Divide 2𝑥 4 - 9𝑥 3 +21𝑥 2 - 26𝑥 + 12 by 2𝑥 - 3.


Remember Long
Division from
Grade 4-7

The expression 𝑥 3 - 3𝑥 2 + 6𝑥 - 4 is called the quotient.


The expression 2𝑥-3 is called the divisor and the term 0 is called the remainder.

What is an algebraic expression?

The combination of constants and variables, connected by signs of fundamental


operations (+, −, ×, ÷) is called an algebraic expression.
230

2𝑥 – 3𝑦 + 9𝑧 is an algebraic expression.

In this algebraic expression 3𝑥² + 7𝑦³ − 4𝑥𝑦:

3𝑥², 7𝑦³, −4𝑥𝑦 are called terms of the expression.

Power of literal quantities means when a quantity is multiplied by itself, any


number of times, the product is called a power of that quantity. This product is
expressed by writing the number of factors in it to the right of the quantity and
slightly raised.

For example:

(i) m × m has two factors so to express it we can write m × m = m2

(ii) b × b × b has three factors so to express it we can write b × b × b = b3

(iii) z × z × z × z × z × z × z has seven factors so to express it we can write z × z × z ×


z × z × z × z = z7

Learn how to read and write the power of literal quantities

(i) Product of 𝑥 × 𝑥 is written as 𝑥 2 and it is read as 𝑥 squared or 𝑥 raised to the


power 2.

(ii) Product of 𝑦 × 𝑦 × 𝑦 is written as 𝑦3 and it is read as 𝑦 cubed or 𝑦 raised to the


power 3.

(iii) Product of n × n × n × n is written as n4 and it is read as forth power of n or n


raised to the power 4.

(iv) Product of 3 × 3 × 3 × 3 × 3 is written as 35 and it is read as fifth power of 3 or


3 raised to the power 5.
231

How to identify the base and exponent of the power of the given quantity?

(i) In a5 here a is called the base and 5 is called the exponent or index or power.

(ii) In Mn here M is called the base and n is called the exponent or index or power.

Solved examples:

1. Write a × a × b × b × b in index form.

a × a × b × b × b = a2b3

2. Express 5 × m × m × m × n × n in power form.

5 × m × m × m × n × n = 5m3n2

3. Express -5 × 3 × p × q × q × r in exponent form.

-5 × 3 × p × q × q × r = -15pq2r

4. Write 3𝑥 3𝑦4 in product form.

3𝑥 3𝑦4 = 3 × 𝑥 × 𝑥 × 𝑥 × 𝑦 × 𝑦 × 𝑦 × 𝑦

5. Express 9a4b2c3 in product form.

9a4b2c3 = 3 × 3 × a × a × a × a × b × b × c × c × c

Classification of algebraic expressions

Monomial: An algebraic expression containing only one term is called a


monomial.

5
For example; 3𝑥, -7, a²bc are all monomials.
9

Binomial: An algebraic expression containing two terms is called a binomial.


232

For example; 𝑥 – 7, 5𝑥 + 9𝑦, ab + c are all binomials.

Trinomial: An algebraic expression containing three terms is called a trinomial.

For example; 𝑥 – 𝑦 + 7, 3𝑥 + 4𝑦 – 5z, a³ + b² + c⁴ are all trinomials.

Multinomial: An algebraic expression of two or more than three terms is called a


multinomial

For example; 𝑥³ y² + 2𝑥²𝑦 – 3𝑥𝑦 + 7, a² + b² - 4c² - d², 𝑙 + m + n – p are all


multinomials.

Polynomial: In an algebraic expression, if the power of variables is a non-negative


integer; then that expression is called a polynomial.

For example; 3𝑥² + 4𝑥 + 7 is a polynomial.

3
While 𝑥² + is not a polynomial.
𝑥

3 3
[The power of 𝑥 in is negative. Therefore, =3𝑥 −1 ]
𝑥 𝑥
5√𝑥 + 2𝑥² - 5 is not a polynomial.

[The power of 𝑥 in 5√𝑥 is in fraction. Therefore, It is not a Polynomial]

Types of algebraic expressions may further be distinguished by the following five


categories.

They are: monomial, polynomial, binomial, trinomial, multinomial.

1. Monomial: An algebraic expression which consists of one non-zero term only is


called a monomial.

Examples of monomials:

a is a monomial with one variable a.


233

10ab2 is a monomial with two variables a and b.

5m2n is a monomial with two variables m and n.

-7pq is a monomial with two variables p and q.

5b3c is a monomial with two variables b and c.

2b is a monomial with one variable b.

2ax/3y is a monomial with three variables a, x and y.

k2 is a monomial with one variable k.

2. Polynomial: An algebraic expression which consists of one, two or more terms


is called a polynomial.

Examples of polynomials:

2a + 5b is a polynomial of two terms with two variables a and b.

3𝒙y + 5𝒙 + 1 is a polynomial of three terms with two variables 𝑥 and 𝑦.

3𝒚4 + 2𝒚3 + 7𝒚2 - 9𝒚 + 3/5 is a polynomial of five terms with one variable .

m + 5mn – 7m2n + nm2 + 9 is a polynomial of four terms with two variables m and
n.

3 + 7𝒙5 + 4𝒙2 is a polynomial of three terms with one variable𝑥.

3 + 5𝒙2 - 4𝒙2𝒚 + 5𝒙𝒚2 is a polynomial of four terms with two variables 𝑥 and 𝑦.

x + 5yz – 7z + 11 is a polynomial of four terms with three variables 𝑥, 𝑦 and 𝑧.

1 + 2p + 3p2 + 4p3 + 5p4 + 6p5 + 7p6 is a polynomial of seven terms with one
variable p.
234

3. Binomial: An algebraic expression which consists of two non-zero terms is


called a binomial.

Examples of binomials:

m + n is a binomial with two variables m and n.

a2 + 2b is a binomial with two variables a and b.

5𝒙3 – 9𝒚2 is a binomial with two variables 𝑥 and 𝑦.

-11p – q2 is a binomial with two variables p and q.

b3/2 + c/3 is a binomial with two variables b and c.

5m2n2 + 1/7 is a binomial with two variables m and n.

4. Trinomial: An algebraic expression of three non-zero terms only is called a


trinomial.

Examples of trinomial:

𝒙 + 𝒚 + z is a trinomial with three variables 𝑥, 𝑦 and z.

2a2 + 5a + 7 is a trinomial with one variable a.

𝒙𝒚 + 𝒙 + 2𝒚2 is a trinomial with two variables 𝑥 and 𝑦.

-7m5 + n3 – 3m2n2 is a trinomial with two variables m and n.

5abc – 7ab + 9ac is a trinomial with three variables a, b and c.

𝒙2/3 + a𝒚 – 6bz is a trinomial with five variables a, b, 𝑥, 𝑦 and z.

5. Multinomial: An algebraic expression of two or more than three terms is called


a multinomial.
235

Note: binomial and trinomial are the multinomial.

Examples of multinomial:

p + q is a multinomial of two terms with two variables p and q.

a + b + c is a multinomial of three terms with three variables a, b and c.

a + b + c + d is a multinomial of four terms with four variables a, b, c and d.

𝒙4 + 2x3 + 1/𝒙 + 1 is a multinomial of four terms with one variable 𝑥.

a + ab + b2 + bc + cd is a multinomial of five terms with four variables a, b, c and d.

5𝒙8 + 3𝒙7 + 2𝒙6 + 5𝒙5 - 2𝒙4 - 𝒙3 + 7𝒙2 - 𝒙 is a multinomial of eight terms with one
variable 𝑥.

Summary: What is a polynomial?

An algebraic expression which consists of one, two or more terms is called a


polynomial.

How to find a degree of a polynomial?

The degree of the polynomial is the greatest of the exponents (powers) of its
various terms.

Examples of polynomials and its degree:

Example: For polynomial 2𝒙2 - 3𝒙5 + 5𝒙6.

Solution: We observe that the above polynomial has three terms. Here the first
term is 2𝑥 2, the second term is -3𝑥 5 and the third term is 5𝑥 6.

Now we will determine the exponent of each term.


236

(i) The exponent of the first term 2𝑥 2 = 2

(ii) The exponent of the second term -3𝑥 5 = 5

(iii) The exponent of the third term 5𝑥 6 = 6

Since, the greatest exponent is 6, the degree of 2𝑥 2 - 3𝑥 5 + 5𝑥 6 is also 6.

Therefore, the degree of the polynomial 2𝑥 2 - 3𝑥 5 + 5𝑥 6 = 6

Example: Find the degree of the polynomial 16 + 8𝒙 – 12𝒙2 + 15𝒙3 - 𝒙4

Solution: We observe that the above polynomial has five terms. Here the first
term is 16, the second term is 8𝑥, the third term is – 12𝑥 2, the fourth term is
15𝑥 3 and the fifth term is - 𝑥 4.

Now we will determine the exponent of each term.

(i) The exponent of the first term 16 = 0

(ii) The exponent of the second term 8𝑥 = 1

(iii) The exponent of the third term – 12𝑥 2 = 2

(iv) The exponent of the fourth term 15𝑥 3 = 3

(v) The exponent of the fifth term - 𝑥 4 = 4

Since, the greatest exponent is 4, the degree of 16 + 8𝑥 – 12𝑥 2 + 15𝑥 3 - 𝑥 4 is also


4.

Therefore, the degree of the polynomial 16 + 8𝑥 – 12𝑥 2 + 15𝑥 3 - 𝑥 4 = 4.


237

Example: Find the degree of a polynomial 7𝒙 – 4

Solution: We observe that the above polynomial has two terms. Here the first
term is 7𝑥 and the second term is -4.

Now we will determine the exponent of each term.

(i) the exponent of the first term 7𝑥 = 1

(ii) the exponent of the second term -4 = 1

Since, the greatest exponent is 1, the degree of 7𝑥 – 4 is also 1.

Therefore, the degree of the polynomial 7𝑥 – 4 = 1.

Example: Find the degree of a polynomial 11𝒙3 - 13𝒙5 + 4𝒙

Solution: We observe that the above polynomial has three terms. Here the first
term is 11𝑥 3, the second term is - 13𝑥 5 and the third term is 4𝑥.

Now we will determine the exponent of each term.

(i) the exponent of the first term 11𝑥 3 = 3

(ii) the exponent of the second term - 13𝑥 5 = 5

(iii) the exponent of the third term 4𝑥 = 1

Since, the greatest exponent is 5, the degree of 11𝑥 3 - 13𝑥 5 + 4𝑥 is also 5.

Therefore, the degree of the polynomial 11𝑥 3 - 13𝑥 5 + 4𝑥 = 5.

Example: Find the degree of the polynomial 1 + 𝒙 + 𝒙2 + 𝒙3

Solution: We observe that the above polynomial has four terms. Here the first
term is 1, the second term is 𝑥, the third term is 𝑥 2 and the fourth term is 𝑥 3.
238

Now we will determine the exponent of each term.

(i) the exponent of the first term 1 = 0

(ii) the exponent of the second term 𝑥 = 1

(iii) the exponent of the third term 𝑥 2 = 2

(iv) the exponent of the fourth term 𝑥 3 = 3

Since, the greatest exponent is 3, the degree of 1 + 𝑥 + 𝑥 2 + 𝑥 3 is also 3.

Therefore, the degree of the polynomial 1 + 𝑥 + 𝑥 2 + 𝑥 3 = 3.

Example: Find the degree of a polynomial -2𝒙

Solution: We observe that the above polynomial has one term. Here the term is
-2𝑥.

Now we will determine the exponent of the term.

(i) The exponent of the first term -2𝑥 = 1

Therefore, the degree of the polynomial -2𝑥 = 1.

In addition of algebraic expressions, while adding algebraic expressions we collect


the like terms and add them. The sum of several like terms is the like term whose
coefficient is the sum of the coefficients of these like terms.

Two Methods to solve addition of algebraic expressions are:

Horizontal Method: In this method, all expressions are written in a horizontal line
and then the terms are arranged to collect all the groups of like terms and then
added.

Column Method: In this method each expression is written in a separate row such
that the like terms are arranged one below the other in a column. Then the
239

addition of terms is done column wise.

The following illustrations will illustrate these methods.

Examples on addition of algebraic expressions:

Example: Add: 6a + 8b - 7c, 2b + c - 4a and a - 3b - 2c

Solution:

Horizontal Method:

(6a + 8b - 7c) + (2b + c - 4a) + (a - 3b - 2c)

= 6a + 8b - 7c + 2b + c - 4a + a - 3b - 2c

Arrange the like terms together, then add.


Thus, the required addition

= 6a - 4a + a + 8b + 2b - 3b - 7c + c - 2c

= 3a + 7b - 8c

Column Method:

Solution:
Write the terms of the given expressions in the same order in the form of rows
with like terms below each other and adding column wise;

6a + 8b - 7c

- 4a + 2b + c

a - 3b - 2c

3a + 7b - 8c

= 3a + 7b - 8c
240

Example: Add: 5𝒙² + 7𝒚 - 8, 4𝒚 + 7 - 2𝒙² and 6 – 5𝒚 + 4𝒙².

Solution:

Write the given expressions in descending powers of 𝑥 in the form of rows with
like terms below each other and adding column wise;

5𝑥² + 7𝑦 - 8

- 2𝑥² + 4𝑦 + 7

4𝑥² – 5𝑦 + 6
___________
7𝑥² + 6𝑦 + 5
___________

= 7𝑥² + 6𝑦 + 5

Example: Add: 8𝒙² - 5𝒙𝒚 + 3𝒚², 2𝒙𝒚 - 6𝒚² + 3𝒙² and 𝒚² + 𝒙𝒚 - 6𝒙²

Solution:

Arrange the given expressions in descending powers of 𝑥 with like terms under
each other and adding column wise;

8𝑥² - 5𝑥𝑦 + 3𝑦²

3𝑥² + 2𝑥𝑦 - 6𝑦²

-6𝑥² + 𝑥𝑦 + 𝑦²
_____________
5𝑥² - 2𝑥𝑦 - 2𝑦²
_____________

= 5𝑥² - 2𝑥𝑦 - 2𝑦²


241

Example: Add: 11a² + 8b² - 9c², 5b² + 3c² - 4a² and 3a² - 4b² - 4c².

Solution:

Write the terms of the given expressions in the same order in the form of rows
with like terms below each other and adding column wise;

11a² + 8b² - 9c²

- 4a² + 5b² + 3c²

3a² - 4b² - 4c²


________________
10a² + 9b² - 10c²
________________

= 10a² + 9b² - 10c²

Example: Add 3𝒙 + 2y and 𝒙 + 𝒚.

Solution:

Horizontal Method:

(3𝑥 + 2𝑦) + (𝑥 + 𝑦)

Arrange the like terms together, then add.

Thus, the required addition

= 3𝑥 + 2𝑦 + 𝑥 + 𝑦

= 3𝑥 + 𝑥 + 2𝑦 +𝑦

= 4𝑥 + 3𝑦
242

Column Method:

Solution:

Arrange expressions in lines so that the like terms with their signs are below each
other i.e. like terms are in the same vertical column and then add different groups
of like terms.

3𝑥 + 2𝑦

+𝑥+𝑦
_________
4 𝑥 + 3𝑦

Example: Add: 𝒙 + 𝒚 + 3 and 3𝒙 + 2𝒚 + 5

Solution:

Horizontal Method:

(𝑥 + 𝑦 + 3) + (3𝑥 + 2𝑦 + 5)

= 𝑥 + 𝑦 + 3 + 3𝑥 + 2𝑦 + 5

Arrange the like terms together, then add.

Thus, the required addition

= 𝑥 + 3𝑥 + 𝑦 + 2𝑦 + 3 + 5

= 4𝑥 + 3𝑦 + 8

Column Method:

Solution: Arrange expressions in lines so that the like terms with their signs are
below each other i.e. like terms are in the same vertical column and then add
different groups of like terms.
243

𝑥+𝑦+3
+ 3𝑥 + 2𝑦 + 5
_________________
4𝑥 + 3𝑦 + 8

Example: Add: 2𝒙 + 3𝒚 + z and 2𝒙 - 𝒚 - z

Solution:

Horizontal Method:

(2𝑥 + 3𝑦 + z) + (2𝑥 - y – z)

=2𝑥 + 3𝑦 + z + 2𝑥 - y – z

Arrange the like terms together, then add.

Thus, the required addition

= 2𝑥 + 2𝑥 + 3𝑦 - 𝑦 + z - z

=4𝑥 + 2𝑦

Column Method:

Solution:
Arrange expressions in lines so that the like terms with their signs are below each
other i.e. like terms are in the same vertical column and then add different groups
of like terms.

2𝑥 + 3𝑦 + z

+ 2𝑥 - 𝑦 - z
_____________
4𝑥 + 2𝑦
244

Example: Add: 5𝒙³ – 2𝒚³ and 7𝒙³ – 3𝒚³

Solution:

Horizontal Method:

(5𝑥³ – 2𝑦³) + (7𝑥³ – 3𝑦³)

=5𝑥³ - 2𝑦³ + 7𝑥³ – 3𝑦³

Arrange the like terms together, then add.

Thus, the required addition

= 5𝑥³ + 7𝑥³ - 2𝑦³ - 3𝑦³

=12𝑥³ – 5𝑦³

Column Method:

Solution:

Arrange expressions in lines so that the like terms with their signs are below each
other i.e. like terms are in the same vertical column and then add different groups
of like terms.

5𝑥³ – 2𝑦³

+ 7𝑥³ - 3𝑦³
_____________
12𝑥³ – 5𝑦³
245

Example: Add: a² + b² + c² – 3abc and a² – b² + c² + abc

Solution:

Horizontal Method:

(a² + b² + c² - 3abc) + (a² – b² + c² + abc)

= a² + b² + c² - 3abc + a² – b² + c² + abc

Arrange the like terms together, then add.

Thus, the required addition

= (a² + a²) + (b² – b²) + (c² + c²) - 3abc + abc

= 2a² + 2c² -2abc

Column Method:

Solution:

Arrange expressions in lines so that the like terms with their signs are below each
other i.e. like terms are in the same vertical column and then add different groups
of like terms.

a² + b² + c² – 3abc

+ a² – b² + c² + abc
__________________
2a² + 0 + 2c² – 2abc
246

Example: Add: 𝒙𝒚² + 4𝒙²𝒚 – 7𝒙²𝒚 - 3𝒙𝒚² + 3 and 𝒙²𝒚 + 𝒙𝒚²

We have;

𝑥𝑦² + 4𝑥²𝑦 -7𝑥²𝑦 - 3𝑥𝑦² + 3

= - 2𝑥𝑦² - 3𝑥²𝑦 + 3

Solution:
Horizontal Method:

(𝑥𝑦² + 4𝑥²𝑦 – 7𝑥²𝑦 - 3𝑥𝑦² + 3) +(𝑥²𝑦 + 𝑥𝑦²)

= (-2𝑥𝑦² - 3𝑥²𝑦 + 3) + 𝑥²𝑦 + 𝑥𝑦²

= -2𝑥𝑦² - 3𝑥²𝑦 + 3 + 𝑥²𝑦 + 𝑥𝑦²

Arrange the like terms together, then add.


Thus, the required addition

= -2𝑥𝑦² + 𝑥𝑦² - 3𝑥²𝑦 + 𝑥²𝑦 + 3

= - 𝑥𝑦² - 2𝑥²𝑦 + 3

Column Method:

Solution:
Arrange expressions in lines so that the like terms with their signs are below each
other i.e. like terms are in the same vertical column and then add different groups
of like terms.

- 2𝑥𝑦² - 3𝑥²𝑦 +3

+ 𝑥𝑦² + 𝑥²𝑦
________________
- 𝑥𝑦² - 2𝑥²𝑦 + 3
247

Subtraction of algebraic expressions is explained in each step:

Step I: Arrange the terms of the given expressions in the same order.

Step II: Write the given expressions in two rows in such a way that the like terms
occur one below the other, keeping the expression to be subtracted in the second
row.

Step III: Change the sign of each term in the lower row from + to - and from - to
+.

Step IV: With new signs of the terms of lower row, add column wise.

Example: Subtract 4a + 5b - 3c from 6a - 3b + c


Solution:
6a - 3b + c

+ 4a + 5b - 3c

(-) (-) (+)


_____________
2a - 8b + 4c
_____________

Example: Subtract 3𝒙² - 6𝒙 - 4 from 5 + 𝒙 - 2𝒙².

Solution:
Arranging the terms of the given expressions in descending powers of 𝑥 and
subtracting column-wise;

- 2𝑥² + 𝑥 + 5

+ 3𝑥² - 6𝑥 - 4

(-) (+) (+)


_____________
- 5𝑥² + 7𝑥 + 9
_____________
248

Example: Subtract 3𝒙 + 𝒚 – 3z from 9𝒙 – 5𝒚 + z

Solution:

9𝑥 – 5𝑦 + z

+ 3𝑥 + 𝑦 – 3z

(-) (-) (+)


_____________
6𝑥 – 6𝑦 + 4z
_____________

MULTIPLICATION:

In multiplication of algebraic expression before taking up the product of


algebraic expressions, let us look at two simple rules.

(i) The product of two factors with like signs is positive, and the product of two
factors with unlike signs is negative.

(ii) if 𝑥 is a variable while m and n are positive integers, then

(𝑥ᵐ × 𝑥ⁿ) = (𝑥)𝑚+𝑛

Thus, (𝑥³ × 𝑥⁵) = 𝑥⁸, (𝑥⁶ + 𝑥⁴) = (𝑥)6+4 = (𝑥)10

I. Multiplication of Two Monomials

Rule:

Product of two monomials = (product of their numerical coefficients) × (product


of their variable parts)

Example: Find the product of: (i) 6𝑥𝑦 and -3𝑥²𝑦³


249

Solution:

(6𝑥𝑦) × (-3𝑥²𝑦³)

= {6 × (-3)} × {𝑥𝑦 × 𝑥²𝑦³}

= -18 (𝑥)3 (𝑦)4

= -18𝑥³𝑦⁴

(ii) 7ab², -4a²b and -5abc


Solution:

(7ab²) × (-4a²b) × (-5abc)

= {7 × (-4) × (-5)} × {ab² × a²b × abc}

= 140a⁴b⁴c.

II. Multiplication of a Polynomial by a Monomial

Rule:

Multiply each term of the polynomial by the monomial, using the distributive law
a × (b + c) = a × b + a × c

Example: Find each of the following products:

(i) 5a²b² × (3a² - 4ab + 6b²)

Solution:

5a²b² × (3a² - 4ab + 6b²)

= (5a²b²) × (3a²) + (5a²b²) × (-4ab) + (5a²b²) × (6b²)

= 15a⁴b² - 20a³b³ + 30a²b⁴


250

(ii) (-3𝑥²𝑦) × (4𝑥²𝑦 - 3𝑥𝑦² + 4𝑥 - 5𝑦)

Solution:

(-3𝑥²𝑦) × (4𝑥²𝑦 - 3𝑥𝑦² + 4𝑥 - 5𝑦)

= (-3𝑥²𝑦) × (4𝑥²𝑦) + (-3𝑥²𝑦) × (-3𝑥𝑦²) + (-3𝑥²𝑦) × (4𝑥) + (-3𝑥²𝑦) × (-5𝑦)

= -12𝑥⁴𝑦² + 9𝑥³𝑦³ - 12𝑥³𝑦 + 15𝑥²𝑦²

III. Multiplication of Two Binomials

Suppose (a + b) and (c + d) are two binomials. By using the distributive law of


multiplication over addition twice, we may find their product as given below.

(a + b) × (c + d)

= a × (c + d) + b × (c + d)

= (a × c + a × d) + (b × c + b × d)

= ac + ad + bc + bd

Note: This method is known as the horizontal method.

Example: (i) Multiply (3𝒙 + 5𝒚) and (5𝒙 - 7𝒚).

Solution:

(3𝑥 + 5𝑦) × (5𝑥 - 7𝑦)

= 3𝑥 × (5𝑥 - 7𝑦) + 5𝑦 × (5𝑥 - 7𝑦)

= (3𝑥 × 5𝑥 - 3𝑥 × 7𝑦) + (5𝑦 × 5𝑥 - 5𝑦 × 7𝑦)

= (15𝑥² - 21𝑥𝑦) + (25𝑥𝑦 - 35𝑦²)


251

= 15𝑥² - 21𝑥𝑦 + 25𝑥𝑦 - 35𝑦²

= 15𝑥² + 4𝑥𝑦 - 35𝑦²

Column wise multiplication

The multiplication can be performed column wise as shown below.

3𝑥 + 5𝑦

× (5𝑥 - 7𝑦)
_____________
15𝑥² + 25𝑥𝑦 ⇐ multiplication by 5𝑥

- 21𝑥𝑦 - 35𝑦² ⇐ multiplication by -7𝑦


__________________
15𝑥² + 4𝑥𝑦 - 35𝑦² ⇐ multiplication by (5𝑥 - 7𝑦)
__________________

(ii) Multiply (3𝑥² + 𝑦²) by (2𝑥² + 3𝑦²)

Solution:

Horizontal method,

= 3𝑥² (2𝑥² + 3𝑦²) + 𝑦² (2𝑥² + 3𝑦²)

= (6𝑥⁴ + 9𝑥²y²) + (2𝑥²𝑦² + 3𝑦⁴)

= 6𝑥⁴ + 9𝑥²𝑦² + 2𝑥²𝑦² + 3𝑦⁴

= 6𝑥⁴ + 11𝑥²𝑦² + 3𝑦⁴

Column method,

3𝑥² + 𝑦²

× (2𝑥² + 3𝑦 2 )
252

_____________
6𝑥⁴ + 2𝑥²𝑦² ⇐ multiplication by 2𝑥²

+ 9𝑥²𝑦² + 3𝑦⁴ ⇐ multiplication by 3𝑦 2


___________________
6𝑥⁴ + 11𝑥²𝑦² + 3𝑦⁴ ⇐ multiplication by (2𝑥² + 3𝑦³)
___________________

IV. Multiplication by Polynomial

We may extend the above result for two polynomials, as shown below.

(i) Multiply (5𝑥² – 6𝑥 + 9) by (2𝑥 -3)

5𝑥² – 6𝑥 + 9

× (2𝑥 - 3)
____________________
10𝑥³ - 12𝑥² + 18𝑥 ⇐ multiplication by 2𝑥

- 15𝑥² + 18𝑥 - 27 ⇐ multiplication by -3


______________________
10𝑥³ – 27𝑥² + 36𝑥 - 27 ⇐ multiplication by (2𝑥 - 3)
______________________

Therefore, (5𝑥² – 6𝑥 + 9) x (2𝑥 - 3) = 10𝑥³ – 27𝑥² + 36𝑥 – 27

(ii) Multiply (2𝑥² – 5𝑥 + 4) by (𝑥² + 7𝑥 – 8)

Solution:

By column method

2𝑥² – 5𝑥 + 4

× (𝑥² + 7𝑥 – 8)
___________________________
2𝑥⁴ – 5𝑥³ + 4𝑥² ⇐ multiplication by 𝑥²
253

+ 14𝑥³ - 35𝑥² + 28𝑥 ⇐ multiplication by 7𝑥

- 16𝑥² + 40𝑥 - 32 ⇐ multiplication by -8


___________________________
2𝑥⁴ + 9𝑥³ - 47𝑥² + 68𝑥 - 32 ⇐ multiplication by (𝑥² + 7𝑥 - 8)
___________________________

Therefore, (2𝑥² – 5𝑥 + 4) x (𝑥² + 7𝑥 – 8) = 2𝑥⁴ + 9𝑥³ - 47𝑥² + 68𝑥 – 32

(iii) Multiply (2𝑥³ – 5𝑥² – 𝑥 + 7) by (3 - 2𝑥 + 4𝑥²)

Solution:

Arranging the terms of the given polynomials in descending power of 𝑥 and then
multiplying,

2𝑥³ – 5𝑥² – 𝑥 + 7

× (3 - 2𝑥 + 4𝑥²)
_________________________________
8𝑥⁵ - 20𝑥⁴ – 4𝑥³ + 28𝑥² ⇐ multiplication by 4𝑥²

- 4𝑥⁴ + 10𝑥³ + 2𝑥² – 14𝑥 ⇐ multiplication by -2𝑥

+ 6𝑥³ – 15𝑥² - 3𝑥 + 21 ⇐ multiplication by 3


_________________________________
8𝑥⁵ – 24𝑥⁴ + 12𝑥³ + 15𝑥² – 17𝑥 + 21 ⇐ multiplication by (3 - 2𝑥 + 4𝑥²)
_________________________________

I. Division of a Monomial by a Monomial


In division of algebraic expression if 𝑥 is a variable while m and n are positive
integers such that m > n then (𝑥ᵐ ÷ 𝑥ⁿ) = (𝑥)𝑚−𝑛
254

Quotient of two monomials is a monomial which is equal to the quotient of their


numerical coefficients, multiplied by the quotient of their literal coefficients.

Rule:

Quotient of two monomials = (quotient of their numerical coefficients) x (quotient


of their variables)

Divide:

(i) 8𝒙2𝒚3 by -2𝒙𝒚

Solution:

8𝑥 2 𝑦 3
(i)
−2𝑥𝑦

8
= 𝑥 2 - 1𝑦3 - 1 [Using quotient law 𝒙m ÷ 𝒙n = 𝒙m - n]
−2

= -4𝑥𝑦2

(ii) 35𝒙3𝒚z2 by -7𝒙𝒚z

Solution:

35𝑥 3𝑦z2 by -7𝑥𝑦z

35
= 𝑥 3 - 1𝑦1 - 1z2 - 1 [Using quotient law 𝒙m ÷ 𝒙n = 𝒙m - n]
−7

= -5 𝑥 2𝑦0z1 [y0 = 1]

= -5𝑥 2z

(iii) -15𝒙3𝒚z3 by -5𝒙𝒚z2


255

Solution:

-15𝑥 3𝑦z3 by -5𝑥𝑦z2

−15
= 𝑥 3 - 1𝑦1 - 1𝑧3 - 2. [Using quotient law 𝒙m ÷ 𝒙n = 𝒙m - n].
−5

= 3 𝑥 2y0z1 [y0 = 1].

= 3𝑥 2𝑧

II. Division of a Polynomial by a Monomial


Rule:

For dividing a polynomial by a monomial, divide each term of the polynomial by


the monomial. We divide each term of the polynomial by the monomial and then
simplify.

Divide:

(i) (i) 6𝒙5 + 18𝒙4 - 3𝒙2 by 3𝒙2

Solution:

6𝑥 5 + 18𝑥 4 - 3𝑥 2 by 3𝑥 2

5 4 2 2 6𝑥 5 18𝑥 4 3𝑥 2
= (6𝑥 + 18𝑥 - 3𝑥 ) ÷ 3𝑥 = + -
3𝑥 2 3𝑥 2 3𝑥 2

=2𝑥 3 + 6𝑥 2 - 1

(ii) 20𝒙3𝒚 + 12𝒙2𝒚2 - 10𝒙𝒚 by 2𝒙𝒚

Solution:
256

20𝑥 3𝑦 + 12𝑥 2𝑦2 - 10𝑥𝑦 by 2𝑥𝑦

= (20𝑥 3𝑦 + 12𝑥 2𝑦2 - 10𝑥𝑦) ÷ 2𝑥𝑦

20𝑥 3 y 12𝑥 2 y2 10𝑥𝑦


= + - (Showing Cancellation of numerator and
2𝑥𝑦 2𝑥𝑦 2𝑥𝑦
denominator)

= 10𝑥 2 + 6𝑥𝑦 - 5

III. Division of a Polynomial by a Polynomial

We may proceed according to the steps given below:

(i) Arrange the terms of the dividend and divisor in descending order of their
degrees.

(ii) Divide the first term of the dividend by the first term of the divisor to obtain
the first term of the quotient.

(iii) Multiply all the terms of the divisor by the first term of the quotient and
subtract the result from the dividend.

(iv)Consider the remainder (if any) as a new dividend and proceed as before.

(v) Repeat this process till we obtain a remainder which is either 0 or a polynomial
of degree less than that of the divisor.

Let us understand it through some examples.

Example: Divide 12 – 14a² – 13a by (3 + 2a)

Solution:
257

12 – 14a² – 13a by (3 + 2a)

Write the terms of the polynomial (dividend and divisor both) in decreasing order
of exponents of variables.

So, dividend becomes – 14a² – 13a + 12 and divisor become 2a + 3.

Divide the first term of the dividend by the first term of the divisor which gives
first term of the quotient.

Multiply the divisor by the first term of the quotient and subtract the product
from the dividend which gives the remainder.

Now, this remainder is treated as, new dividend but the divisor remains the
same.

Now, we divide the first term of the new dividend by the first term of the divisor
which gives second term of the quotient.

Now, multiply the divisor by the term of the quotient just obtained and subtract
the product from the dividend.

Thus, we conclude that divisor and quotient are the factors of dividend if the
remainder is zero.

Quotient = -7a + 4

Remainder = 0
258

Verification:

Dividend = divisor × quotient + remainder

= (2a + 3)(-7a + 4) + 0

= 2a(-7a + 4) +3(-7a + 4) + 0

= – 14a² + 8a – 21a + 12 + 0

= – 14a² – 13a + 12

Example: Divide 2𝑥² + 3𝑥 + 1 by (𝑥 + 1).

Solution:

Therefore, quotient = (2𝑥 + 1) and remainder = 0.

Example: Divide 𝑥² + 6𝑥 + 8 by (𝑥 + 4)

Solution:
259

Therefore, Dividend = 𝑥² + 6𝑥 + 8

Divisor = 𝑥 + 4

Quotient = 𝑥 + 2 and

Remainder = 0.

Example: Divide 9𝑥 - 6𝑥² + 𝑥³ - 2 by (𝑥 - 2)

Solution:

Arrange the terms of the dividend and divisor in descending order and then
divide.
260

Therefore, quotient = (𝑥² - 4𝑥 + 1) and remainder = 0.

Example: Divide (29𝑥 - 6𝑥² - 28) by (3𝑥 -4)

Solution:

Arrange the terms of the dividend and divisor in descending order and then
divide.

Therefore, (29𝑥 - 6𝑥² - 28) ÷ (3𝑥 - 4) = (-2𝑥 + 7).

Example: Divide (5𝑥³- 4𝑥² + 3𝑥 + 18) by (3 - 2𝑥 + 𝑥²)

Solution:

The terms of the dividend are in descending order.

Arrange the terms of the divisor in descending order and then divide.
261

Therefore, 5𝑥³- 4𝑥² + 3𝑥 + 18) ÷ (𝑥² - 2𝑥 + 3) = (5𝑥 + 6)

Example: Using division, show that (𝑥 - 1) is a factor of (𝑥³ - 1).

Solution:

(𝑥 - 1) completely divides (𝑥³ - 1).

Hence, (𝑥 - 1) is a factor of (𝑥³- 1).


262

Example: Find the quotient and remainder when (7 + 15𝑥 - 13𝑥² + 5𝑥³) is divided
by (4 - 3𝑥 + 𝑥²).

Solution:

Arrange the terms of dividend and divisor in descending order and then divide.

Therefore, quotient is (5𝑥 + 2) and remainder is (𝑥 - 1).

Example: Divide (10𝑥⁴ + 17𝑥³ - 62𝑥² + 30𝑥 - 3) by (2𝑥² + 7𝑥 - 1).

Solution:

The terms of the dividend and that of the divisor are in descending order. So, we
divide them as;
263

5.3.1 Problem Set 5.3


1. A rectangular swimming pool is twice as long as it is wide. A small concrete
walkway surrounds the pool. The walkway is a constant 2 m wide and has
an area of 196 square m. Find the dimensions of the pool.
2. Evaluate the following expressions:
a) (𝑥 + 1)(𝑥 − 1) = 𝑥 2 − 1
b) (𝑥 + 4)2 = 𝑥 2 + 8𝑥 + 16
c) (2𝑥 2 )(3𝑥 2 + 5𝑥 − 4) = 6𝑥 4 + 10𝑥 3 − 8𝑥 2
1 3 3
d) ( 𝑦 − 50) ( 𝑦 + 10) = 𝑦 2 − 25𝑦 − 500
2 5 10
3. Divide 𝑥 2 − 9𝑥 − 10 𝑏𝑦 𝑥 + 1
𝑥 2 +9𝑥+14
4. Simplify
𝑥+7
5. Find the quotient and remainder when 𝑥 2 + 7𝑥 − 5 is divided by 𝑥 − 4
6. Find the remainder when the polynomial 𝑓(𝑥) = 𝑥 3 + 𝑥 − 5 is divided by
a) (𝑥 − 3)
b) (𝑥 + 2)
c) (2𝑥 − 1)

5.4 Equations and Inequalities


General Objectives: Solve linear equations, quadratic equations and linear
inequalities.
Specific Objectives:
1. Solve simple linear equations in one unknown
2. Solve simultaneous linear equations in two unknowns
3. Solve quadratic equations given in factorized form e.g., (𝑥 − 2)(𝑥 − 3) = 0
4. Solve quadratic equations by factorization by completing the square and by
−𝑏±√𝑏2 −4𝑎𝑐
use of the formula
2𝑎
𝑥−2 𝑥+8
5. Solve equations with fractions e.g., −1−
𝑥−1 𝑥+14
6. Solve simultaneous equations, one linear and one quadratic
7. Solve simple linear inequalities
264

Simple linear equations in one unknown:


Example: Solve 𝑥 – 3 = –5
Since I want to get 𝑥 by itself, I don't want the "–3" that's with the variable. The
opposite of subtraction is addition, so I'll undo the –3 by adding 3 to both sides,
and then adding down:
𝑥 - 3 + 3 = -5 + 3
so 𝑥 = -2
Solve the following pair of simultaneous linear equations:
Equation 1: 2𝑥 +3𝑦 =8
Equation 2: 3𝑥 + 2𝑦 = 7
Step 1: Multiply each equation by a suitable number so that the two equations
have the same leading coefficient. An easy choice is to multiply Equation 1 by 3,
the coefficient of 𝑥 in Equation 2, and multiply Equation 2 by 2, the 𝑥 coefficient
in Equation 1:

3 ×(Eqn1)=> 3× (2𝑥 +3𝑦 =8)--> 6𝑥 +9𝑦 =24

2 × (Eqn 2)=> 2 × (3𝑥 +2𝑦 =7)--> 6𝑥 +4𝑦 =14


Both equations now have the same leading coefficient = 6

Step 2: Subtract the second equation from the first.

(6𝑥 +9𝑦 =24)

(-6𝑥 -4𝑦 =-14

5𝑦 = 10
Step 3: Solve this new equation for 𝑦.
10
𝑦= =2
5
265

Step 4: Substitute 𝑦 = 2 into either Equation 1 or Equation 2 above and solve


for 𝑥. We'll use Equation 1.

2𝑥 + 3(2) = 8
2𝑥 + 6 = 8 Subtract 6 from both sides
2𝑥 = 2 Divide both sides by 2

𝑥=1
Solution: 𝑥 = 1, 𝑦 = 2 or (1,2).

Quadratic equation:
A quadratic equation in 𝑥 is an equation that can be written in the standard
form , where a, b, and c are real numbers, but a≠0.
Solve (𝒙 – 3)(𝒙 – 4) = 0.
𝑥 – 3 = 0 or 𝑥 – 4 = 0
𝑥 = 3 or 𝑥 = 4
𝑥 =( 3, 4 )
The Quadratic Formula:
For a𝒙2 + b𝒙 + c = 0, the value of 𝒙 is given by:
−𝑏−√(𝑏2 −4𝑎𝑐) −𝑏+√(𝑏2 −4𝑎𝑐)
𝑋= 𝑋=
2𝑎 2𝑎

Q: Use the Quadratic Formula to solve 𝒙2 – 4𝒙 – 8 = 0


Looking at the coefficients, I see that a = 1, b = –4, and c = –8. I'll plug them into
the Formula, and simplify. I should get the same answer as before:

−𝑏± √(𝑏2 −4𝑎𝑐)


𝑋=
2𝑎

4± √((−4)2 −4×1×−8)
𝑋=
2×1
266

After Solving the above equation we get:


𝑥 = 2 ± 2 √3

In quadratic equations we will learn about :

● Solving a quadratic equation (factorization method)

● Roots of the quadratic equation.

In polynomials, we studied that a polynomial of degree 1 is called a linear


polynomial.

Example: 𝑥 - 5, 7𝑥, 3 - 2𝑥 are linear polynomials which may be monomials or


binomials.

A polynomial of degree 2 (two) is called a quadratic polynomial.

Example: 3𝑥², 𝑥² + 7 , 𝑥² – 3𝑥 + 4 are quadratic polynomials which may be


monomials, binomials or trinomials.

What is known as quadratic equation?

When these quadratic polynomials are equated to zero, equation is formed and is
known as a quadratic equation.

The standard form of quadratic equation is a𝑥² + b𝑥 + c = 0. Here a, b, c are real


numbers and a ≠ 0. The power of 𝑥 in the equation must be a non-negative
integer.

Examples of quadratic equation

(i) 3𝑥² - 6𝑥 + 1 = 0 is a quadratic equation.


267

1
(ii) 𝑥 + = 5 is a quadratic equation.
𝑥

1
By solving it, we get 𝑥 × 𝑥 + × 𝑥 = 5 × 𝑥
𝑥
⇒ 𝑥² + 1 = 5𝑥
⇒ 𝑥² - 5𝑥 + 1 = 0

(iii) √2𝑥² - 𝑥 - 7 = 0 is a quadratic equation.

(iv) 3𝑥² - √𝑥 + 1 = 0 is not a quadratic equation, since the power of 𝑥 must be a


positive integer.

1
(v) 𝑥² - + 7 = 0 is not a quadratic equation, since by solving it becomes an
𝑥
equation of degree 3.

(vi) 𝑥² - 4 = 0 is a quadratic equation.

(vii) 𝑥² = 0 is a quadratic equation.

Solving a quadratic equation and finding the roots of quadratic equation

● Write the quadratic equation in the standard form, i.e.,

a𝑥² + b𝑥 + c = 0.

● Factorize the quadratic equation.

● Express it as the product of two linear factors, say (p𝑥 + q) and (r𝑥 + s), where p,
q, r, s are real numbers and p, r are not equal to zero.

Then, a𝑥² + b𝑥 + c = 0

(p𝑥 + q) (r𝑥 + s) = 0

● Put each of the linear factors equal to zero


268

i.e., p𝑥 + q = 0 and r𝑥 + s = 0

⇒ p𝑥 = - q ⇒ r𝑥 = - s
𝑞 𝑆
⇒x=− ⇒x=−
𝑝 𝑟

● Thus, the two values of 𝑥 are called the roots of the quadratic equation.

𝑞 𝑆
● Therefore, the solution set = {− , − }
𝑝 𝑟

How to solve quadratic equations?

Worked-out problems on solving quadratic equation will help students


understand the detailed explanation showing step-by-step quadratic equation
solution.

Example: Solve: 𝑥² + 6𝑥 + 5 = 0

Solution:

𝑥² + 6𝑥 + 5 = 0

⇒ 𝑥² + 5𝑥 + 𝑥 + 5 = 0

⇒ 𝑥(𝑥 + 5) + 1(𝑥 + 5) = 0

⇒ (𝑥 + 1) (𝑥 + 5) = 0

⇒ 𝑥 + 1 = 0 and 𝑥 + 5 = 0

⇒ 𝑥 = -1 and 𝑥 = -5

Therefore, solution set = {-1, -5}

Example: Solve: 8𝑥² = 21 + 22𝑥

Solution:
269

8𝑥² = 21 + 22𝑥

⇒ 8𝑥² - 21 - 22𝑥 = 0

⇒ 8𝑥² - 22𝑥 - 21 = 0

⇒ 8𝑥² - 28𝑥 + 6𝑥 - 21 = 0

⇒ 4𝑥 (2𝑥 - 7) + 3(2𝑥 - 7) = 0

⇒ (4𝑥 + 3) (2𝑥 - 7) = 0

⇒ 4𝑥 + 3 = 0 and 2𝑥 - 7 = 0

⇒ 4𝑥 = -3 and 2𝑥 = 7
3 7
⇒ 𝑥 =− and 𝑥 =
4 2
3 7
Therefore, solution set = {− , }
4 2

1 1 11
Example: − =
𝑥+4 𝑥−7 30

Solution:

1 1 11
- =
𝑥+4 𝑥−7 30

(𝑥−7)−(𝑥+4) 11
⇒ =
(𝑥+4) (𝑥−7) 30

(𝑥−7−𝑥−4) 11
⇒ =
(𝑥²−3𝑥−28) 30

11 11
⇒- =
(𝑥²−3𝑥−28) 30

11 11
⇒− =
(𝑥²−3𝑥−28) 30
270

⇒ -30 = 𝑥² - 3𝑥 - 28

⇒ 𝑥² - 3𝑥 + 2 = 0

⇒ 𝑥² - 2𝑥 - 𝑥 + 2 = 0

⇒ 𝑥(𝑥 - 2) - 1(𝑥 - 2) = 0

⇒ (𝑥 - 1) (𝑥 - 2) = 0

⇒ 𝑥 - 1 = 0 and 𝑥 - 2 = 0

⇒ 𝑥 = 1 and 𝑥 = 2

Therefore, Solution set = {1, 2}

Let’s discuss different cases of discriminant to understand the nature of the


roots of a quadratic equation.

We know that α and β are the roots of the general form of the quadratic equation
𝑎𝑥 2 + 𝑏𝑥 + 𝑐 = 0 (a ≠ 0) .................... (i) then we get

−𝑏−√(𝑏2 −4𝑎𝑐) −𝑏+√(𝑏2 −4𝑎𝑐)


α = −b− and β = −b−
2𝑎 2𝑎

Here a, b and c are real and rational numbers.

Then, the nature of the roots α and β of equation :


𝑎𝑥 2 + 𝑏𝑥 + 𝑐 = 0 depends on the quantity or expression of Terms inside the
Square root.
Thus the expression √(𝑏 2 − 4𝑎𝑐) is called the discriminant of
the quadratic equation 𝑎𝑥 2 + 𝑏𝑥 + 𝑐 = 0.

Generally we denote discriminant of the quadratic equation by ‘∆ ‘ or ‘D’.

Therefore,

Discriminant ∆ = √(𝑏2 − 4𝑎𝑐)


Depending on the discriminant we shall discuss the following cases about the
nature of roots α and β of the quadratic equation 𝑎𝑥 2 + 𝑏𝑥 + 𝑐 = 0.
271

When a, b and c are real numbers, a ≠ 0

Case I: √(𝑏2 − 4𝑎𝑐)> 0


When a, b and c are real numbers, a ≠ 0 and discriminant is positive (i.e.,
√(𝑏 2 − 4𝑎𝑐) > 0), then the roots α and β of the quadratic equation 𝑎𝑥 2 + 𝑏𝑥 +
𝑐 = 0. are real and unequal.

Case II: √(𝑏2 − 4𝑎𝑐) = 0

When a, b and c are real numbers, a ≠ 0 and discriminant is zero (i.e., √(𝑏2 −
4𝑎𝑐) = 0), then the roots α and β of the quadratic equation 𝑎𝑥 2 + 𝑏𝑥 + 𝑐 =
0. are real and equal.

Case III: √(𝑏 2 − 4𝑎𝑐) < 0

When a, b and c are real numbers, a ≠ 0 and discriminant is negative (i.e., b² - 4ac
< 0), then the roots α and β of the quadratic equation 𝑎𝑥 2 + 𝑏𝑥 + 𝑐 = 0. are
unequal and imaginary. Here the roots α and β are a pair of the complex
conjugates.

Case IV: √(𝑏 2 − 4𝑎𝑐)> 0 and perfect square

When a, b and c are real numbers, a ≠ 0 and discriminant is positive and perfect
square, then the roots α and β of the quadratic equation 𝑎𝑥 2 + 𝑏𝑥 + 𝑐 = 0. are
real, rational unequal.

Case V: √(𝑏2 − 4𝑎𝑐) > 0 and not perfect square

When a, b and c are real numbers, a ≠ 0 and discriminant is positive but not a
perfect square then the roots of the quadratic equation 𝑎𝑥 2 + 𝑏𝑥 + 𝑐 = 0. are
real, irrational and unequal.

Here the roots α and β form a pair of irrational conjugates.

Case VI: √(𝑏 2 − 4𝑎𝑐)is perfect square and a or b is irrational

When a, b and c are real numbers, a ≠ 0 and the discriminant is a perfect square
but any one of a or b is irrational then the roots of the quadratic equation𝑎𝑥 2 +
𝑏𝑥 + 𝑐 = 0.are irrational.
272

Notes:

(i) From Case I and Case II we conclude that the roots of the quadratic equation
a𝑥² + b𝑥 + c = 0 are real when 𝑏 2 − 4𝑎𝑐 ≥ 0 or 𝑏 2 − 4𝑎𝑐 ≮ 0.
(ii) From Case I, Case IV and Case V we conclude that the quadratic equation with
real coefficient cannot have one real and one imaginary roots; either both the
roots are real when 𝑏 2 − 4𝑎𝑐 > 0 or both the roots are imaginary when 𝑏 2 − 4𝑎𝑐
< 0.
(iii) From Case IV and Case V we conclude that the quadratic equation with
rational coefficient cannot have only one rational and only one irrational roots;
either both the roots are rational when 𝑏 2 − 4𝑎𝑐 is a perfect square or both the
roots are irrational 𝑏 2 − 4𝑎𝑐 is not a perfect square.

Various types of Solved examples on nature of the roots of a quadratic equation:

Example: Find the nature of the roots of the equation 3𝑥 2 - 10𝑥 + 3 = 0 without
actually solving them.

Solution:

Here the coefficients are rational.

The discriminant D of the given equation is

D = 𝑏 2 − 4𝑎𝑐
= (−10)2 - 4 ∙ 3 ∙ 3

= 100 - 36

= 64 > 0

Clearly, the discriminant of the given quadratic equation is positive and a perfect
square.

Therefore, the roots of the given quadratic equation are real, rational and
unequal.
273

Example: Discuss the nature of the roots of the quadratic equation


2𝑥 2 - 8𝑥 + 3 = 0.

Solution:

Here the coefficients are rational.

The discriminant D of the given equation is

D = 𝑏 2 − 4𝑎𝑐
= (−8)2 - 4 ∙ 2 ∙ 3

= 64 - 24

= 40 > 0

Clearly, the discriminant of the given quadratic equation is positive but not a
perfect square.

Therefore, the roots of the given quadratic equation are real, irrational and
unequal.

Example: Find the nature of the roots of the equation 𝑥 2 - 18𝑥 + 81 = 0 without
actually solving them.

Solution:

Here the coefficients are rational.

The discriminant D of the given equation is

D = 𝑏 2 − 4𝑎𝑐
= (−18)2 - 4 ∙ 1 ∙ 81

= 324 - 324

=0
274

Clearly, the discriminant of the given quadratic equation is zero and the
coefficients of 𝑥 ² and 𝑥 are rational.

Therefore, the roots of the given quadratic equation are real, rational and equal.

To Solve an Equation With fractions, we transform it into an equation without


fractions , which we know how to solve. The technique is called clearing of
fractions.
Example: Solve for 𝒙:
𝒙 𝒙−𝟐
− = 𝟔
𝟑 𝟓
Multiply both sides of the equation of every term by the LCM of
denominators. Each denominator will then divide into its multiple. We will then
have an equation without fractions.
The LCM of 3 and 5 is 15. Therefore, multiply every term on both sides by 15:
𝒙 𝒙−𝟐
15 × − 𝟏𝟓 × = 𝟏𝟓 × 𝟔
𝟑 𝟓
5𝒙 + 3(𝒙-2)=90
8𝒙-6=90
8𝒙=90+6
𝟗𝟔
𝒙=
𝟖
𝒙=12

The simultaneous solution of a linear equation and a quadratic equation is given


by the point or points of intersection of the line and parabola representing the
equations.
To find the solution, eliminate 𝑦 (or 𝑥) by substitution; and solve the quadratic
equation thus formed in 𝑥 (or 𝑦). Then find the corresponding value(s) of 𝑦 (or 𝑥).
275

Example: Solve the following simultaneous equations:


𝑌 = 𝑋+2
𝑌 = 𝑋2

Solution:
𝑌 = 𝑋+2
𝑌 = 𝑋2
Substituting 𝑌= 𝑋 2 in the other equation we get,
𝑋2 = 𝑋 + 2
Factorize the above equation as
𝑋2 − 𝑋 + 2 = 0
𝑋=2 or 𝑋=-1
When 𝑋 = 2, 𝑦= 4 and when 𝑋= -1, 𝑌=1
So the Solution set are ((2,4),(-1,1))

Example: Solve the inequality: −2(𝑥+3)<10 (Where 𝑥 is a variable)


= −2(𝑥+3)<10
= −2𝑥−6<10
= −2𝑥−6<10
= −2𝑥−6+6<10+6
= −2𝑥−6+6<10+6
= −2𝑥<16
= −2𝑥<16
= −2𝑥−2>16−2
= −2𝑥−2>16−2
= 𝑥 > −7
276

5.4.1 Problem Set 5.4


1. Solve the simultaneous equation 2𝑥 + 3𝑦 = 23 𝑎𝑛𝑑 4𝑥 − 2𝑦 = 6
2. If three times of a number is 6 more than twice the number. What is the
value of the number?
3. Smith bought two black pencils and three red pencils for $ 23. After a week,
he bought three black pencils and two red pencils for $16. If price of pencils
remained the same for both transactions, how much she paid for one black
pencil?
4. One positive number is 2/3 of the other and their product is 24. What is the
sum of the two?
5. Factor the quadratic expression completely 6𝑥 2 − 13𝑥 + 6

5.5 Sequences
General Objectives: Find patterns in sequences
Specific Objectives:
1. Recognize patterns in sequences
2. Find the nth term of a linear sequence.
3. Distinguish between arithmetic and geometric sequences, use the formulae
for the nth term and the sum of the first n terms to solve problems involving
arithmetic and geometric progressions.
A sequence, in mathematics, is a string of objects, like numbers, that follow a
particular pattern. The individual elements in a sequence are called terms. Some
of the simplest sequences can be found in multiplication tables:

Q: 3, 6, 9, 12, 15, 18, 21, …


Pattern: “add 3 to the previous number to get the next number”

Q: 0, 12, 24, 36, 48, 60, 72, …


Pattern: “add 12 to the previous number to get the next number”
Finding the nth term
Sometimes, rather than finding the next number in a linear sequence, you want to
find the 46th number, or 140th number to say. Writing out 4 or 110 numbers
takes a long time, so you can use a general rule.
277

Example: What is the nth term of this sequence?


5, 10, 15……
Common difference: 5
10-5=5
15-10=5
1st = 5 (5 × 1); 2nd = 10 (5 × 2); 3rd = 15 (5 × 3)
So the nth term is 5 × n or 5n.

The two simplest sequences to work with are arithmetic and geometric
sequences.
An arithmetic sequence goes from one term to the next by always adding (or
subtracting) the same value.
For instance, 2, 5, 8, 11, 14,... is arithmetic, because each step adds three; and 7,
3, –1, –5,... is arithmetic, because each step subtracts 4.

The number added (or subtracted) at each stage of an arithmetic sequence is


called the "common difference" d, because if you subtract (that is, if you find the
difference of) successive terms, you'll always get this common value.

Following this pattern, the n-th term an will have the form: an = a + (n – 1)d.
A geometric sequence goes from one term to the next by always multiplying (or
dividing) by the same value. So 1, 2, 4, 8, 16,... is geometric, because each step
multiplies by 2; and 81, 27, 9, 3, 1,1/3 ,... is geometric, because each step divides
by 3.
The number multiplied (or divided) at each stage of a geometric sequence is
called the "common ratio" r, because if you divide (that is, if you find the ratio of)
successive terms, you'll always get this common value.

Following this pattern, the n-th term an will have the form: an= ar(n-2)
278

Examples on Arithmetic Progression:

1. -2, 1, 4, 7, 10 ……………. is an A.P. whose first term is formed by adding a


constant quantity with the preceding term.

Definition of Arithmetic Progression: A sequence of numbers is known as an


arithmetic progression (A.P.) if the difference of the term and the preceding term
is always same or constant.

The constant quantity stated in the above definition is called the common
difference of the progression. The constant difference, generally denoted by d is
called the common difference.

an+1 − an = constant (=d) for all n∈ N

From the definition, it is clear that an arithmetic progression is a sequence of


numbers in which the difference between any two consecutive terms is constant.

….. and common difference is 1 - (-2) = 1 + 2 = 3

2. The sequence {3, 7, 11, 15, 19, 23, 27, …………………} is an Arithmetic Progression
whose common difference is 4, since

Second term (7) = First term (3) + 4

Third term (11) = Second term (7) + 4

Fourth term (15) = Third term (11) + 4

Fifth term (19) = Fourth term (15) + 4 etc.

3. The sequence {58, 43, 28, 13, -2, -17, -32, …………………} is an Arithmetic
Progression whose common difference is -15, since

Second term (43) = First term (58) + (-15)

Third term (28) = Second term (43) + (-15)


279

Fourth term (13) = Third term (28) + (-15)

Fifth term (-2) = Fourth term (13) + (-15) etc.

4. The sequence {11, 23, 35, 47, 59, 71, 83, …………………} is an Arithmetic
Progression whose common difference is 12, since

Second term (23) = First term (11) + 12

Third term (35) = Second term (23) + 12

Fourth term (47) = Third term (35) + 12

Fifth term (59) = Fourth term (47) + 12 etc.

Note: To obtain the common difference of a given arithmetic progression we


required to subtract any term from a term which follows it. That is,

Common Difference = Any term - Its preceding term

Solved examples on general form of an Arithmetic Progress

Example: Show that the sequence 3, 5, 7, 9, 11, ......... is an Arithmetic


Progression. Find its 15th term and the general term.

Solution:

First term of the given sequence = 3

Second term of the given sequence = 5

Third term of the given sequence = 7

Fourth term of the given sequence = 9

Fifth term of the given sequence = 11

Now, Second term - First term = 5 - 3 = 2


280

Third term - Second term = 7 - 5 = 2

Fourth term - Third term = 9 - 7 = 2

Therefore, the given sequence is an Arithmetic Progression with the common


difference 2.

We know that the nth term of an Arithmetic Progression, whose first term is a
and common difference d is tn = a + (n - 1) × d
Therefore, 15th term of the Arithmetic Progress = t15 = 3 + (15 - 1) × 2 = 3 + 14 × 2
= 3 + 28 = 31.
General term = nth term = an= a + (n - 1)d = 3 + (n - 1) × 2 = 3 + 2n - 2 = 2n + 1

Example: Which term of the sequence 6, 11, 16, 21, 26, ....... is 126?

Solution:

First term of the given sequence = 6

Second term of the given sequence = 11

Third term of the given sequence = 16

Fourth term of the given sequence = 21

Fifth term of the given sequence = 26

Now, Second term - First term = 11 - 6 = 5

Third term - Second term = 16 - 11 = 5

Fourth term - Third term = 21 - 16 = 5

Therefore, the given sequence is an Arithmetic Progression with the common


difference 5.

Let 126 be the nth term of the given sequence. Then,

An = 126

⇒ a + (n - 1)d = 126
281

⇒ 6 + (n - 1) × 5 = 126

⇒ 6 + 5n - 5 = 126

⇒ 5n + 1 = 126

⇒ 5n = 126 - 1

⇒ 5n = 125

⇒ n = 25

Hence, 25th term of the given sequence is 126.

Example: Find the seventeenth term of the Arithmetic Progression {31, 25, 19, 13,
..................... }.

Solution:

The given Arithmetic Progression is {31, 25, 19, 13, ..................... }.

First term of the given sequence = 31

Second term of the given sequence = 25

Third term of the given sequence = 19

Fourth term of the given sequence = 13

Now, Second term - First term = 25 - 31 = -6

Third term - Second term = 19 - 25 = -6

Fourth term - Third term = 13 - 19 = -6

Therefore, common difference of the given sequence = -6.

Thus, the 17th term of the given Arithmetic Progress is:

a + (n -1)d = 31 + (17 - 1) × (-6)

= 31 + 16 × (-6) = 31 - 96 = -65
282

Note: Any term of an Arithmetic Progression can be obtained if its first term and
common difference are given.

We will learn how to find the sum of first n terms of an Arithmetic Progression.

Prove that the sum Sn of n terms of an Arithmetic Progression (A.P.) whose first
term ‘a’ and common difference ‘d’ is
n
S = [2a + (n - 1)d]
2
n
Or, S = [a + l], where l = last term = a + (n - 1)d
2

Solved examples to find the sum of first n terms of an Arithmetic Progression:

Example: Find the sum of the following Arithmetic series:

1 + 8 + 15 + 22 + 29 + 36 + ………………… to 17 terms

Solution:

First term of the given arithmetic series = 1

Second term of the given arithmetic series = 8

Third term of the given arithmetic series = 15

Fourth term of the given arithmetic series = 22

Fifth term of the given arithmetic series = 29

Now, Second term - First term = 8 - 1 = 7

Third term - Second term = 15 - 8 = 7

Fourth term - Third term = 22 - 15 = 7

Therefore, common difference of the given arithmetic series is 7.

The number of terms of the given A. P. series (n) = 17

We know that the sum of first n terms of the Arithmetic Progress, whose first
term = a and common difference = d is
283

n
S= [2a + (n - 1)d]
2
Therefore, the required sum of first 20 terms of the series = 17/2[2 ∙ 1 + (17 - 1) ∙
7]
17
= [2 + 16 ∙ 7]
2

17
= [2 + 112]
2

17
= × 114
2

= 17 × 57

= 969

Example: Find the sum of the series: 7 + 15 + 23 + 31 + 39 + 47 + ……….. + 255

Solution:

First term of the given arithmetic series = 7

Second term of the given arithmetic series = 15

Third term of the given arithmetic series = 23

Fourth term of the given arithmetic series = 31

Fifth term of the given arithmetic series = 39

Now, Second term - First term = 15 - 7 = 8

Third term - Second term = 23 - 15 = 8

Fourth term - Third term = 31 - 23 = 8

Therefore, the given sequence is an arithmetic series with the common difference
8.

Let there be n terms in the given arithmetic series. Then


284

An = 255

⇒ a + (n - 1)d = 255

⇒ 7 + (n - 1) × 8 = 255

⇒ 7 + 8n - 8 = 255

⇒ 8n - 1 = 255

⇒ 8n = 256

⇒ n = 32
32
Therefore, the required sum of the series = [2 ∙ 7 + (32 - 1) ∙ 8]
2

= 16 [14 + 31 ∙ 8]

= 16 [14 + 248]

= 16 × 262

= 4192

Property I: If a constant quantity is added to or subtracted from each term of an


Arithmetic Progression (A. P.), then the resulting terms of the sequence are also in
A. P. with same common difference (C.D.).

Proof:

Let {a1, a2, a3, a4, ..............} ........... (i) be an Arithmetic Progression with
common difference d.

Again, let k be a fixed constant quantity.

Now k is added to each term of the above A.P. (i)

Then the resulting sequence is a1 + k, a2 + k, a3 + k, a4 + k ..................


Let bn = an + k, n = 1, 2, 3, 4, ............
Then the new sequence is b1, b2, b3, b4, ...............
We have bn+1 - bn = (an+1 + k) - (an + k) = an+1 - an = d for all n ∈ N, [Since, <an>
is a sequence with common difference d].
285

Therefore, the new sequence we get after adding a constant quantity k to each
term of the A.P. is also an Arithmetic Progression with common difference d.

To get a clear concept of property (I) let us follow the below explanation.

Let’s assume ‘a’ be the first term and ‘d’ be the common difference of an
Arithmetic Progression. Then, the Arithmetic Progression is {a, a + d, a + 2d, a +
3d, a + 4d, ..........}

1. By adding a constant quantity:

If a constant quantity k is added to each term of the Arithmetic Progression {a, a


+ d, a + 2d, a + 3d, a + 4d, ............} we get,

{a + k, a + d + k, a + 2d + k, a + 3d + k, a + 4d + k, ...........} ............. (i)

First term of the above sequence (i) is (a + k).

Common difference of the above sequence (i) is (a + d + k) - (a + k) = d

Therefore, the terms of the above sequence (i) form an Arithmetic Progression.

Hence, if a constant quantity be added to each term of an Arithmetic Progression,


the resulting terms are also in Arithmetic Progression with the same common
difference.

2. By subtracting a constant quantity:

If a constant quantity k is subtracted from each term of the Arithmetic


Progression {a, a + d, a + 2d, a + 3d, a + 4d, ...........} we get,

{a - k, a + d - k, a + 2d - k, a + 3d - k, a + 4d - k, ...........} ................. (ii)

First term of the above sequence (ii) is (a - k).

Common difference of the above sequence (ii) is (a + d - k) - (a - k) = d

Therefore, the terms of the above sequence (ii) form an Arithmetic Progression.
286

Hence, if a constant quantity is subtracted from each term of an Arithmetic


Progression, the resulting terms are also in Arithmetic Progression with the same
common difference.

Property II: If each term of an Arithmetic Progression is multiplied or divided by a


non-zero constant quantity, then the resulting sequence form an Arithmetic
Progression.

Proof:

Let’s assume {a1, a2, a3, a4, ..............} ........... (i) be an Arithmetic Progression
with common difference d.

Again, let k be a fixed non-zero constant quantity.

Let us obtain, b1, b2, b3, b4, ............. be the sequence, after multiplying each
term of the given A.P. (i) by k.
b1 = a1k
b2 = a2k
b3 = a3k
b4 = a4k

..............

..............

bn = ank

............

...........

Now, bn+1 - bn = an+1k - ank = (an+1 – an)k = dk for all n ∈ N, [Since, <an> is a
sequence with common difference d]

Therefore, the new sequence we get after multiplying a non-zero constant


quantity k to each term of the A. P. is also an Arithmetic Progression with
common difference dk.
287

To get a clear concept of property II let us follow the below explanation.

Let us assume ‘a’ be the first term and ‘d’ be the common difference of an
Arithmetic Progression. Then, the Arithmetic Progression is {a, a + d, a + 2d, a +
3d, a + 4d, ............}

1. On multiplying a constant quantity:

If a non-zero constant quantity k (≠ 0) is multiplied by each term of


the Arithmetic Progression {a, a + d, a + 2d, a + 3d, a + 4d, ................} we get,

{ak, ak + dk, ak + 2dk, ak + 3dk, .............} ............... (iii)

First term of the above sequence (iii) is ak.

Common difference of the above sequence (iii) is (ak + dk) - ak = dk

Therefore, the terms of the above sequence (iii) form an Arithmetic Progression.

Hence, if a non-zero constant quantity be multiplied by each term of an


Arithmetic Progression, the resulting terms are also in Arithmetic Progression.

2. On dividing a constant quantity:

If a non-zero constant quantity k (≠ 0) is divided by each term of the Arithmetic


Progression {a, a + d, a + 2d, a + 3d, a + 4d, ...........} we get,

{ak, ak + dk, ak + 2dk, ak + 3dk, ............} ................. (iv)


First term of the above sequence (iv) is akak.
Common difference of the above sequence (iv) is (ak + dk) - ak = dk

Therefore, the terms of the above sequence (iv) form an Arithmetic Progression.

Hence, if a non-zero constant quantity be divided by each term of an Arithmetic


Progression, the resulting terms are also in Arithmetic Progression.
288

Property III:

In an Arithmetic Progression of finite number of terms the sum of any two terms
equidistant from the beginning and the end is equal to the sum of the first and
last terms.

Proof:

Let us assume ‘a’ be the first term, ‘d’ be the common difference, ‘l’ be the last
term and ‘n’ be the number of terms of an A.P. (n is finite).

The second term from the end = l - d

The third term from the end = l - 2d

The fourth term from the end = l - 3d

The rth term from the end = l - (r - 1)d

Again, the rth term from the beginning = a + (r - 1)d

Therefore, the sum of the rth terms from the beginning and the end is:

= a + (r - 1)d + l - (r - 1)d

= a + rd - d + l - rd + d

=a+l

Hence, the sum of two terms equidistant from the beginning and the end is
always the same or equal to the sum of the first and last terms.

Property IV:

Three numbers 𝑥, 𝑦, and z are in Arithmetic Progression if and only if 2𝑦 = 𝑥 + z.

Proof:

Let us assume that, 𝑥, 𝑦, z be in Arithmetic Progression.

Now, common difference = 𝑦 − 𝑥 and again, common difference = z - y


289

⇒𝑦-𝑥=z-𝑦

⇒2𝑦 = 𝑥 + z

Conversely, let 𝑥, 𝑦, z be three numbers such that 2𝑦 = 𝑥 + z. Then we prove can


that 𝑥, 𝑦, z are in Arithmetic Progression.

We have, 2𝑦 = 𝑥 + z

⇒𝑦–x=z–𝑦

⇒ 𝑥, 𝑦, z are in Arithmetic Progression.

Property V:

A sequence is an Arithmetic Progression if and only if its nth term is a linear


expression in n i.e., an = An + B , where A, B are two constant quantities.

In this case the coefficient of n in an is the common difference (C.D.) of the


Arithmetic Progression.

Property VI:

A sequence is an Arithmetic Progression if and only if the sum of its first n terms is
of the form An2 + Bn, where A, B are two constant quantities that are
independent of n.
In this case the common difference is 2A that is 2 times the coefficient of n22.

Property VII:

A sequence is an Arithmetic Progression if the terms are selected at a regular


interval from an Arithmetic Progression.

Property VIII:

If 𝑥, 𝑦, and z are three consecutive terms of an Arithmetic Progression then 2y =


𝑥+z
290

Geometric Progression:

Let’s discuss about the Geometric Progression along with examples.

A sequence of numbers is said to be Geometric Progression if the ratio of any


term and its preceding term is always a constant quantity.

Definition of Geometric Progression:

A sequence of non-zero number is said to be in Geometric Progression


(abbreviated as G.P.) if each term, after the first, is obtained by multiplying the
preceding term by a constant quantity (positive or negative).

The constant ratio is said to be the common ratio of the Geometric Progression
and is denoted by dividing any term by that which immediately precedes it.

In other words, the sequence {a1, a2, a3, a4, ..................., an, ................. } is said
to be in Geometric Progression, if an+1/an = constant for all n ϵ N i.e., for all
integral values of a, the ratio an+1/an is constant.

Examples on Geometric Progression

1. The sequence of numbers {4, 12, 36, 108, 324, ........... } forms a Geometric
Progression whose common ratio is 3, because,

Second term (12) = 3 × First term (4),

Third term (36) = 3 × Second term (12),

Fourth term (108) = 3 × Third term (36),

Fifth term (324) = 3 × Fourth term (108) and so on.

In other words,

Second term(12)÷First term(4) = Third term(36)÷Second term(12) = Fourth


term(108)÷Third term(36) =Fifth term(324)÷Fourth term(108) = 3 (a constant).
291

Geometric Series:

If a1, a2, a3, a4, ..............., an, .......... is a Geometric Progression, then the
expression a1 + a2 + a3 + ......... + an + .................... is called a geometric series.

Notes:

(i) The geometric series is finite as the corresponding Geometric Progression


consists of finite number of terms.

(ii) The geometric series is infinite as the corresponding Geometric Progression


consists of infinite number of terms.

SUM OF GEOMETRIC SERIES:

The Sum of n terms of a geometric Progression is given by


(𝑟 𝑛 −1)
Sn = a
(𝑟−1)

Note: The above formula does not hold for r=1 . For r=1 , the sum of n terms of
the Geometric Progression is equal to na.

Question. Find the sum of the geometric series:

4 - 12 + 36 - 108 + .............. to 10 terms

Solution:

The first term of the given Geometric Progression = a = 4 and its common ratio = r
12
=- = -3
4
Therefore, the sum of the first 10 terms of the geometric series is:
(𝑟 𝑛 −1)
a
(𝑟−1)
((−3)10 −1)
Putting the Values, 4 gives -59048 as the answer.
(−3−1)

We will discuss here about some of the important relations between Arithmetic
Mean and Geometric Mean.
292

The following properties are:

Property I: The Arithmetic Mean of two positive numbers can never be less than
their Geometric Mean.

Proof:

Let A and G be the Arithmetic Mean and Geometric Mean respectively of two
positive numbers m and n.
n
Then, we have A = m + and G = ±√mn
2

Since, m and n are positive numbers, hence it is evident that A > G when G = -
√mn. Therefore, we are to show A ≥ G when G = √mn.
n n
We have, A - G = m + - √mn = m + n - 2√m
2 2

A - G = ½[(√m - √n)²] ≥ 0

Therefore, A - G ≥ 0 or, A ≥ G.

Hence, the Arithmetic Mean of two positive numbers can never be less than their
Geometric Mean. (Proved).

Property II: if A is the Arithmetic Mean and G be the Geometric Mean between
two positive numbers m and n, then the quadratic equation whose roots are m, n
is 𝑥² - 2A𝑥 + G²= 0

Proof:

Since, A and G are the Arithmetic Mean and Geometric Mean respectively of two
positive numbers m and n then, we have
n
A = m + and G = √mn.
2

The equation having m, n as its roots is


293

𝑥² - 𝑥(m + n) + nm = 0
n
⇒ 𝑥²- 2A𝑥 + G²= 0, [Since, A = m + and G = √nm]
2

Property III: If A is the Arithmetic Mean and G be the Geometric Mean between
two positive numbers, then the numbers are A ± √A²- G²

Proof:

Since, A and G are the Arithmetic Mean and Geometric Mean respectively then,
the equation having its roots as the given numbers is:

𝑥²- 2A𝑥 + G²= 0


4AG²
⇒ 𝑥 = 2A ± √4A²-
2

⇒ 𝑥 = A ± √A²- G²

Property IV: If the Arithmetic Mean of two numbers 𝑥 and 𝑦 is to their Geometric
Mean as p : q, then, 𝑥 : 𝑦 = (p + √(p²- q²) : (p - √(p² - q²)

Solved examples on the properties of Arithmetic and Geometric Means


between two given quantities:

1. The Arithmetic and Geometric Means of two positive numbers are 15 and 9
respectively. Find the numbers:

Solution:

Let the two positive numbers be 𝑥 and 𝑦. Then according to the problem,
y
𝑥+ = 15
2

or, 𝑥 + 𝑦 = 30 .................. (i)

and √𝑥𝑦 = 9
294

or 𝑥𝑦 = 81

Now, (𝑥 − 𝑦)²= (𝑥 + 𝑦)² - 4𝑥𝑦 = (30)²- (4 ×81) = 576 = (24)²

Therefore, 𝑥 - 𝑦 = ± 24 .................. (ii)

Solving (ii) and (iii), we get,

2𝑥 = 54 or 2𝑥 = 6

𝑥 = 27 or 𝑥 = 3

When 𝑥 = 27 then 𝑦 = 30 - 𝑥 = 30 - 27 = 3

and when 𝑥 = 27 then 𝑦 = 30 - 𝑥 = 30 - 3 = 27

Therefore, the required numbers are 27 and 3.

2. Find two positive numbers whose Arithmetic Mean if increased by 2 then


the Geometric Mean and their difference is 12.

Solution:

Let the two numbers be m and n. Then,

m - n = 12 ........................ (i)

It is given that AM - GM = 2
n
⇒ m + - √mn = 2
2

⇒ m + n - √mn = 4

⇒ (√m - √n)²= 4

⇒ √m - √n = ±2 ........................ (ii)

Now, m - n = 12

⇒ (√m + √n)(√m - √n) = 12


295

⇒ (√m + √n)(±2) = 12 ........................ (iii)

⇒ √m + √n = ± 6, [using (ii)]

Solving (ii) and (iii), we get m = 16, n = 4

Hence, the required numbers are 16 and 4.

5.5.1 Problem Set 5.5


1. Find the sum of the first 20 terms of the sequence 4, 6, 8, 10, ….
2. Find the 9th term of the sequence 1, √2, 2, ….
3. Evaluate this series using a formula:
14

∑(1 − 2𝑘)
𝑘=1

4. A display of cans on a grocery shelf consists of 20 cans on the bottom, 18


cans in the next row, and so on in an arithmetic sequence, until the top row
has 4 cans. How many cans, in total, are on the display?
5. Find the sum of the series
1 + 3.5 + 6 + 8.5 + ⋯ + 101
6. Find the sum of the geometric series
2 + 6 + 18 + 54 + ⋯
Where there are 6 terms in the series

5.6 Indices
General Objectives: Use indices
Specific Objectives:
1. Use and interpret positive, negative and zero indices
2. Apply the laws of indices correctly
3. Use and interpret fractional indices e.g., 21/2 = √2
296

3𝑥−𝑦 × 3𝑥+𝑦
4. Simplify expressions involving indices e.g.,
32𝑥
1
5. Solve simple exponential equations e.g., 2𝑥 =
4

Let us take a look at the example 34.The number 4 is called the index and the
number 3 is the base. so 40 also indicates that 0 is the index and 4 is the base. The
index which is also called the power or exponent can be positive, negative or
zero.

To manipulate expressions involving indices we use rules known as the laws of


indices.
First law :
am× an = a(m+n)
Second Law:
am
= a(m-n)
an
Third law:
(am)n = amn
When expressions with the same base are multiplied, the indices are added.
Q:We can write
76 × 74 = 7(6+4) = 710
76
= 7(6-4) = 72
74
(76)4 = 7(6x4) = 724
297

5.6.1 Fractions in Power


Same rules of exponents are applied for an algebraic expression if it has power in
a fraction.
𝐱
√𝐱 = 𝐱 𝟏/𝟐 √𝐱 × √𝐱 =x = √𝐱
𝟑 √𝐱
√𝐱 = 𝐱 𝟏/𝟑
𝟑 𝟑 𝟑
√𝐱 × √𝐱 × √𝐱 = 𝐱
√𝐱 𝟏
√𝐱 𝟐 = ±𝐱 √𝐱 𝟑 = +𝐱 𝐚𝐧𝐝 √−𝐱 𝟑 =
𝐱 √𝐱
= −𝐱

The laws of exponents are explained here along with their examples.

1. Multiplying powers with same base

For example: 2³ × 2², 34 × 32 , (-3)³ × (-3)⁴, m5 × m3

In multiplication of exponents, if bases are the same then we need to add


exponents together.

Consider the following:

1. 2³ × 2²= (2 × 2 × 2) × (2 × 2) = 2⁵

2. 3⁴ × 3² = (3 × 3 × 3 × 3) × (3 × 3) = 3⁶

3. (-3)³ × (-3)⁴ = [(-3) × (-3) × (-3)] × [(-3) × (-3) × (-3) × (-3)] =(-3)⁷

4. m⁵ × m³ = (m × m × m × m × m) × (m × m × m) = m⁸
298

From examples on the previous page, we can generalize that during multiplication
when bases are the same then exponents are added together.

aᵐ × aⁿ = am+n

In other words, if ‘a’ is a non-zero integer or a non-zero rational number and m


and n are positive integers, then

aᵐ × aⁿ = am+n

𝐚 𝐚 𝐚
Similarly, ( )ᵐ × ( )ⁿ = ( )m+n
𝐛 𝐛 𝐛

Note:

(i) Exponents can be added only when the bases are same.

(ii) Exponents cannot be added if the bases are not the same.

2. Dividing powers with the same base


For example:

2⁷ ÷ 2⁴, 5⁶ ÷ 5², 10⁵ ÷ 10³, 7⁴ ÷ 7⁵

In division, if bases are same then we need to subtract the exponents.

Consider the following:

2⁷ (2 × 2 × 2 × 2 × 2 × 2 × 2)
2⁷ ÷ 2⁴ = = = 2(7- 4) = 2³
2⁴ (2 × 2 × 2 × 2)

56 (5 × 5 × 5 × 5 × 5 × 5)
5⁶ ÷ 5² = = = 5(6 - 2) = 5⁴
52 (5 × 5)

105 (10 × 10 × 10 × 10 × 10)


10⁵ ÷ 10³ = = =10(5 - 3) =10²
103 (10 × 10 × 10)
299

74 (7 × 7 × 7 × 7)
7⁴ ÷ 7⁵ = = =7(4 - 5) = 7−1
75 (7 × 7 × 7 × 7 × 7)

Let a be a non zero number, then

a5 (a × a × a × a × a)
a⁵ ÷ a³ = = = a²
a3 (a × a × a)

a3 (a × a × a)
again, a³ ÷ a⁵ = = = a−2
a5 (a × a × a × a × a)

Thus, in general, for any non-zero integer a,

𝐚𝐦
aᵐ ÷ aⁿ = = am−n
𝐚𝐧

5.6.2 Problem Set 5.6


1. Evaluate the value of x in expression(𝑥 − 3)2/5 = (4𝑥)1/5 .
2. Evaluate (𝑥 − 7)4 = 16
3. Evaluate the following expressions:
a) (2𝑎3 )2
b) (5𝑦 4 )2
c) 5𝑎−2 × 6𝑎−3
d) 4(3𝑎3 )2
2
4. What is the value of (𝑎 + 𝑏)2 when 𝑎 = 𝑎𝑛𝑑 𝑏 = −2a
3
5. Find the value of (2𝑥 − 6)0 when 𝑥 does not equal to 3.
300

The log rules might be expressed as:

1) Multiplication inside the log can be turned into addition outside the log, and
vice versa.
2) Division inside the log can be turned into subtraction outside the log, and vice
versa.
3) An exponent on everything inside a log can be moved out front- side as a
multiplier, and vice versa.
Example: 𝒂𝒙 = 𝒃
Taking logarithms both sides,
𝑥 log a = log b
log(b)
𝑥=
log(𝑎)

𝑥 = log a b

5.7 Logarithms
General Objectives: Understand the relationship between indices and logarithms
(only to base 10)
Specific Objectives:
1. Use the laws of logarithms to simplify expressions
2. Solve equations of the form 𝑎 𝑥 = 𝑏 using logarithms
5.7.1 Log Rules
I. 𝑙𝑜𝑔𝑏 (𝑚𝑛) = 𝑙𝑜𝑔𝑏 (𝑚) + 𝑙𝑜𝑔𝑏 (𝑛)
𝑚
II. 𝑙𝑜𝑔𝑏 ( ) = 𝑙𝑜𝑔𝑏 (𝑚) − 𝑙𝑜𝑔𝑏 (𝑛)
𝑛
III. 𝑙𝑜𝑔𝑏 (𝑚𝑛 ) = 𝑛 . 𝑙𝑜𝑔𝑏 (𝑚)
IV. 𝑙𝑜𝑔𝑏 (𝑏) = 1
V. 𝑙𝑜𝑔𝑏 (1) = 0
VI. 𝑙𝑜𝑔𝑏 (𝑏𝑘 ) = 𝑘
VII. 𝑏. 𝑙𝑜𝑔𝑏 (𝑥) = 𝑥

In mathematics, logarithms were developed for making complicated calculations


simple.
301

For example, if a right circular cylinder has a radius r = 0.375 meters and height h
= 0.2321 meters, then its volume is given by: V = A = πr2h = 3.146 × (0.375)2×
0.2321. Use for logarithm tables makes such calculations quite easy. However,
even calculators have functions like multiplication; power etc. still, logarithmic
and exponential equations and functions are very common in mathematics.

Definition:

If ax = M (M > 0, a > 0, a ≠ 1), then x (i.e., index of the power) is called the
logarithm of the number M to the base a and written as x = loga M .

Hence, if ax = M then x = loga M;

conversely, if x = loga M then ax = M.

If ‘a’ is a positive real number (except 1), n is any real number and an = b, then n is
called the logarithm of b to the base a. It is written as loga b (read as log of b to
the base a). Thus,

an = b ⇔ loga b = n.

an is called the exponential form and loga b = n is called the logarithmic form.

For example:
● 32 = 9 ⇔ log3 9 = 2

● 54 = 625 ⇔ log5 625 = 4

● 70 = 1 ⇔ log7 1 = 0
302

1 1
● 2-3 = ⇔ log2 ( )= -3
8 8

● 10-2 = 0.01 ⇔ log10 0.01 = -2

● 26 = 64 ⇔ log2 64 = 6

14 1 1
● 3- 4 = = ⇔ log3 = -4
3 81 81

1
● 10-2 = = 0.01 ⇔ log10 0.01 = -2
100

Notes on basic Logarithm Facts:


1. Since a > 0 (a ≠ 1), an > 0 for any rational n. Hence logarithm is defined only by
positive real numbers.

From the definition it is clear that the logarithm of a number has no meaning if
the base is not mentioned.

2. The above examples shows that the logarithm of a (positive) real number may
be negative, zero or positive.

3. Logarithmic values of a given number are different for different bases.

4. Logarithms to the base 10 are called common logarithms. Also, logarithm


tables assume base 10. If no base is given, the base is assumed to be 10.
Example: log 21 means log10 21

5. Logarithm to the base ‘e’ (where e = 2.7183 approx.) is called natural


logarithm, and usually written as ln. Thus ln x means loge x.
303

6. If ax = - M (a > 0, M > 0), then the value of x will be imaginary i.e., logarithmic
value of a negative number is imaginary.

7. Logarithm of 1 to any finite non-zero base is zero.

Proof: We know, a0 = 1 (a ≠ 0). Therefore, from the definition, we have, loga 1 = 0.

8. Logarithm of a positive number to the same base is always 1.

Proof: Since a1 = a. Therefore, loga a = 1.

Note:
From 7 and 8 we said that, loga 1 = 0 and loga a = 1 for any positive real ‘a’ except
1.

9. If x = loga M then a log a M = a

Proof: x = loga M. Therefore, ax = M or, a logaM = M [Since, x = loga M].

Summary: There are four math logarithm formulas:

● Product Rule Law:

loga (MN) = loga M + loga N

● Quotient Rule Law:

M
loga ( )= loga M - loga N
N
304

● Power Rule Law:

IogaMn = n Ioga M

● Change of base Rule Law:

loga M = logb M × loga b


Let’s observe the detailed step-by-step explanation of mathematical proof of
logarithm rules or log rules.

1. Proof of Product Rule Law:


loga (MN) = loga M + loga N

Let loga M = x ⇒ a sup>x = M

and Ioga N= y ⇒ ay = N

Now ax ∙ ay = MN or, ax + y = MN

Therefore, from the definition we have:

loga (MN) = x + y = loga M + loga N [putting the values of x and y]

Corollary: The law is true for more than two positive factors i.e.,

loga (MNP) = loga M + loga N + loga P

since, loga (MNP) = loga (MN) + loga P = loga M+ loga N+ loga P

Therefore in general, loga (MNP ….... )= loga M + loga N + loga P + ……. .

Hence, the logarithm of the product of two or more positive factors to any positive
base other than 1 is equal to the sum of the logarithms of the factors to the same
base.
305

Solved Examples:
Example: Find the logarithms of:

(i) 1728 to the base 2√3

Solution:

Let x denote the required logarithm.

Therefore, log2√3 1728 = x

or, (2√3)x = 1728 = 26 ∙ 33 = 26 ∙ (√3)6

or, (2√3)x = (2√3)6

Therefore, x = 6.

(ii) 0.000001 to the base 0.01

Solution:

Let y be the required logarithm.

Therefore, log0.01 0.000001 = y

or, (0.01y = 0.000001 = (0.01)3

Therefore, y = 3
306

Example: Convert the following exponential form to logarithmic form:

(i) 104 = 10000

Solution:

104 = 10000

⇒ log10 10000 = 4

(ii) 3-5 = x

Solution:

3-5 = x

⇒ log3 x = -5

(iii) (0.3)3 = 0.027

Solution:

(0.3)3 = 0.027

⇒ log0.3 0.027 = 3

Example: Convert the following logarithmic form to exponential form:

(i) log3 81 = 4
307

Solution:

log3 81 = 4

⇒ 34 = 81, which is the required exponential form.

5
(ii) log8 32 =
3

Solution:

5
log8 32 =
3

⇒ 85/3 = 32

(iii) log10 0.1 = -1

Solution:

log10 0.1 = -1

⇒ 10-1 = 0.1

Example: By converting to exponential form, find the values of the following:

(i) log2 16

Solution:

Let log2 16 = x
308

⇒ 2x = 16

⇒ 2x = 24

⇒x=4

Therefore, log2 16 = 4

1
(ii) log3 ( )
3

Solution:

1
Let log3 ( ) = x
3

1
⇒ 3x =
3

⇒ 3x = 3-1

⇒ x = -1,

1
Therefore, log3( ) = -1
3

(iii) log5 0.008

Solution:

Let log5 0.008 = x

⇒ 5x = 0.008

1
⇒ 5x =
125
309

⇒ 5x = 5-3

⇒ x = -3

Therefore, log5 0.008 = -3

Example: Solve the following for x:

(i) logx 243 = -5

Solution:

logx 243 = -5

⇒ x-5 = 243

⇒ x-5 = 35

1
⇒ x-5 = ( )-5
3

1
⇒x=
3

(ii) log√5 x = 4

Solution:

log√5 x = 4

⇒ x = (√5)4
310

⇒ x = (51/2)4

⇒ x = 52

⇒ x = 25

(iii) log√x 8 = 6

Solution:

log√x 8 = 6

⇒ (√x)6 = 8

⇒ (x1/2)6 = 23

⇒ x3 = 23

⇒x=2
Example: log(𝑥-2) – log(2𝑥-3) = log2
Solution: log(𝑥-2) – log(2𝑥-3) = log2
(𝑥−2)
log (2𝑥−3) = log2
(𝑥−2)
Cancelling log both sides, (2𝑥−3) = 2

𝑥-2 = 4𝑥-6
3𝑥 =4
4
𝑥=
3
311

Example: If log10 2 = 0.30103, log10 3 = 0.47712 and log10 7 = 0.84510, find the
values of :

(i) log10 45

(ii) log10 105

(i) log10 45

Solution:

log10 45 = log10 (5 × 9)

= log10 5 + log10 9

10
= log10 ( ) + log10 32
2

= log10 10 - log10 2 + 2 log10 3

= 1 - 0.30103 + 2 × 0.47712

= 1.65321

(ii) log10 105

Solution:

log10 105

= log10 (7 x 5 x 3)

= log10 7 + log10 5 + log10 3


312

10
= log10 7 + log10 ( ) + log10 3
2

= log10 7 + log10 10 - log10 2 + log10 3

= 0.8450 + 1 - 0.30103 + 0.47712

= 2.02119

Example: Prove that, logb a × logc b × logd c = logd a

Solution:

L. H. S. = logb a × logc b × logd C

= logc a × logd c [since logb M × loga b = loga M]

= logd a (using the same formula)

Alternative Method:

Let, logb a = x Since, bx = a

logc b = y Therefore, cy = b

and logd c = z Therefore, dz = c

Now, a = bx = (cy)x = cxy = (dz)xy = dxyz

Therefore logd a = xyz = logb a × logc b × logd c. (putting the value of x, y, z)


313

Example: Show that, log4 2 × log2 3= log4 5 × log5 3

Solution:

L. H. S. = log4 2 × log2 3

= log4 3

= log5 3 × log4 5 Proved

Example: Show that, log2 10 - log8 125 = 1

Solution:

We have, log8 125 = log8 53 = 3 log8 5

1 1 1
=3∙( )= 3 ∙ ( )=3∙( ) = log2 5
log5 8 log5 2³ 3log5 2

Therefore, L.H. S. = log2 10 - log8 125 = log2 10 - log2 5

10
= log2 ( ) = log2 2 = 1 Proved.
5

𝐱 𝐲 𝐳
Example: If log = log = log
(𝐲 − 𝐳) (𝐳− 𝐱) (𝐱−𝐲)
x y z
show that, x y z = 1

Solution:

𝐱 𝐲 𝐳
Let, log = log = log =k
(𝐲 − 𝐳) (𝐳− 𝐱) (𝐱−𝐲)

Therefore, log x = k(y - z) ⇔ x log x = kx(y - z )

or, log xx = kx(y - z) ... (1)


314

Similarly, log yy = ky (z - x) ... (2)

and log zz = kz(x - y) ... (3)

Now, adding (1), (2) and (3) we get,

log xx + log yy + log zz = k (xy - xz + yz - xy + zx - yz)

or, log (xx yy zz) = k × 0 = 0 = log 1

Therefore, xx yy zz = 1 Proved

5.7.2 Problem Set 5.7


1. Expand 𝑙𝑜𝑔3 (2𝑥).
2. Expand 𝑙𝑜𝑔5 (𝑥 3 ).
3. Solve 𝑙𝑜𝑔2 (𝑥 + 1) + 𝑙𝑜𝑔2 (3) = 4
4. Simplify 3𝑙𝑜𝑔2 (𝑥) − 4𝑙𝑜𝑔2 (𝑥 + 3) + 𝑙𝑜𝑔2 (𝑦)
315

Chapter 6

6.1 Graphs in practical situations

General Objectives: understand the use of graphs in practical situations


Specific Objectives:
1. demonstrate familiarity with the Cartesian coordinates in two dimensions
2. Interpret and use graphs in practical situations, including travel graphs and
conversion graphs
3. Draw graphs from given data
4. Apply the idea of rate of change to easy kinematics involving distance-time
and speed-time graphs, acceleration and deceleration
5. Calculate distance travelled as area under a linear speed-time graph
6.1.1 Graph

The above table tells us that the bus is covering equal distances in equal intervals
of time i.e., the bus is moving with uniform speed. If the bus continues to move
with uniform speed then we can calculate the distance covered by the bus at any
given time.
Consider an object moving with uniform speed v from its initial position xi to final
position xf in time t.
316

The equation (1) gives the relation between distance, time and average speed.
This relation can be used for constructing distance-time tables and also to
determine the position of any moving object at any given time.
But it is a long and tedious process especially when we have to determine the
position after a long time or when we have to compare the motion of two objects.
In such situations we can make use of graphs like distance-time graph.
A distance-time graph is a line graph showing the variation of distance with time.
In a distance-time graph, time is taken along x-axis and distance along y-axis.
Let us now plot a distance-time graph for the example on the previous page.

Distance-time graph for uniform motion


 Take time along x-axis and distance along y-axis.
 Analyze the given data and make a proper choice of scale for time and
distance.
 Plot the points.
 Consider any two points (A, B) on the straight line graph.
 Draw perpendiculars from A and B to x and y axes.
 Join A to C to get a right-angled triangle ACB.
317

Calculation:

 Write the title and scale chosen for the graph.


 Consider another two points P and Q on the graph and construct a right-
angled triangle PRQ.

The distance-time graph is a straight line showing that the motion is uniform.
Thus from the distance-time graph (S-t) we calculate the speed as:

BC
The ratio is the slope of the graph. Hence, the slope of the distance-time graph
AC
gives the speed of the moving object.

6.1.2 Speed-Time Graph


If the speed of an object is increasing at a constant rate, its motion can be
represented by the line segment OA, which has a positive gradient in a speed-
time graph. The rate of increase of speed is called acceleration. The gradient gives
the value of the acceleration.
318

If the object is moving with a constant speed, its motion is represented by the
horizontal line segment AB. If the speed is decreasing at a constant rate, the line
segment BC representing the motion has a negative gradient. The rate of
decrease of speed is called retardation or deceleration.
6.1.3 Problem Set 6.1
1. The diagram below shows the distance from A of an object at various times.
Distance from A (m)

Time (s)

a) Find from the graph


i. The distance from A when time is 2 seconds
ii. The time when the distance from A is 15m.
b) Calculate the speed of the object in m/s

2. Find the slope of the line through the points (1,4) and (4,8).

3. The speed-time graph of a car consists of the line segments as shown


below:
319

30
0

Speed in m/s
20

10

0 5 10 15 20 25 30 35 40 45 50 55
Time in seconds (S)

a) Find the acceleration for the first 5 seconds.


b) Find the acceleration for the next 15 seconds.
c) Find the retardation for the last 15 seconds.
d) What is the maximum speed of the car? For how long does the car travel
at this speed?

6.2 Linear Programming

General Objectives: represent and apply linear inequalities graphically


Specific Objectives: represent inequalities graphically and use this representation
in the solution of simple linear programming problems (the convention of using
broken lines for strict inequalities and shading unwanted regions will be
expected).

Solution of simple linear programming Problems

Linear programming is the process of taking various linear inequalities relating to


some situation, and finding the "best" value obtainable under those conditions.

In "real life", linear programming is part of a very important area of mathematics


called "optimization techniques".

Example : Find the maximal and minimal value of z = 3𝑥 + 4y subject to the


following constraints:
320

𝒙 + 2y <= 14,

3𝒙 - y >= 0,

𝒙 - y <= 2

The three inequalities in the curly braces are the constraints. The area of the
plane that they mark off will be the feasibility region.

The formula "z = 3𝑥 + 4y" is the optimization equation. I need to find the (𝑥, y)
corner points of the feasibility region that return the largest and smallest values
of z.

My first step is to solve each inequality for the more-easily graphed equivalent
forms:

It's easy to graph the system:


321

y = –( 1/2 )x + 7 y = –( 1/2 )x + 7 y = 3x
y = 3x y=x–2 y=x–2
–( 1/2 )x + 7 = 3x –( 1/2 )x + 7 = x – 2
3x = x – 2
–x + 14 = 6x –x + 14 = 2x – 4
2x = –2
14 = 7x 18 = 3x
x = –1
2=x 6=x
y = 3(–1) = –3
y = 3(2) = 6 y = (6) – 2 = 4
corner point at (2, 6) corner point at (6, 4) corner point at (–1, –3)

So the corner points are (2, 6), (6, 4), and (–1, –3).

It has been proved that, for linear systems like this, the maximum and minimum
values of the optimization equation will always be on the corners of the feasibility
region. So, to find the solution to this exercise, I only need to plug these three
points into "z = 3𝑥 + 4y".

(2, 6): z = 3(2) + 4(6) = 6 + 24 = 30

(6, 4): z = 3(6) + 4(4) = 18 + 16 = 34

(–1, –3): z = 3(–1) + 4(–3) = –3 – 12 = –15

Then the maximum of z = 34 occurs at (6, 4),

And the minimum of z = –15 occurs at (–1, –3).

Example: Graph the solution to 2𝒙– 3y < 6.


First, I'll solve for y:
2𝑥 – 3y < 6
–3y < –2𝑥 + 6
2
y > ( )𝑥 – 2
3
2
Now I need to find the "equal" part, which is the line y = ( ) 𝑥 – 2
3
322

But this exercise is what is called a "strict" inequality.


That is, it isn't "or equals to" inequality; it's only "y greater than".
When I had strict inequalities on the number line (such as x < 3), I denote this by
using a parenthesis (instead of a square bracket) or an open [unfilled] dot (instead
of a closed [filled] dot). In the case of these linear inequalities, the notation for a
strict inequality is a dashed line. So the border of my solution region actually looks
like this:

By using a dashed line, I still know where the border is, but I also know that the
border isn't included in the solution. Since this is a "y greater than" inequality, I
want to shade above the line, so my solution looks like this:
323

6.2.1 Problem Set 6.2

1. Use graphical method to solve the following linear programming problem.

𝑀𝑎𝑥𝑖𝑚𝑖𝑧𝑒 𝑧 = 2𝑥 + 10𝑦

𝑆𝑢𝑏𝑗𝑒𝑐𝑡 𝑡𝑜 𝑡ℎ𝑒 𝑐𝑜𝑛𝑠𝑡𝑟𝑎𝑖𝑛𝑡𝑠 2𝑥 + 5𝑦 ≤ 16,

𝑥 ≤ 5,

𝑥 ≥ 0, 𝑦 ≥ 0.

2. Graph the feasible region of the following system of linear inequalities and
find the corner points in each case:

a)

2𝑥 − 3𝑦 ≤ 6

2𝑥 + 3𝑦 ≤ 12

𝑥 ≥ 0, 𝑦 ≥ 0

b)

𝑥+5≤5

−2𝑥 + 𝑦 ≥ 2

𝑥≥0

6.3 Functions

General Objectives: understand the functions idea and use function notations.
Specific Objectives:
324

1. Use function notation e.g., 𝑓(𝑥) = 3𝑥 − 5 ; 𝑓: 𝑥 → 3𝑥 − 5 to


describe simple functions and the notation 𝑓 −1 (𝑥) to describe their
inverses.
2. Form composite functions as defined by 𝑔𝑓(𝑥) = 𝑔(𝑓(𝑥))

A function relates an input to an output. It is like a machine that has an input and
an output.

An inverse function is a function that undoes the action of another function. By


using a function machine metaphor, forming an inverse function means running
the function machine backwards. The backwards function machine will work only
if the original function machine produces a unique output for each unique input.

Example 1: Find the inverse of y = 3x – 2.


Solution: Here's how it works:
𝒙+𝟐
Then the inverse is y = ( )
𝟑

−𝟐
Example 2: Find the inverse of y = ( ) , and determine whether the inverse is
𝒙−𝟓
also a function?
Solution: Since the variable is in the denominator, this is a rational function.
Here's the algebra:
325

𝟓𝒙−𝟐
The inverse function is y = ( 𝒙
)

Function composition is the point wise application of one function to the result of
another to produce a third function. For instance, the
functions f : X → Y and g : Y → Z can be composed to yield a function which
maps x in X to g(f(x)) in Z. Intuitively, if z is a function of y, and y is a function
of x, then z is a function of x. The resulting composite function is
denoted g ∘ f : X → Z, defined by (g ∘ f )(x) = g(f (x)) for all x in X.

More generally, if we have two functions y = f(x) and z= g(y) , we


call the new function z = g (f(x))

The composite of f and g and denote it by gof (x) = g(f(x)) .

Example: f(x) = √x and g(x) = x2

Solution: The Domain of f(x) = √x is all non-negative Real Numbers

The Domain of g(x) = x2 is all the Real Numbers

The composed function is:

(g of)(x) = g (f(x))

= (√x)²

=x

Now, "x" normally has the Domain of all Real Numbers. But because it is a
composed function you must also consider f(x),

Example : Given f(x) = 2x + 3 and g(x) = –x2 + 5, find ( f o g)(x).

Solution: In this case, I am not trying to find a certain numerical value. Instead, I
am trying to find the formula that result from plugging the formula for g(x)
326

into the formula for f(x). I will write the formulas at each step, using
parentheses to indicate where the inputs should go:

( f o g)(x) = f (g(x))

= f (–x2 + 5)

= 2( )+3 ..insert the input formula

= 2(–x2 + 5) + 3

= –2x2 + 10 + 3

= –2x2 + 13

6.3.1 Problem Set 6.3


1. Evaluate the following composite functions
a) 𝑓(𝑥) = 𝑥 + 4 ; 𝑔(𝑥) = 𝑥 − 4 ; 𝐹𝑖𝑛𝑑 (𝑓𝑜𝑔)(𝑥)𝑎𝑛𝑑 (𝑔𝑜𝑓)(𝑥)
b) 𝑔(𝑥) = 4 + √𝑥 2 + 4𝑥 ; ℎ(𝑥) = 𝑥 − 1 ; 𝐹𝑖𝑛𝑑 (𝑔𝑜ℎ)(𝑥)
c) 𝑓(𝑥) = 𝑥 2 − 4𝑥 + 4 ; 𝑔(𝑥) = 𝑥 + 1 ; 𝐹𝑖𝑛𝑑 (𝑓𝑜𝑔)(𝑥)𝑎𝑛𝑑 (𝑔𝑜𝑓)(𝑥)
2. Evaluate the inverses of the following functions
a) 𝑓(𝑥) = 𝑥 + 4
b) 𝑓(𝑥) = 1 + √𝑥 + 4
c) 𝑓(𝑥) = (𝑥 + 4 )2 -5
d) 𝑓(𝑥) = (1 + √𝑥 2 + 4)2
327

6.4 Graphs of functions

General Objectives: construct tables of values for functions, draw and interpret
graphs and solve equations graphically.
Specific Objectives:
1. Construct tables of values for functions of the form 𝑦 = 𝑎𝑥 + 𝑏, 𝑦 = ±𝑥 2 +
𝑎
𝑎𝑥 + 𝑏, 𝑦 = , (𝑥 ≠ 0) where 𝑎 and 𝑏 are integral constants.
𝑥
2. Draw and interpret such graphs.
3. Find the gradient of a straight line graph and determine the equation of a
straight line in the form 𝑦 = 𝑚𝑥 + 𝑐
4. Solve linear and quadratic equations approximately by graphical methods.
5. Construct Tables of values and draw graphs for functions of the form 𝑦 = 𝑎𝑥 𝑛
where a is a rational constant and n= -2,-1,0,1,2,3 and simple sums of not
more than three of these and for functions of the form 𝑦 = 𝑎 𝑥 where a is a
positive integer
6. Estimate gradients of curves by drawing tangents
7. Solve associated equations approximately by graphical methods.

6.4.1 Drawing Straight Line graphs


Following are some of the linear relations because they generate straight line
graphs
𝑦 = 3𝑥 − 2
𝑦 = 1 + 2𝑥
𝑦 = 6𝑥
𝑦 = 10 − 5𝑥
For plotting 𝑦 = 1 + 2𝑥, the table can be tabulated as shown below:
x 0 1 2 3 4
y 1 3 5 7 9
The points appear to lie on a straight line as shown on the next page.
328

𝑦 =1+2𝑥

6.4.2 The slope-intercept form of straight line equation


𝑦 = 𝑚𝑥 + 𝑐
Above is the equation of straight line written in the form of intercept-slope form.
𝑚 = 𝑡ℎ𝑒 𝑠𝑙𝑜𝑝𝑒 𝑜𝑓 𝑡ℎ𝑒 𝑔𝑟𝑎𝑝ℎ
𝑐 = 𝑡ℎ𝑒 𝑦 − 𝑖𝑛𝑡𝑒𝑟𝑐𝑒𝑝𝑡 𝑜𝑓 𝑡ℎ𝑒 𝑔𝑟𝑎𝑝ℎ
This form is used while modelling various practical scenario problems. Two-
variable statistics i.e., bivariate problems also make use of the above form.

Example : Draw Y=3?


Solution: Since y=3 does not contain a variable to solve for. Move it to the right.
Use the slope-intercept form to find the slope and y-intercept.
The slope-intercept form is :
y=m𝑥+c
y=m𝑥+c, where m is the slope(gradient) and c is the y-intercept.
y=m𝑥+c
Find the values of m and c using the form y=m𝑥+c
m=0
c=3. The slope of the line is the value of m and the y-intercept is the value of
c . Slope: 0 , Y-Intercept: 3
329

The Gradient (also called Slope) of a straight line shows how steep a straight line
is.
Gradient = Change in Y / Change in X

 A line that goes straight across (Horizontal) has a Gradient of zero.


 straight up and down (vertical) line's Gradient is "undefined".

Example : Find the gradient of the curve y = x² at the point (3, 9).
330

This line is the tangent.


9

2.3 3

(change in y)
Solution: Gradient of tangent =
(change in x)
(9 − 5)
=
(3 − 2.3)
= 5.71

Note: this method only gives an approximate answer. The more accurate your
graph is, the closer you would be to the correct answer.

There are two main things about graphs of Polynomials:

1) The graphs of polynomials are continuous, which is a special term with an exact
definition in calculus, but here we will use this simplified definition.

2) The graphs of polynomials are also smooth. No sharp "corners" or "cusps".

Power Function: Including a multiplier of “a “we get what is called a "Power


Function"

F(𝑥) = a𝑥 n
f(𝒙) equals a times 𝑥 to the "power" (i.e. exponent) n

The "a" changes it this way:

 Larger values of a squash the curve (inwards to y-axis)


 Smaller values of a expand it (away from y-axis)
 And negative values of a flip it upside down
331

Quadratic Graph:
The graph of y = a𝑥² + b𝑥 + c:
A nonlinear function that can be written in standard form a𝑥²+b𝑥+c, where, a≠0 is
called a quadratic function.
All quadratic functions have a U-shaped graph called a parabola. The parent
quadratic function is
y=𝑥 2

Vertex

The lowest or the highest point on a parabola is called the vertex. The vertex has
𝐛
the x-coordinate -𝒙=
𝟐𝒂
The y-coordinate of the vertex is the maximum or minimum value of the function.
332

a>0 parabola opens up minimum value

a<0 parabola opens down maximum value

The vertical line that passes through the vertex and divides the parabola in two is
called the axis of symmetry. The axis of symmetry has the equation
𝐛
x= − 𝟐𝒂
333

The y − intercept of the equation is 𝐜.

If you would like to graph a quadratic function you begin by making a table of
values for some values of your function and then plot those values in a coordinate
plane and draw a smooth curve through the points.

Example: Graph y=𝑥 2+2𝑥+1


Solution: Make a table of value for some values of 𝑥. Use both positive and
negative values!
x Equation y

-3 Y=(-3)²+2×(-3)+1 4

-2 Y=(-2)²+2×(-2)+1 1

-1 Y=(-1)²+2×(-1)+1 0

0 Y=(0)²+2×(0)+1 1

1 Y=(1)²+2×(1)+1 4

2 Y=(2)²+2×(2)+1 9

3 Y=(3)²+2×(3)+1 16

Graph the points and draw a smooth line through the points and extend it in both
directions.
334

Notice that we have a minimum point which was indicated by a positive a value
(a = 1). The vertex has the coordinates (-1, 0) which is what you will get if you use
the formula for the x-coordinate of the vertex:
𝑏 2
𝑥=− =− = −1
2𝑎 2×1
Also, the line has a y-intercept of (0, 1) which could be determined from the c-
value which is 1.
6.4.3 Problem Set 6.4
1. Plot the graph of 𝑦 = 8 − 2𝑥 by taking different values of 𝑥 where
0≤𝑥≤4
2. Sketch the graph of 𝑦 = 8 + 2𝑥
3. Sketch 𝑦 = 𝑥 2 + 2𝑥 + 1 where −3 ≤ 𝑥 ≤ 3
3 −2
4. 𝑦 = − 1 and 𝑦 = −2
𝑥−2 𝑥+1
5. The temperature was recorded every hour and the results are given below:

Time 0900 1000 1100 1200 1300 1400 1500 1600


Temperature 12 13 16 18 22 21 20 18

a) Draw a line graph for the above mentioned problem


b) Estimate the temperature at 1230
c) Estimate the temperature at 1530
335

Chapter 7
7.1 Graphs and Cartesian Coordinates
General Objectives: use rectangular Cartesian coordinates in two dimensions and
understand the relationship between a graph and an associated algebraic
equation
Specific Objectives:
1. Calculate the distance between two points given in coordinate form, the
gradient of the line-segment joining them and the coordinates of their
midpoint.
2. Find the equation of a straight line given sufficient information (e.g., the
coordinates of two points on it or one point on it and its gradient).
3. Interpret and use equations of the form 𝑎𝑥 + 𝑏𝑦 + 𝑐 = 0, including
knowledge of the relationships involving gradients of parallel and
perpendicular lines.
4. Apply coordinate geometry to quadrilaterals.

We use Cartesian Coordinates to mark a point on a graph by how far along and
how far up it is. To measure the change in 'x' and 'y' between the two points, we
draw a right-angled triangle beneath the graph. The length of the vertical leg is:
(y2-y1), and the horizontal leg has length (x2-x1). In the diagram below, (x1, y1) = (2,
3) and (x2, y2) = (6, 8).
(𝑋2 , 𝑌2 ) = (6,8)

∆𝑌 = (𝑌2 − 𝑌1 ) = 8 − 3
=5
(𝑋1 , 𝑌1 ) = (2,3)
∆𝑋 = (𝑋2 − 𝑋1 ) = 6 − 2
=4

So the gradient of this line segment is:


∆𝑦 5
𝑔𝑟𝑎𝑑𝑖𝑒𝑛𝑡 ′𝑚′ = = = 1.25
∆𝑥 4
336

Q: Find the equation of a line through the points (3,7) and (5,11).
Calculate the slope from 2 points

step 1 find the slope:

Step 2
Substitute the slope for 'm' in the slope intercept form of the equation.

y = m𝑥 +b
y = 2𝑥 +b

Step 3
Substitute either point into the equation. You can use either (3,7) or (5,11)

Step 4
Solve for b, which is the y-intercept of the line

-6+7=b

Step 5
Substitute b, 1, into the equation from step 4

Example: Find the linear equation of the line that passes through the points (2,1)
and (5,7). Your answer must be in the form of A𝑥 + B𝑦 + C = 0.
337

Using the equation:


(y1 – y2)𝑥 + (𝑥2 – 𝑥1)y + (𝑥1y2 – 𝑥2y1) = 0

We’ll just plug numbers in:


(1 – 7)𝑥 + (5 – 2)𝑦 + ( (2 x 7) – (5 x 1) ) = 0
-6𝑥 + 3y + (14 – 5) = 0
-6𝑥 + 3y + 9 = 0

Factoring -3 out:
-3( 2𝑥 – y – 3 ) = 0

Dividing both sides by -3:


2𝑥 – y – 3 = 0

Example: What is the area in square units, of a quadrilateral whose vertices are
(5,3),(6,−4),(−3,−2),(−4,7)?
The distance between two points (a1,a2)(a1,a2) and (b1,b2)(b1,b2) is given by
d= √(a1−b1)2+(a2−b2)2
(1) Determine the diagonal "vectors" (by subtracting the opposite vertices):
d1 = (5, 3) − (-3,-2) = (8, 5)
d2 = (6,-4) − (-4, 7) = (10,-11)

1
(2) The area equals of the absolute value of the "cross-product" of the
2
diagonals:
1 1
𝑥 = × | (8 ×-11) - (10 × 5) | = ×138 = 69
2 2

This works for any irregular quadrilateral.


338

7.1.1 Equations of a Straight Line


A straight line is completely determined if it passes through two fixed points or it
passes through a fixed point and is parallel to a given vector. We first find
equation of a line L through a given point and parallel to a given vector.
Let A (x1, y1, z1) be a fixed point on the line L whose direction is parallel to a vector
d=ai + bj +ck
Suppose P (x, y, z) is any point on L. Then
⃗⃗⃗⃗⃗
𝐴𝑃 = 𝑡𝐝
Where t is some real number. It is a vector equation of the line through A and
parallel to d.
Now
⃗⃗⃗⃗⃗
𝐴𝑃 = (𝑥 − 𝑥1 )𝐢 + (𝑦 − 𝑦1 )𝐣 + (𝑧 − 𝑧1 )𝐤
The vector equation of the line L is
(𝑥 − 𝑥1 )𝐢 + (𝑦 − 𝑦1 )𝐣 + (𝑧 − 𝑧1 )𝐤 = 𝑡(𝑎𝐢 + 𝑏𝐣 + 𝑐𝐤)
Equating components
(𝑥 − 𝑥1 ) = 𝑎𝑡
(𝑦 − 𝑦1 ) = 𝑏𝑡
(𝑧 − 𝑧1 ) = 𝑐𝑡
These are called parametric equations of the required line L.
339

7.1.2 Problem Set 7.1


1. Calculate the distance between two points (-2, 1) and (1, 5).

2. Find the equation of the straight line that has slope m=4 and passes
through the point (-1,-6).
3. What is the slope (or gradient) of the line formed by two pints A (6, 4) and
B (2, 3).
4. Find the midpoint of line segment𝐴𝐵 ̅̅̅̅ , where A (-3, 4) and B (2, 1).
5. Find parametric equations for the straight line through the point A (2, 4, 3)
and parallel to the vector d= [4, 0, -7].
6. Find the point of intersection (if it exists) of the pair of lines L and M given
by:
L: through P (4, 0, -2) and parallel to c= [2, 3, 0]
M: through Q (-1, 2, -3) and parallel to d= [9, 4, 1]
340

Chapter 8
8.1 Bearings and the trigonometric ratios

General Objectives: interpret and use the sine, cosine and tangent ratios
Specific Objectives:
1. Interpret and use three-figure bearings measured clockwise from the north
(0ᣞ to 360ᣞ).
2. Apply Pythagoras theorem and the sine, cosine and tangent ratios for acute
angles to the calculation of a side or of an angle of a right-angled triangle.
3. Solve Trigonometrically problems in two dimensions involving angles of
elevation and depression
4. Extend trigonometrical ratios to angles between 90ᣞ and 360ᣞ.
5. Solve problems using sine and cosine rules for any triangle and the formula:
1
area of triangle = 𝑎𝑏 𝑠𝑖𝑛𝐶.
2
6. Solve simple trigonometrically problems in three dimensions including
angle between a line and a plane.
A bearing is an angle, measured clockwise from the northern direction. Below, the
bearing of B from A is 025 degrees. The bearing of A from B is 205 degrees.

Note: Always start by facing north, and always move in a clockwise direction.
Let’s have a look at the next page for another example:
341

Here is an example. Imagine that you are at point B. You need to travel to point A.
On which bearing should you travel?

Three-figure bearings are an alternative to compass bearings that are much more
precise. They are measured in a special way:
 Start by facing north.
 Move clockwise to face A.
 Measure the clockwise angle between the north line and the line AB.

Fact : Airline pilots and ships' helmsmen use three-figure bearings so that they
can point their craft in exactly the right direction to safely reach their destination.

Example :
A, B and C are three ships. The bearing of A from B is 045°. The bearing of C from
A is 135° . If AB= 8km and AC= 6km, what is the bearing of B from C?
342

TanC = 8/6
=1.333Tan−1 =53.13°
so C = 53.13°
y = 180° - 135° = 45° (interior angles)
x = 360° - 53.13° - 45° (angles round a point)
= 262° (to the nearest whole number)

Example : Find the bearing of A from B?

Solution: It’s 056°


Remember : write your answer as a 3-figure bearing.

The word ‘trigonometry’ is derived from the Greek words ‘tri’ (meaning three),
‘gon’ (meaning sides) and ‘metron’ (meaning measure). In fact, trigonometry is
the study of relationships between the sides and angles of a triangle. The earliest
known work on trigonometry was recorded in Egypt and Babylon. Early
astronomers used it to find the distances of the stars and planets from the Earth.
Even today, most of the technologically advanced methods used in Engineering
and Physical Sciences are based on trigonometrical concepts. In this chapter, we
will study some ratios of the sides of a right-angled triangle with respect to its
acute angles, called trigonometric ratios of the angle. We will restrict our
discussion to acute angles only. However, these ratios can be extended to other
angles also. We will also define the trigonometric ratios for angles of measure
0° and 90°.
343

Trigonometric Ratios

Let us take a look at triangle ABC as shown above with its acute angle as ∠A. Note
the position of the side BC with respect to angle A, It faces ∠ A. We call it the side
opposite to angle A. AC is the hypotenuse of the right-angled triangle and the
side AB is a part of ∠ A. So, we call it the side adjacent to angle A.
Note that, the position of sides change when you consider angle C in place of A.
You have studied the concept of ‘ratio’ in your earlier classes. We now define
certain ratios involving the sides of a right-angled triangle, and call them
trigonometric ratios.
The trigonometric ratios of angle A in right-angled triangle ABC are defined as
follows :
1. Sine of ∠ A = (side opposite to angle A)/ Hypotenuse =BC/AC.
2. Cosine of ∠A =( side adjacent to angle A )/hypotenuse = AB/AC.
3. Tangent of ∠ A = (side opposite to angle A)/(side adjacent to angle A) A= BC/AB
4. Cosecant of ∠ A = 1/ sine of∠ A = AC/BC
5. Secant of ∠ A = 1/ cosine of ∠A =AC/AB
6. Cotangent of ∠ A = 1/ tangent of ∠A = AB/BC
344

The ratios defined above are abbreviated as sin A, cos A, tan A, cosec A, sec A and
cot A respectively. Note that the ratios cosec A, sec A and cot A are respectively,
the reciprocals of the ratios sin A, cos A and tan A.
Also, observe that: tan A = sin A/cos A
cot A = cosA/sin A
So, the trigonometric ratios of an acute angle in a right-angled triangle express
the relationship between the angle and the length of its sides.
Remark 1: Note that the symbol sin A is used as an abbreviation for ‘the sine of
the angle A’. Sin A is not the product of ‘sin’ and A. ‘sin’ separated from A has no
meaning. Similar is the condition for Cos A.
Remark 2: Since the hypotenuse is the longest side in a right-angled triangle, the
value of sin A or cos A is always less than 1.
345

Three Impotant triangles to remember:

SineƟ =O/H CosƟ=A/H TanƟ=O/A

O= SinƟ x H A= CosƟ x H O= TanƟ x A


H=O/SinƟ H=A/ CosƟ A=O/ TanƟ

Basic concepts of trigonometry :


Negative of a Function

 sin (–x) = –sin x


 cos (–x) = cos x
 tan (–x) = –tan x
 cosec (–x) = –cosec x
 sec (–x) = sec x
 cot (–x) = –cot x

Pythagorean Identities

 sin2 θ + cos 2 θ = 1
 tan2 θ + 1 = sec 2 θ
 cot 2 θ + 1 = cosec 2 θ

Using the Pythagoras Theorem :


we get the Pythagorean identities, which are
=cos2(θ) + sin2(θ)
= ( Adjacent side/ Hypotenuse)2 + (Opposite side/ Hypotenuse)2
= 1.
346

This leads to other variations. If we divide both sides by cos2 (θ) and then use the
reciprocal and quotient identities we get, 1 + tan2 (θ) = sec2 (θ). Similarly, if we
divide both sides of the equation by sin2 (θ) we get, cot2 (θ) + 1 = cosec2 (θ).

TOP CONCEPTS
1. An angle is a measure of rotation of a given ray about its initial point. The
original ray is called the initial side and the final position of the ray after rotation
is called the terminal side of the angle. The point of rotation is called the vertex .

Terminal

Initial
2. If the direction of the rotation is anticlockwise, the angle is said to be positive
and if the direction of the rotation is clockwise, then the angle is negative.
347

3. If a rotation from the initial side to terminal side is of a revolution, the angle is
said to have a measure of one degree.
4. A degree is divided into 60 minutes, and a minute is divided into 60 seconds.
One sixtieth of a degree is called a minute, written as 1’, and one sixtieth of a
minute is called a second, written as 1”.
5. Angle subtended at the Centre by an arc of length 1 unit in a unit circle is said
to have a measure of 1 radian.

6. If a point on the unit circle is on the terminal side of an angle in standard


position, then the sine of such an angle is simply the y-coordinate of the point,
and the cosine of the angle is the x-coordinate of that point.

7. All the angles with terminal side on the x-axis or y-axis are called quadrant
angles.
348

Signs of Trigonometric functions in various quadrants

In quadrant I, all the trigonometric functions are positive. In quadrant II, only sine
and cosec are positive. In quadrant III, only tan and cot are positive, in quadrant
IV, only cosine and sec functions are positive. This is depicted as follows:

1. In quadrants where Y-axis is positive (i.e. I and II), sine is positive and in
quadrants where X-axis is positive (i.e. I and IV), cosine is positive
2. A function f is said to be a periodic function if there exists a real number T>0,
such that f(x + T) = f(x) for all x. This T is the period of a function.
3. sin (2 π + x ) = sin x ,so the period of sine is 2π. Period of its reciprocal is also 2π.
4. cos (2π + x) = cos x ,so the period of cos is 2π. Period of its reciprocal is also 2π.
5. tan (π + x) = tan x, so the period of tangent and cotangent function is π.
349

6. The tan function differs from the previous two functions in two ways (i)tan is
not defined at the odd multiples of π/2 (ii) tan function is not bounded.
TRIGONOMETRICAL IMPORTANT ANGLE VALUES

Degree
Radians

N.D means ------> Not Defined

Common angle values in degrees and radians:

Angle Values in degrees and


Radians
350

Domain and range

Trigonometric Angle Behavior


351

Area of Triangle:

C a
B

Area = ½ × base × height


We can use this formula when we are given two sides and the included angle.
Area of triangle ABC = ½(ab sin C)
or, ½(ac sin B)
or, ½(bc sin A)
Example : Find the area of triangle ABC.

Solution: Area = 12.4cm²


1
The area of the triangle is = × 5.2 × 7.1 x sin42 = 12.4cm2
2

Example : Find the area of this triangle:


352

Solution: First of all we must decide what we know.


We know angle C = 25°, and sides a = 7 and b = 10.
So let's get going:
Start with: Area = (½) ab sin C
Put in the values we know: Area =½ × 7 × 10 × sin (25°)
Do some calculations: Area =35 × 0.4226
Area =14.8 to one decimal place
Exercise: Farmer Shilongo has a triangular piece of land. The length of the fence
AB is 150 m. The length of the fence BC is 231 m. The angle between fence AB
and fence BC is 123°. How much land does Farmer Shilongo own?

Solution: lengths and angles we know:


AB = c = 150 m,
BC = a = 231 m,
and angle B = 123°
So we use: Area = (½) ca sin B
Start with: Area = (½ ) ca sinB
Put in the values we know: Area = ½ × 150 × 231 × sin (123°) m2
Do some calculations: Area = 17,325 × 0.838 m2
Area = 14,530 m2
Farmer shilongo has 14,530 m2 of land.
353

The Angle between a line and plane


The angle between line PQ and the plane ABCD:
Draw a line from Q which is perpendicular to the plane ABCD and meets the plane
at X.
Join X to P.

We now have a right-angled triangle, so we can use trigonometry. The angle


between PQ and the plane ABCD is the angle we have to find.
This method can be applied to all three-dimensional (3-D) problems.
Other Method:

If in space, given the direction vector of line L


s = {l; m; n}
and equation of the plane: Ax + By + Cz + D = 0,
then the angle between this line and a plane can be found using this formula:
|A·l+B·m+C·n|
Sin φ =
.
Note: a dot sign ( ) means multiplication √𝐀² + 𝐁² + 𝐂²·√𝐥² + 𝐦² + 𝐧²
354

Example : The pyramid VABCD has a square base that is 4cm long. The height of
the pyramid is 3cm. Calculate the length of VA?

3 cm

A
O
√𝟑𝟐
𝐜𝐦
𝟐

Solution: Using Pythagoras Theorem,


AC2= 42 + 42
AC2 = 32
Therefore, AC = √32
O is the centre of the base, so AO is half of AC.
So AO = (√32 / 2)
Applying Pythagoras' theorem to the triangle AVO, we get
AV2= (√32/2)2+ 32
AV2= 32 /4 + 9
AV2 = 17
AV = √17 = 4.12cm (3 significant figures)
Example : The triangular prism PQRSTU has a length of 10cm. PTQ and SUR are
equilateral triangles with sides that are 4cm long.
355

Solution: Find the perpendicular height of triangle SUR.


U

4 cm

S 2 cm
V R

The perpendicular height of triangle SUR √12 or 3.46cm (3 s.f.).


Let V be the midpoint of SR.
Then VR = 2cm, and UVR is a right-angled triangle.
Using Pythagoras theorem:
UV2 = UR2 – VR2
UV2 = 42– 22
UV2 = 16 - 4
UV2 = 12
So the perpendicular height of triangle SUR = √12 or 3.46cm (3 s.f.)
Example : Find the angle between the line
(x – 4)/2 = (y + 2)/6 = - (z – 6)/3
and the plane x - 2y + 3z + 4 = 0?

Solution.
From the equation of the line we find the direction vector:
s = {2; 6; -3}
From the equation of the plane we find the normal vector:
q = {1; -2; 3}
356

Using the formula, we find the angle between the line and the plane
| 2 · 1 + 6 · (-2) + (-3) · 3 |
sin φ = 2 =
√2² + 6² + (– 3)² . √1² + (– 2) + 3²
According to properties of
| 2 - 12 - 9 | -19 19 modulus function, -ve answer
sin φ = = = becomes +ve.
√4 + 36 + 9 · √1 + 4 + 9 √49 · √14 7√14
So -19 will become +19
19
Answer: sin φ =
7√14

.
Note: a dot sign ( ) means multiplication

What is angle of depression?


The angle of depression is the angle between a horizontal line, from the observer
and the line of sight to an object that is below the horizontal line.
What is angle of elevation?
The angle of elevation is the angle between a horizontal line, from the observer
and the line of sight to an object that is above the horizontal line.

Example :(i) A 15m ladder leans against a wall at an angle of elevation of 60°. (a)
How high up the wall does the ladder rest? (b) How far from the wall is the base
357

of the ladder? (ii) A 50m pole has a support wire that runs from its top to the
ground with an angle of depression of 75°. (a) How far from the base of the pole
does the wire connect to the ground? (b) How much wire is used?

15m
50m

Solution:
Let the height be h and the base be b .
Then, (a) sin60°= h/15
h = 15 sin 60° = 12.99 m
(b) Cos60° = b/15
b = 15 cos60° = 7.5 m
(ii)Let the base be b and the wire be w.
Then, (a) tan 75°= 50/b
b = 50/ tan 75° =13.4 m
(b) Sin 75°= 50/w
w = 50/sin75° = 51.76 m

Example : A flat 12 meter plank rests with one end on the ground and the other
end upon a 4m ledge. (a) How far from the base of the ledge is the far end of the
plank? (b) What is the grade (i.e., angle of elevation)
358

12
4

Solution:
(a) 42 + b2 = 122
b=11.3 m
(b) Sin A = 4/12
Sin A = 1/3 =19.47°
Example : A building is 60m high. From a distance at point A on the ground, the
angle of elevation to the top of the building is 40°. From a little nearer at point B,
the angle of elevation is 70°. Find the distance from point A to point B.

60

Solution:
From the above diagram tan (70°) = 60 /𝑥 and tan (40°) = 60/d
𝒙 = 60/tan (70°) and d = 60/tan (40°)
The distance from point A to point B is d-𝒙 = 60/ tan (40°) – 60 /tan(70°) =49.667
m
359

Example :

A ladder 5m long, leaning against a vertical wall makes an angle of 65˚ with the
ground.

a) How high on the wall does the ladder reach?


b) How far is the foot of the ladder from the wall?

5m

65°
Q
R

Solution:

a) The height that the ladder reaches is PQ, Applying Triangles Ratios in the
above Figure we get:

PQ = sin 65˚ × 5 = 4.53 m

b) QR= cos 65° × 5 = 2.11 m


360

8.1.1 Problem Set 8.1


1. A, B and C are three ships. The bearing of A from B is 045ᣞ. The bearing of C
from A is 135ᣞ. If AB= 8km and AC= 6km, what is the bearing of B from C?

2. With the help of Pythagoras theorem, calculate the value of ‘x’ in the
figures given below:
a)

b)

c)
361

c)

3. In a computer catalog, a computer monitor is listed as being 19 inches. This


distance is the diagonal distance across the screen. If the screen measures
10 inches in height, what is the actual width of the screen?
4. Use sine, cosine or tangent to find the value of ‘unknown sides’ in the
triangles given below:
a)

b)

5. A person 100 meters from the base of a tree observes that the angle
between the ground and the top of the tree is 18 degrees. Estimate the
height h of the tree to the nearest tenth of a meter.
362

6. The angle of elevation of a hot air balloon, climbing vertically, changes from
25 degrees at 10:00 am to 60 degrees at 10:02 am. The point of
observation of the angle of elevation is situated 300 meters away from the
takeoff point. What is the upward speed, assumed constant, of the
balloon? Give the answer in meters per second and round to two decimal
places.
7. An airplane is approaching point A along a straight line and at a constant
altitude h. At 10:00 am, the angle of elevation of the airplane is 20o and at
10:01 it is 60o. What is the altitude h of the airplane if the speed of the
airplane is constant and equal to 600 miles/hour?

8. A bird sits on top of a lamppost. The angle of depression from the bird to
the feet of an observer standing away from the lamppost is 35 degree. The
distance from the bird to the observer is 25 meters. How tall is the
lamppost?

9. Use Heron’s Formula to find out the area of a triangle ABC if side AB = 3 cm,
BC = 2 cm, CA = 4 cm.

10.Given a triangle, with an area of 8.94 square-units, a parameter of 16 and


side lengths AB= 3 and CA = 7, what is the length of side BC?

11.Find the height of the plane:


363

Chapter 9
9.1 Vectors
General Objectives: understand and use vectors
Specific Objectives:
𝑥
1. Describe a translation using a vector represented by (𝑦) 𝑜𝑟 ⃗⃗⃗⃗⃗ 𝐴𝐵 or a. Add
and subtract vectors and multiply a vector by a scalar.
𝑥
2. Calculate the magnitude of a vector (𝑦) as √𝑥 2 + 𝑦 2 (Vectors will be
printed as ⃗⃗⃗⃗⃗ ⃗⃗⃗⃗⃗ | or |𝒂|. In their answers to
𝐴𝐵 or a and their magnitudes as |𝐴𝐵
questions candidates are expected to indicate a in some definite way e.g.,
by an arrow or by underlining thus ⃗⃗⃗⃗⃗
𝐴𝐵 or a)
3. Represent vectors by directed line segments
4. Use the sum and difference of two vectors to express given vectors in terms
of two coplanar vectors.
5. Use position vectors.
In physics, mathematics and engineering, we encounter with two important
quantities known as “Scalars and Vectors”.
A scalar quantity or simply a scalar is one that possesses only magnitude. It can
be specified by a number along with unit. In Physics, the quantities like mass,
time, density, temperature, length, volume, speed and work are examples of
scalars.
A vector quantity, or simply a vector is one that possesses both magnitude and
direction. In Physics, the quantities like displacement, velocity, acceleration,
weight, force, momentum, electric and magnetic fields are examples of vectors.
9.1.1 Geometric Interpretation of vector
A vector is represented by a directed line segment, a segment with an arrow at
one end indicating the direction of movement. Unlike a geometric ray, a directed
line segment has a specific length.
364

The direction of the vector is indicated by an arrow pointing from the tail (the
initial point) to the head (the terminal point). If the tail is at point A and the head
is at point B, the vector from A to B can be written as:

Vector Addition:

We can then add vectors by adding the 𝒙 parts and adding the y parts:
Q:

The vector (8,13) and the vector (26,7) add up to the vector (34,20)
Example: add the vectors a = (8,13) and b = (26,7)
c=a+b
365

c = (8,13) + (26,7) = (8+26,13+7) = (34,20)


Vector Subtraction:

Q: To subtract, first reverse the vector we want to subtract, then add.


Example: subtract k = (4,5) from v = (12,2)
a = v + −k
a = (12,2) + −(4,5) = (12,2) + (−4, −5) = (12−4,2−5) = (8, −3)
Multiplying a Vector by a Scalar
When we multiply a vector by a scalar it is called "scaling" a vector, because we
know how big or small the vector is.
Q: multiply the vector m = (7,3) by the scalar 3

a = 3m = (3×7,3×3) = (21,9)

It still points in the same direction, but it is 3 times longer than the previous
vector.

Magnitude of a Vector
The magnitude of a vector is shown by two vertical bars on either side of the
vector: |a|
366

or it can be written with double vertical bars (so as not to confuse it with absolute
value): ||a||
We use Pythagoras' theorem to calculate it:

|a| = √𝑥² + 𝑦²
Example: what is the magnitude of the vector b = (6,8)?

|b| = √6² + 8² = √36 + 64 = √100 = 10

Directed Line Segments and Vectors


A directed line segment is defined as an initial point, P, and a terminal point Q.
Q:
P = (2,3) and Q = (-1,4)
is equivalent to the directed line segment
"Q - P” = <-3, 1>
When we write the <> we meant that the vector has initial point at the origin and
terminal point at (-3,1). This notation is called the component form of the vector.
The length of the vector <x, y> is called the norm or magnitude.
We can find it by the formula:

<x, y>= √𝑥 2 + 𝑦 2
Length of a Vector.
Q: Solve

<-3, 1>= √−32 + 12 = √10


We also use the notation: -3i + j
Coplanar vectors
Vectors parallel to the same plane, or lie on the same plane are called coplanar
vectors.
367

It is always possible to find a plane parallel to the two random vectors. Therefore
in that, any two vectors are always coplanar.

Q: Check whether the three vectors are coplanar a = {1; 2; 3}, b = {1; 1; 1} & c =
{1; 2; 1}.
Solution: calculate a scalar triple product of vectors

1 2 3
a · [b × с] = 1 1 1 =
1 2 1

= 1·1·1 + 1·1·2 + 1·2·3 - 1·1·3 - 1·1·2 - 1·1·2 = 1 + 2 + 6 - 3 - 2 - 2 = 2


Answer: vectors are not coplanar as their scalar triple product is not zero
Q: Prove that the three vectors a = {1; 1; 1}, b = {1; 3; 1} & c = {2; 2; 2} are
coplanar.
Solution: calculate a scalar triple product of vectors
1 1 1
a · [b × с] = 1 3 1 =
2 2 2

= 1·2·3 + 1·1·2 + 1·1·2 - 1·2·3 - 1·1·2 - 1·1·2 = 6 + 2 + 2 - 6 - 2 - 2 = 0


368

Answer: vectors are coplanar as their scalar triple product is zero.


Addition of Coplanar Vector:

Subtraction of coplanar vector:

What is a Position Vector?


A vector that starts from the origin (O) is called a position vector.
In the following diagram, point A has the position vector a and point B has the
position vector b.
Example:

3
if 𝑎 = ( ), then the coordinates of A will be (3, 7).
7
8
Similarly, if 𝑏 = ( ), then the coordinates of B will be (8, 4)
4
369

9.1.2 Properties of vectors


Vectors both in the plane and in space have the following properties:
𝑢 ⃗ ( 𝐶𝑜𝑚𝑚𝑢𝑡𝑎𝑡𝑖𝑣𝑒 𝑃𝑟𝑜𝑝𝑒𝑟𝑡𝑦)
⃗ +𝑣 =𝑣+𝑢
(𝑢
⃗ + 𝑣) + 𝑤
⃗⃗ = 𝑢 ⃗⃗ ) ( 𝐴𝑠𝑠𝑜𝑐𝑖𝑎𝑡𝑖𝑣𝑒 𝑃𝑟𝑜𝑝𝑒𝑟𝑡𝑦)
⃗ + (𝑣 + 𝑤
⃗ + (−1)𝑢
𝑢 ⃗ =𝑢 ⃗ = 0 (𝐼𝑛𝑣𝑒𝑟𝑠𝑒 𝑓𝑜𝑟 𝑉𝑒𝑐𝑡𝑜𝑟 𝐴𝑑𝑑𝑖𝑡𝑖𝑜𝑛)
⃗ −𝑢
⃗ + 𝑣) = 𝑎𝑢
𝑎(𝑢 ⃗ + 𝑎𝑣 (𝐷𝑖𝑠𝑡𝑟𝑖𝑏𝑢𝑡𝑖𝑣𝑒 𝑃𝑟𝑜𝑝𝑒𝑟𝑡𝑦)
⃗ ) = (𝑎𝑏)𝑢
𝑎(𝑏𝑢 ⃗ (𝑆𝑐𝑎𝑙𝑎𝑟 𝑀𝑢𝑙𝑡𝑖𝑝𝑙𝑖𝑐𝑎𝑡𝑖𝑜𝑛)

9.1.3 Problem Set 9.1


1. For v = [1, -3] and w = [2, 5], find 𝑣 + 𝑤 ⃗⃗ , |𝑣|, 4𝑣 + 2𝑤 ⃗⃗ .
2. Find the unit vector in the same direction as the vector v = [3, -4].
3. Find the magnitude of the vector u :
u = 2i – 7j
u = [3, -4]
4. If u = 2i – 3j + k, v = 4i +6j + 2k and w = -6i - 9j - 3k then find
a) u + 2v
b) |𝑢⃗ −𝑣−𝑤 ⃗⃗ |
c) Show that u, v and w are parallel to each other.

9.2 Transformations
General Objectives: transform simple plane figures and describe these
transformations
Specific Objectives:
1. Reflect simple plane figures in horizontal and vertical lines
2. Rotate simple plane figures about the origin, vertices and midpoints of
edges of the figures, through multiples of 90ᣞ.
3. Construct given translations and enlargements of simple plane figures.
4. Recognize and describe reflections, rotations, translations and
enlargements.
5. Use the following transformations of the plane: reflection, rotation,
translation and enlargement and their combinations.
370

6. Identify and give precise descriptions of transformations connecting given


figures.

Reflection
Triangle ABC and its reflection A'B'C' have the same size and shape and are
congruent.

Translation
A translation is defined by specifying the distance and the direction of a
movement. For example, triangle ABC is translated by 2 units to the right.

Rotation : A rotation is a transformation in a plane that turns every point of a pre-


image through a specified angle and direction about a fixed point. The fixed point
is called the center of rotation. The amount of rotation is called the angle of
rotation and it is measured in degrees. Use a protractor to measure the specified
angle counterclockwise.
371

Examples of transformation geometry in the coordinate plane.

 Reflection over x-axis: T(x, y) = (x, -y)


 Reflection over y-axis: T(x, y) = (-x, y)
 Reflection over line y = x: T(x, y) = (y, x)
 Rotation 180° around the origin: T(x, y) = (-x, -y)

Q: A rotation by 90° about the origin is?

Rotation about origin:


372

Draw and describe reflection of Plane Figures in vertical and Horizontal Lines
The reflection has the same size as the original image.

Mirror Lines can be in any direction.


Imagine turning the photo at the top in different directions. The reflected image is
always the same size; it just faces the other way.

A reflection is a flip over a line

Example : Draw a reflection of the following image?


Solution:
1. Measure from the point to the mirror line (must hit the mirror line at a right
angle) .
373

2. Measure the same distance again on the other side and place a dot.

3. Then connect the new dots up!

Labels
It is common to label each corner with letters, and to use a little dash (called a
Prime) to mark each corner of the reflected image.

Here the original image is ABC and the reflected image is A'B'C'
Some Tricks:

X-Axis
When the mirror line is the x-axis :
We change each (x,y) into (x,−y)
374

Y-Axis
When the mirror line is the y-axis:
We change each (x,y) into (−x,y)

Example : Rotate line OA by 270° about the origin: O(origin, 270°)?

Solution: A rotation by 270° about the origin can be seen in the picture below in
which A is rotated to its image A'. The general rule for a rotation by 270° about
the origin is (A,B) (B, -A).
375

Draw and describe rotations of Plane Figures around a vertex or around the
midpoint of a line and through angles which are multiples of 90o.

A rotation is a transformation in which the object is rotated about a fixed point.


The direction of rotation can be clockwise or anticlockwise.

The fixed point in which the rotation takes place is called the center of rotation.
The amount of rotation made is called the angle of rotation.

Example :

For any rotation, we need to specify the center, the angle and the direction of
rotation.
Example : Determine the image of the straight line XY under an anticlockwise
rotation of 90˚ about O.
376

Solution:
Step 1: Join point X to O.
Step 2: Using a protractor, draw a line 90˚ anticlockwise from the line OX. Mark
on the line the point X such that the line of OX = OX’

Step 3: Repeat steps 1 and 2 for point Y. Join the points X and Y to form the line
XY.

Resizing
If one shape can become another by Resizing (also called dilation, contraction,
compression, enlargement or even expansion), then the shapes are Similar:

These Shapes are Similar!

There may be Turns, Flips or Slides.


Sometimes it can be hard to see if two shapes are Similar, because you may need
to turn, flip or slide one shape as well as resizing it.
377

Rotation -> Reflection -> Translation

9.2.1 Problem Set 9.2


1. Translate the following figure by (x-5, y+2).
378

2. Reflect the figure along y-axis

3. Rotate the figure below by 90ᣞ four times.

4. What combination of transformations is shown below? Either translation,


rotation or reflection. Explain your answer in proper sequence of the
events taking place in figure.
379

Chapter 10
10.1 Statistics
General Objectives: collect and manipulate statistical data and understand the
purpose of measures of central tendency
Specific Objectives:
1. Collect, classify and tabulate statistical data
2. Read, interpret and draw simple inferences from tables and statistical
diagrams
3. Construct and use bar charts, pie charts, pictograms, simple frequency
distributions and histograms with equal intervals
4. Calculate the range, mean, median and mode for individual and discrete
data and distinguish between purposes for which they are used.
5. Construct and read histograms with equal and unequal intervals (areas
proportional to frequencies and vertical axis labeled ‘frequency density’)
6. Construct and use cumulative frequency diagrams
7. Estimate and interpret median, percentiles, quartiles and interquartile
range
8. Calculate and estimate of the mean for grouped and continuous data
9. Identify the modal class from a grouped frequency distribution
Everybody collects, interprets and uses information, much of it in numerical or
statistical form in a day-to-day life. It is a common practice that people receive
large quantities of information every day through conversations, televisions,
computers, radios, and newspapers.
Classification:
The collected data, also known as raw data or ungrouped data is always in an un-
organized form and need to be organized and presented in a meaningful and
readily comprehensible form in order to facilitate further statistical analysis.
For Example, letters in the post office are classified according to their
destinations e.g., Windhoek, Tsumeb, Oshakati, Rundu, Walvis Bay etc.,
380

Types of classification:
Statistical data are classified in respect of their characteristics.
Broadly there are four basic types of classification. Namely:
a) Chronological classification
b) Geographical classification
c) Qualitative classification
d) Quantitative classification.

a) Chronological classification: In chronological classification, the collected


data are arranged according to the order of time expressed in years,
months, weeks, etc., The data is generally classified in ascending order of
time.
The estimates of birth rates in Namibia during 1970 – 76 are:

b) Geographical classification: In this type of classification, the data are


classified according to geographical region or place. For instance, the
production of wheat in different countries etc.,

c) Qualitative classification: In this type of classification, data are classified on


the basis of same attributes or quality like gender, literacy, religion,
employment etc., and such attributes cannot be measured along with a
scale. For example, if the population to be classified in respect to one
attributes e.g. gender, then we can classify it into two. Namely: that of male
and female. Similarly, they can also be classified into ‘married or ‘single’ on
the basis of another attribute ‘marital status’.
381

Further classification:

d) Quantitative classification: Quantitative classification refers to the


classification of data according to some characteristics that can be
measured such as height, weight, etc., For example the group of children
may be classified according to weight as given below.

In this type of classification there are two elements, namely (i) the variable
(i.e.) the weight in the above example, and (ii) the frequency in the number
of children. There are 50 children with a weight ranging from 5 to 10 kg,
200 children having a weight ranging between 10 to 15 kg and so on.

Tabulation:
Tabulation is the process of summarizing classified or grouped data in the
form of a table so that it is easily understood. Hence an investigator can
immediately be able to locate the desired information.
382

A table is a systematic arrangement of classified data in columns and rows.

Q: The marks awarded for an assignment set for a Grade 8 class of 20 students
were as follows:
6 7 5 7 7 8 7 6 9 7
4 10 6 8 8 9 5 6 4 8
Present this information in a frequency table.
Solution: To construct a frequency table, we proceed as follows:
Step 1:
Construct a table with three columns. The first column shows what is being
arranged in ascending order (i.e. the marks). The lowest mark is 4. So, start from
4 in the first column as shown below.

Step 2:
Go through the list of marks. The first mark in the list is 6, so put a tally mark
against 6 in the second column. The second mark in the list is 7, so put a tally
mark against 7 in the second column. The third mark in the list is 5, so put a tally
mark against 5 in the third column as shown on the next page.
383

We continue this process until all marks in the list are tallied.
Step 3:
Count the number of tally marks for each mark and write it in the third
column. The finished frequency table is as follows:

Statistical diagrams:
Statistical diagrams cover: data collection; extracting data from tables, mileage
charts and timetables; data presentation using stem and leaf diagrams, line
graphs, tally charts, pie charts and frequency tables; the statistical measures of
mean, mode, median and range; plotting scatter diagrams; lines of best fit and
finding the equation of the line of best fit.
Bar Graph:
A bar chart is the most common type of diagram, and is frequently used.
384

It represents data in terms of bars of equal width, whose height varies to


represent the size of the data. Also note that the bars can be of any width, and
sometimes it can be a thin vertical line.
A bar chart is particularly useful when one or two categories 'dominate' results.
In the diagram on the previous page, it is clear that red cars are the most popular.
 It is very clear and easy to read.
 Most people understand what is presented without having a detailed
statistical knowledge.
 It represents data expressed as actual numbers, percentages and
frequencies.
 A bar chart can represent either discrete or continuous data.
 If the data is discrete there should be a gap between the bars (as in the
diagram on the previous page).
 If the data is continuous there should be no gap between the bars.
Pie chart:
A pie chart shows data in terms of proportions of a whole group. The 'pie' is
divided into segments that represent this proportion. This is done by dividing the
angles at the center.
Favourite Sport Group of the Grade 12 Students

Pie chart showing percentages of favorite sports amongst Grade 12 Students


385

When to use a pie chart:


It is best used to present the proportions of a sample.
It is most useful where one or two results dominate the findings.
It can represent data expressed as actual numbers or percentages.
Do not use when there are a large number of categories, or where each has a
small, fairly equal share, as this can be unclear.

Pictogram:
A pictogram uses pictures or symbols to represent the number of units of data.
The pictures usually relate to the data shown.

Pictogram showing the number of pets against type of pet

When to use a pictogram:


These can be some of the most visually appealing diagrams to use.
They are often used as part of advertising campaigns.
Make sure that it is appropriate for your needs, don’t let the medium get in the
way of the message. It can be tempting to use pretty pictures but these may make
the information less clear; they may also trivialize the topic.
386

It is not always suitable for large quantities of data. You do not necessarily need
one picture per piece of data (in the diagram on the previous page, one picture
represents two pets). However, you may still have to show smaller quantities (e.g.
half a picture represents one pet) and it may be hard to read if divided further.
Some pictograms can vary by the type of symbols rather than the number. Do not
use this, unless you are confident. It should be the area of the picture that
increases in proportion to the frequencies rather than the length, otherwise this
can be misleading.
Histogram
A histogram looks similar to a bar chart, but in this case the area of the bar or
rectangle varies with the data, rather than just the length.

Histogram showing the number of people on holiday against age of people

When to use a histogram:


 It can only be used to represent continuous data.
 It can represent data expressed as actual numbers, percentages and
frequencies.
 It is only of value if the reader gets more information from a chart where
the classes are of different sizes.
387

Frequency:
Frequency is how often something occurs.
Example: Sam played football on:
Saturday Morning,
Saturday Afternoon
Thursday Afternoon
The frequency was 2 on Saturday, 1 on Thursday and 3 for the whole week.
Frequency Distribution
By counting frequency, we can make a Frequency Distribution table.
Example: Goals
Sam's team scored the following numbers of goals in recent games. Goals
scored:
2, 3, 1, 2, 1, 3, 2, 3, 4, 5, 4, 2, 2, 3
Sam puts the numbers in order, and then added up:
How often 1 occurs (2 times), how often 2 occurs (5 times), etc. and wrote them
down as a Frequency Distribution table.
388

Q: Find the mean, median, mode, and range for the following list of values:
13, 18, 13, 14, 13, 16, 14, 21, 13
The mean is the usual average, so I'll add and then divide:
(13 + 18 + 13 + 14 + 13 + 16 + 14 + 21 + 13) ÷ 9 = 15
Note that the mean, in this case, isn't a value from the original list. This is a
common result. You should not assume that your mean will be one of your original
numbers.
The median is the middle value, so first I'll have to rewrite the list in numerical
order:
13, 13, 13, 13, 14, 14, 16, 18, 21
There are nine numbers in the list, so the middle one will be:
(9 + 1) ÷ 2 = 10 ÷ 2 = 5th number:
13, 13, 13, 13, 14, 14, 16, 18, 21
So the median is 14.
The mode is the number that occurs most often. And if there’s no number in the
list repeated, then there is no mode. So in this case 13 is the mode.
The largest value in the list is 21, and the smallest is 13, so the range is 21 – 13 =
8.
Mean: 15
Median: 14
Mode: 13
Range: 8
Note: The formula for the place to find the median is "([the number of data
points] + 1) ÷ 2", but you don't have to use this formula. You can just count from
both ends of the list until you meet in the middle, if you prefer, especially if your
list is short then both ways will work.
389

Histograms
The following table shows the ages of 25 children on a school bus:

Age Frequency

5-10 6

11-15 15

16-17 4

> 17 0

If we are going to draw a histogram to represent the data, we first need to find
the class boundaries. In this case they are 5, 11, 16 and 18. The class widths are
therefore 6, 5 and 2.
The area of a histogram represents the frequency.
The areas of bars should therefore be 6, 15 and 4.

Remember that in a bar chart the height of the bar represents the frequency. It is
therefore correct to label the vertical axis 'frequency'.
However, as in a histogram, it is the area which represents the frequency.
390

So we know that Area = frequency = Frequency density x class width, hence:


Frequency density = frequency ÷ class width
Cumulative frequency
The cumulative frequency is obtained by adding up the frequencies as you go
along, to give a 'running total'.
Drawing a cumulative frequency diagram
The table shows the lengths (in cm) of 32 cucumbers.
Before drawing a cumulative frequency diagram, we need to work out the
cumulative frequencies. This is done by adding the frequencies in turn.

Length Frequency Cumulative Frequency

21-24 3 3

25-28 7 10 (= 3 + 7)

29-32 12 22 (= 3 + 7 + 12)

33-36 6 28 (= 3 + 7 + 12 + 6)

37-40 4 32 (= 3 + 7 + 12 + 6 + 4)

The points are plotted at the upper class boundary. In this example, the upper
class boundaries are 24.5, 28.5, 32.5, 36.5 and 40.5. Cumulative frequency is
plotted on the vertical axis.
391

There are no values below 20.5cm.


Cumulative frequency graphs are always plotted using the highest value in each
group of data, (because the table gives you the total that are less than the upper
boundary) and the cumulative frequency is always plotted up a graph, as
frequency is plotted upwards.
Cumulative frequency diagrams usually have a characteristic of S-shape, called an
ogive.
Finding the median and quartiles
When looking at a cumulative frequency curve, you will need to know how to find
its median, lower and upper quartiles, and the interquartile range.
By drawing horizontal lines to represent 1/4 of the total frequency, 1/2 of the total
frequency and 3/4 of the total frequency, we can read estimates of the lower
quartile, median and upper quartile from the horizontal axis.
392

Quartiles are associated with quarters. The interquartile range is the difference
between the lower and upper quartile.
From these values, we can also estimate the interquartile range: 33 - 28 = 5.
Remember to use the total frequency, not the maximum value, on the vertical
axis. The values are always read from the horizontal axis.

The mean
We already know how to find the mean from a frequency table. Finding the mean
for grouped or continuous data is very similar.
The grouped frequency table shows the number of CDs bought by a class of
children in the past year.
393

Number of Frequency
CDs (f)

0-4 10

5-9 12

10-14 6

15-19 2

>19 0

 We know that 10 children have bought either 0, 1, 2, 3 or 4 CDs, but we do


not know exactly how many CDs each child bought.
 If we assumed that each child bought 4 CDs, it is likely that our estimate of
the mean would be too big.
 If we assumed that each child bought 0 CDs, it is likely that our estimate
would be too small.
 It therefore seems sensible to use the mid-point of the group and assume
that each child bought 2 CDs.
Finding the mid-points of the other groups, we get:

Number of CDs f Mid-point, x fx

0-4 10 2 20

5-9 12 7 84

10-14 6 12 72

15-19 2 17 34

>19 0 - 0
394

The mean is
Remember: This is only an estimate of the mean.
The median
As explained previously, the median is the middle value when the values are
arranged in order of size.
As the data has been grouped, we cannot find an exact value for the median, but
we can find the class which contains the median.

Number of CDs Frequency (f)

0-4 10

5-9 12

10-14 6

15-19 2

>19 0

There are 30 children, so we are looking for the class which contains the (30 + 1) ÷
1
2 = 15 th value. The median is therefore within the 5-9 class.
2

The mode
The mode is the most common value.
We cannot find an exact value for the mode, and therefore give the modal class
as: The modal class is 5-9.
Mean and modal class
Calculating the mean and modal class for grouped data is very similar to finding
the mean from an ungrouped frequency table, except that you do not have all
the information about the data within the groups so you can only estimate the
mean.
395

Calculating the mean from grouped data:


Example: This table shows the weights of children in a class.
Using this information:
a) Estimate the mean weight
b) Find the modal class

Mass (m) kg Frequency

30 ≤ m < 40 7

40 ≤ m < 50 6

50 ≤ m < 60 8

60 ≤ m < 70 4

Solution:
To estimate the mean weight, you know that 7 children are between 30kg and
40kg, but you don't know exactly how much they weigh, so assume that they all
weigh 35kg (the midpoint of the group).
Do the same for all the other groups:

Mass (m) kg Midpoint Frequency Midpoint × Frequency

30 ≤ m < 40 35 7 245

40 ≤ m < 50 45 6 270

50 ≤ m < 60 55 8 440

60 ≤ m < 70 65 4 260

Total 25 1215

a) Estimate of mean = 1215 ÷ 25 = 48.6 kg


396

b) The modal class is the class that has the highest frequency. In this case the
modal class is 50 ≤ m < 60

10.1.1 Collection of data


Statistics is the science of data. It encompasses set of techniques to extract useful
information from data. Statistics is a ‘number’ that describes a property of the
data. By statistics, we can study the variability and can control it. Variability is
described by their location (mean or average) and variance (scatter) of the data.
For reaching at any conclusion or in decision making process, there are two major
steps:
a) Collect data
b) Test data properly
Data is collected in one of three ways:
1. Observational Study: It observes the process or population during a period
of routine operation.
2. Designed Experiment: Purposeful changes are made in controllable
variables known as factors of the system and observing the resulting
system output and then a decision is usually made.
3. Retrospective Study: It uses either all or a sample of the historical process
data from some period of time.
The data are almost always in form of a sample which is selected from some
population. A population is the entire collection of objects or outcomes about
which data are collected. A sample is a subset of the population containing the
observed objects or the outcomes and the resulting data. A simple random
sample of size ‘n’ is a sample that has been selected from a population in such a
way that each possible sample of size ‘n’ has an equally likely chance of being
selected.
397

Sample Variance and Sample Standard Deviation


𝐼𝑓 𝑛 𝑜𝑏𝑠𝑒𝑟𝑣𝑎𝑡𝑖𝑜𝑛𝑠 𝑖𝑛 𝑎 𝑠𝑎𝑚𝑝𝑙𝑒 𝑎𝑟𝑒 𝑑𝑒𝑛𝑜𝑡𝑒𝑑 𝑏𝑦 𝑥1, 𝑥2, … … . , 𝑥𝑛 , 𝑡ℎ𝑒𝑛 𝑡ℎ𝑒 𝑠𝑎𝑚𝑝𝑙𝑒 𝑣𝑎𝑟𝑖𝑎𝑛𝑐𝑒 𝑖𝑠:
𝑛
2
(𝑥𝑖 − 𝑥̅ )2
𝑠 =∑
𝑛−1
𝑖=1

The sample standard deviation, s, is the positive square root of the sample variance.
Sample Mean
𝐼𝑓 𝑛 𝑜𝑏𝑠𝑒𝑟𝑣𝑎𝑡𝑖𝑜𝑛𝑠 𝑖𝑛 𝑎 𝑠𝑎𝑚𝑝𝑙𝑒 𝑎𝑟𝑒 𝑑𝑒𝑛𝑜𝑡𝑒𝑑 𝑏𝑦 𝑥1, 𝑥2, … … . , 𝑥𝑛 , 𝑡ℎ𝑒𝑛 𝑡ℎ𝑒 𝑠𝑎𝑚𝑝𝑙𝑒 𝑚𝑒𝑎𝑛 𝑖𝑠:
𝑥1 + 𝑥2 + ⋯ + 𝑥𝑛
𝑥̅ =
𝑛
10.1.2 Summarizing and Presentation of data

10.1.2.1 Scatter Diagram


The scatter diagram graphs pairs of numerical data, with one variable on each
axis, to look for a relationship between them. If the variables are correlated, the
points will fall along a line or curve. A scatter plot has been shown in figure below:
pull Strength

Wire length

10.1.2.2 Dot Diagram


A dot diagram is a statistical chart consisting of data points plotted on a fairly
simple scale, typically using filled in circles. Below is an example of dot diagram.
398

Acetone concentration (g/l)

10.1.2.3 Time Series Plot


Data are collected over time and the reason of any variation is not depicted by
dot diagrams. So time series plot fulfills the purpose and depict and change if it
occurs.
Acetone concentration (g/l)

Observation number (taken hourly)

10.1.2.4 Stem-Leaf Plot

Steps for constructing a Stem-and-Leaf Diagram


1. Divide each number 𝑥𝑖 into two parts: a stem, consisting of one or more of
the leading digits and a leaf consisting of the remaining digit.
2. List the stem values in a vertical column
3. Record the leaf for each observation beside its stem.
4. Write the units for stems and leaves on the display
Median: the median is a measure of central tendency that divides the data into
two equal parts, half below the median and half above. If the number of
observations is even the median is halfway between the two central values.
Range: The range is a measure of variability that can be easily computed from the
ordered stem-and-leaf display. It is the maximum minus minimum measurement.
Quartiles: We can also divide the data into more than two parts. When an
ordered set of data is divided into four equal parts, the division points are called
399

quartiles. The first or lower quartile is a value that has approximately 25% of the
observations below it and approximately 75% of the observations above. The
second quartile has approximately 50% of the observations below its value. The
second quartile is exactly equal to the median. The third or upper quartile has
approximately 75% of the observations below its value.
Interquartile Range (IQR): It is the difference between the upper and lower
quartiles and it is sometimes used as a measure of variability.
Percentile: In general 100kth percentile is the data value such that approximately
100k% of the observations are at or below this value and approximately 100(1-
k)% of them are above it.

10.1.3 Problem Set 10.1


1. A person measures the current flowing through a simple circuit several
times and observes that the current measurements are different each time.
Can we view these measurements as simple random experiment?
2. John got his grade points for third semester as shown below. Construct a
stem and leaf plot for this data.
2.3, 2.5, 2.5, 2.7, 2.8, 3.2, 3.6, 3.6, 4.5, 5.0
3. Complete a stem-and-leaf plot for the marks obtained by Marry
73, 42, 67, 78, 99, 84, 91, 82, 86, 94

4. For the data given below, Calculate:


35, 35, 37, 40, 43, 56, 58, 58, 60

a) IQR
b) Lower Quartile
400

c) Upper Quartile
d) Median
5. For the sample shown below, find the 25th sample percentile:

10.2 Probability
General Objectives: understand and use probability
Specific Objectives:
1. Calculate the probability of a single event as either a fraction or a decimal
(not ratio)
2. Understand and use the probability scale from 0 to 1
3. Understand that (the probability of an event occurring) = 1-(the probability
of the event not occurring)
4. Understand probability in practice e.g., relative frequency
5. Calculate the probability of simple combined events, using possibility
diagrams and tree diagrams where appropriate (in possibility diagrams
outcomes will be represented by points on a grid and in tree diagrams
outcomes will be written at the end of branches and probabilities by the
side of the branches)
10.2.1 Random Variable
A random variable is a numerical variable whose measured value can change from
one replicate of the experiment to another.
Discrete and Continuous Random Variables: A discrete random variable is a
random variable with a finite set of real numbers for its range. A continuous
random variable is a random variable with an interval of real numbers for its
range.
A random variable is used to describe a measurement. Probability is used to
quantify the likelihood or chance that a measurement falls within some set of
values.
401

10.2.2 Probability of Single Events


Independent events appear quite frequently in our everyday life. Let’s learn how
to calculate their probability. There is a very simple formula to calculate the
probability of an event:

Let’s look at an example. We have a bag with 9 marbles (3 red, 4 yellow, 2 green)
and we want to find the probability that we will get a red marble when taking one
out of the bag. The number of appropriate outcomes is 3 as there are three red
marbles. The total number of outcomes is 9 as there are nine marbles in the bag.
Consequently, the probability is 3/9 or 1/3. We can also write probability as
percentage or decimals: 33.333...% or 0.333.
Solving the Probability of a single event using Tree Diagrams We can also
use tree diagrams to represent probability!

Describing probabili es
We o en make judgments as to whether an event will take place, and use words
to describe how probable that event is. For example, we might say that it is likely
that the sun will come up tomorrow, or that it is impossible to find somebody
who is more than 3m tall.
402

10.2.3 The probability scale


Math’s uses numbers to describe probabili es. Probabili es can be wri en as
frac ons, decimals or percentages. You can also use a probability scale, star ng at
0 (impossible) and ending at 1 (certain).
Here are some events placed on the probability scale.

Defini ons and Nota on

§ Two events are mutually exclusive or disjoint if they cannot occur at the
same me.
§ The probability that event A occurs, given that event B has occurred, is
called a condi onal probability. The condi onal probability of event A,
given event B, is denoted by the symbol P (A|B).
§ The complement of an event is the event not occurring. The probability
that event A will not occur is denoted by P (A').
403

§ The probability that events A and B both occur is the probability of the
intersec on of A and B. The probability of the intersec on of events A and
B is denoted by P (A ∩ B). If events A and B are mutually exclusive, P (A ∩
B) = 0.
§ The probability that events A or B occur is the probability of the union of A
and B. The probability of the union of events A and B is denoted by P(A B)
§ If the occurrence of event A changes the probability of event B, then events
A and B are dependent. On the other hand, if the occurrence of event A
does not change the probability of event B, then events A and B are
independent.

10.2.4 Rule of Subtrac on

· The probability of an event ranges from 0 to 1.


· The sum of probabili es of all possible events equals 1.
· The rule of subtrac on follows directly from these proper es.

Rule of Subtrac on of the probability that event A will occur is equal to 1 minus
the probability that event A will not occur.

P (A) = 1 – P (A')
Example: The probability that Selma will graduate from the University of Namibia
is 0.80. What is the probability that Selma will not graduate from the University of
Namibia? Based on the rule of subtrac on, the probability that Selma will not
graduate is 1.00 - 0.80 or 0.20.
404

10.2.5 Rule of Mul plica on

The probability that events A and B both occur is equal to the probability that
event A occurs mes the probability that event B occurs, given that A has
occurred.

P (A ∩ B) = P (A) P (B|A)
Example
A Jar contains 6 red marbles and 4 black marbles. Two marbles are drawn
without replacement from the Jar. What is the probability that both of the
marbles are black?
Solu on: Let A = the event that the first marble is black; and let B = the event
that the second marble is black. We know the following:
§ In the beginning, there are 10 marbles in the Jar, 4 of which are black.
Therefore, P (A) = 4/10.
§ A er the first selec on, there are 9 marbles in the Jar, 3 of which are
black. Therefore, P (B|A) = 3/9.
Therefore, based on the rule of mul plica on:

P (A ∩ B) = P (A) P (B|A)
P (A ∩ B) = (4/10) × (3/9) = 12/90 = 2/15

10.2.6 Rule of Addi on

The probability that event A or event B occurs equals to the probability that
event A occurs plus the probability that event B occurs minus the probability
that both events A and B occur.

P (A ∪ B) = P(A) + P(B) - P(A ∩ B)


405

Note: Invoking the fact that P ( A ∩ B) = P (A) P (B | A), the Addi on Rule can also
be expressed as:
P (A B) = P (A) + P (B) – P (A)P( B | A )

Example

A student goes to the library. The probability that she checks out (a) a work of
fic on is 0.40, (b) a work of non-fic on is 0.30, and (c) both fic on and non-fic on
is 0.20. What is the probability that the student checks out a work of fic on, non-
fic on, or both?
Solu on: Let F = the event that the student checks out fic on; and let N = the event
that the student checks out non-fic on. Then, based on the rule of addi on:

P (F N) = P (F) + P (N) – P (F ∩ N)
P (F N) = 0.40 + 0.30 - 0.20 = 0.50

10.2.7 Rela ve Frequency

A frequency is the number that tells how many mes a data or an event is repea ng
in an experiment. It is generally represented graphically in form of histograms.
There are two different types of frequencies:
1) Cumula ve Frequency: It is the total of all the absolute frequencies of all
the events that are below a point in each list of events that is formulated in an
order.
2) Rela ve Frequency: It refers to the absolute frequency of an event that is
normalized by the total number of the given events.
406

Rela ve frequency is used in probability when it is not easy to evaluate the


probability of certain events by looking at their situa ons. For example, the result
of any match for win, lose or draw cannot be determined as all outcomes are
equally likely, but one can es mate the probability by looking at the previous win
or lose of matches and the way the game is proceeding so far.

Example: 30 students in a class did a survey of their favourite movie series and
recorded the results as follows:
Movie series Frequency
Twilight 10
Harry Po er 6
Narnia 2
High school Musical 9
Pirates of the Caribbean 3

What was the rela ve frequency for High School Musical?


Solu on: 9 students out of 30 said their favourite movie was High School Musical.
Therefore the rela ve frequency was 9/30 = 3/10 = 0.3
Example: A dice was thrown 60 mes with the following results:

Score Frequency
1 8
2 11
3 13
4 7
5 9
6 12

What was the rela ve frequency for a score of six?


407

Solu on:
Six occurred 12 mes out of 60 throws altogether.
Therefore, the rela ve frequency for a score of six = 12/60 = 1/5 = 0.2

10.2.8 Tree diagrams (probability of simple combined events)

Tree diagrams allow us to see all the possible outcomes of an event and calculate
their probability. Each branch in a tree diagram represents a possible outcome.
If two events are independent, the outcome of one has no effect on the outcome
of the other. For example, if we toss two coins, ge ng heads with the first coin
will not affect the probability of ge ng heads with the second.
A tree diagram which represent a coin being tossed three mes looks like this:

From the tree diagram, we can see that there are eight possible outcomes. To
find out the probability of an outcome, we need to look at all the available paths
(set of branches).
The sum of the probabili es for any set of branches is always 1.
408

Also, note that in a tree diagram, to find a probability of an outcome we mul ply
along the branches and add ver cally.
The probability of three heads is:

P (H H H) = 1/2 × 1/2 × 1/2 = 1/8


P (2 Heads and a Tail) = P (H H T) + P (H T H) + P (T H H)
= 1/2 × 1/2 × 1/2 + 1/2 × 1/2 × 1/2 + 1/2 × 1/2 × 1/2

= 1/8 + 1/8 + 1/8

= 3/8
Example:
A bag contains 3 black balls and 5 white balls. Paul picks a ball at random from the
bag and replaces it back in the bag. He mixes the balls in the bag and then picks
another ball at random from the bag.

a) Construct a probability tree of the problem.


b) Calculate the probability that Paul picks:
i) Two black balls
ii) A black ball in his second draw
Solu on:

a) Check that the probabili es in the last column add up to 1.


409

b) i) To find the probability of ge ng two black balls, first locate the B branch
and then follow the second B branch. Since these are independent events we
can mul ply the probability of each branch.

ii) There are two outcomes where the second ball can be black. Either (B, B) or
(W, B)

From the probability tree diagram, we get:


P (second ball black)
= P (B, B) or P (W, B)
= P (B, B) + P (W, B)
9 15
= +
64 64
24 3
= =8
64
410

When an experiment is more complex, construc ng a probability diagram


or possibility diagram may be helpful in construc ng the sample space.
Example:
The diagram shows two spinners, each of which is divided into 4 equal sectors.
Each spinner has a pointer which, when spun, is equally likely to come to rest in
any of the four equal sectors.

4 1 4 1

3 2 3 2

In a game, each pointer is spun once.


Find the probability that
a) The pointers will stop at the same number.
b) The first spinner shows the larger number.
Solu on:
Construct the probability diagram. Each dot represents a possible outcome
according to the coordinates.

Two spinners ge ng
the same number s
4
Second 3 First spinner shows a
spinner bigger number
2
1 This dot represents the
ordered pair(4,1)
1 2 3 4
First spinner
411

a) Let A = Event of ge ng the same number on the two spinners.

From the probability diagram, n (A) = 4, n(S) = 16


n (A) 4 1
P(A)= = =
n (S) 16 4

b) Let B = Event of the first spinner shows the bigger number.


From the probability diagram, n (B) = 6
n (B) 6 3
P(B) = = =
n (S) 16 8

10.2.9 Probability Proper es


1. P(X ∈ E) = 1, where R is the set of real numbers.
2. 0 ≤ P(X ∈ E) ≤ 1 for any set E.
3. if E1 , E2, … . E k are mutually exclusive sets,
4. P ( X ∈ E1 U E2 U … . . U Ek)=P(X ∈ E1) + ⋯ + p(X ∈ E k)
Cumula ve Distribu on Func on of a Con nuous random Variable
The Cumula ve Distribu on Func on (or cdf) of a Con nuous random Variable
X with probability density func on f (x) is

10.3 Problem Set 10.2


1. A dice is rolled, find the probability that an even number is obtained.
2. Two dice are rolled, find the probability that the sum is
a) equal to 1
b) equal to 4
c) less than 13
3. A jar contains 3 red marbles, 7 green marbles and 10 white marbles. If a
marble is drawn from the jar at random, what is the probability that this
marble is white?
4. Consider 2 events A and B with P (A) = 0.4 and P (A U B) = 0.8. Find
412

a) P(Ac ∩ B)
b) P(B) if A and B are mutually exclusive
c) P(B) if A and B are mutually independent
5. Let the con nuous random variable X denote the current measured in a thin
copper wire in milli-amperes. Assume that the range of X is [0, 20mA], and
assume that the probability density func on of X isf(x) = 0.05 for 0 ≤
x ≤ 20. What is the probability that a current measurement is less than 10
milli-amperes?

Chapter 11
11.1 Matrices
General objectives: develop an understanding of matrices and basic
operations
Specific objectives:
1. Represent information in the form of a matrix of any order and interpret the
data in a given matrix.
2. Solve problems involving the calculation of the sum and product of two 2 × 2
matrices and interpret the results.
3. Calculate the product of a scalar quantity and a matrix.
4. Use the algebra of 2 × 2 matrices (including the zero and identity matrix).
5. Calculate the determinant and inverse of a non-singular 2 × 2 matrix.
6. Solve simultaneous linear equations by matrix method
Matrices are incredibly useful things that crop up in many different applied areas.
For now, you'll probably only do some elementary manipulations with matrices,
and then you'll move on to the next topic.
We can write any system of equations as a matrix.
413

Let’s take an example:


𝑥 + 2𝑦 + 3𝑧 = −7
{ 2𝑥 − 3𝑦 − 5𝑧 = 9
−6𝑥 − 8𝑦 + 𝑧 = −22

To express this system in matrix form:


 Write all the coefficients in one matrix first (It’s coefficient matrix).
 Multiply this matrix with the variables of the system set up in another
matrix (It’s variable matrix).
 Insert the answers on the other side of the = sign in another matrix (It’s
the answer matrix).
The setup appears as follows:
1 2 3 𝑥 −7
[ 2 −3 −5] [𝑦] = [ 9 ]
−6 −8 1 𝑧 −22

An m x n ( "m by n" ) matrix is a rectangular array of numbers arranged in m rows


(horizontal lines) and n columns (vertical lines).
11.1.1 Addition of two Matrices
Only matrices of the same order (same number of rows and same number of
columns) may be added by adding corresponding elements.

Denote the sum of two matrices A and B (of the same dimensions) by C = A + B..
The sum is defined by adding entries with the same indices.

𝑐𝑖 𝑗≡𝑎𝑖+𝑏
𝑖𝑗

Example:
1 2 5 6 1+5 2+6 6 8
[ ]+[ ]=[ ]= [ ]
3 4 7 8 3+7 4+8 10 12
414

Example:
2 4 3 2 1 3
Matrix form [3 1] | Matrix form [1 1] | Matrix form [2 3]
4 5 5 3 4 6

Solution: Simply add the values by their positions.

2 4 3 2 1 3 2 +3 + 1 4 +2 + 3 6 9
[3 1] + [1 1] + [2 3] = [3 +1 + 2 1 +1 + 3 ]=[ 6 5]
4 5 5 3 4 6 4 +5 + 4 5 +3 + 6 13 14

11.1.2 Multiplication of Matrices


We can only multiply matrices if the number of columns in the first matrix is the
same as the number of rows in the second matrix.
 Multiplying a 2 × 3 matrix by a 3 × 4 matrix is possible and it gives a 2 × 4
matrix as the answer.
 Multiplying a 7 × 1 matrix by a 1 × 2 matrix is okay; it gives a 7 × 2 matrix
 A 4 × 3 matrix times a 2 × 3 matrix is NOT possible.
So, the result of multiplying our 2 matrices is as follows:

𝑢 𝑣
𝑎 𝑏 𝑐 𝑤 𝑎𝑢 + 𝑏𝑤 + 𝑐𝑦 𝑎𝑣 + 𝑏𝑥 + 𝑐𝑧
[ ][ 𝑥] = [ ]
𝑑 𝑒 𝑓 𝑦 𝑧 𝑑𝑢 + 𝑒𝑤 + 𝑓𝑦 𝑑𝑣 + 𝑒𝑥 + 𝑓𝑧

11.1.2.1 Multiplying 2 × 2 Matrices


The process is the same for any size matrix. We multiply across rows of the first
matrix and down columns of the second matrix, element by element. We then
add the products:
415

a b e f ae + bg af + bh
[ ][ ]=[ ]
c d g h ce + dg cf + dh
Here, we multiply a 2 × 2 matrix by a 2 × 2 matrix and we get a 2 × 2 matrix as the
result.

Example: Write down the following equations into matrix form.


−3𝒙 + y = 1
6𝒙 − 3y = −4
This can be written as:
−3 1 𝑥 1
[ ] [𝑦] = [ ]
6 −3 −4

11.1.2.2 Scalar multiplication


Just take a regular number (called a "scalar") and multiply it on every entry in the
matrix.
Example: For the following matrix A, find 2A and –1A.
1 2
𝐴=[ ]
3 4
To do the first scalar multiplication to find 2A, Just multiply 2 on every entry in the
matrix:
1 2 2.1 2.2 2 4
2[ ]=[ ]=[ ]
3 4 2.3 2.4 6 8
To find –1A, it works the same way:
1 2 −1.1 −1.2 −1 −2
−1𝐴 = −1. [ ]=[ ]=[ ]
3 4 −1.3 −1.4 −3 −4

11.1.2.3 Zero Matrix:


Zero matrix is a matrix where all elements are zero.
416

0 0 0
0 0 0
[ ] [ ] [0 0 0 ]
0 0 0
0 0 0
11.1.2.4 Identity matrix
The Identity Matrix In is a n × n square matrix where main diagonal of 1’s and all
other elements are 0’s.
1 0 0
1 0
𝐼2 = [ ] 𝐼3 = [0 1 0]
0 1
0 0 1
If A is a M × N Matrix, then,
Im A=A and AIn =A

If A is a N × N Matrix, then,
AIn = InA = A

4 −6
Example: let 𝑝 = [ ]
−2 8
5
a) Find 2P, b) Find P², c) find Q when P× 𝑄 = [ ]
0

Solutions:
(a) When we have to multiply a matrix by a scalar (number), we need to just
multiply all elements in the matrix by that number.
2 × 4 2 × −6 8 −12
2𝑃 = 2 [ ]=[ ]
2 × −2 2 × 8 −4 16
(b) When we square P, we just multiply it by same matrix itself.

4 −6 4 −6 (4 × 4) + (−6 × −2) (4 × −6) + (−6 × 8) 28 −72


𝑝2 = [ ]×[ ]=[ ]=[ ]
−2 8 −2 8 (−2 × 4) + (8 × −2) (−2 × −6) + (8 × 8) −24 76
417

(c) Since,
4 −6 5
[ ]×𝑄 =[ ]
−2 8 0
we have,
4 −6 −1 5
𝑄=[ ] ×[ ]
−2 8 0
Let’s use our calculator to put P in [A] and [50] in [B]. Then,
2
[𝐴]−1 × [𝐵] = [ ]
.5

11.1.3 Non-singular Matrix


It is a square matrix that is not singular, i.e., one that has a matrix inverse. Non-
singular matrices are sometimes also called regular matrices.
A square matrix is non-singular if its determinant is non-zero.
The following table gives the numbers of non-singular matrices for certain matrix
classes.
Non-singular 2 × 2 Matrix example:
0 1 0 1 1 0 1 0 1 1 1 1
[ ],[ ],[ ],[ ],[ ],[ ].
1 0 1 1 0 1 1 1 0 1 1 0

11.1.3.1 Inverse of a Matrix


The multiplicative inverse of a square matrix is called its inverse matrix. If a
matrix A has an inverse, then A×A is said to be non-singular or invertible.
A singular matrix does not have an inverse.
To find the inverse of a square matrix A, you need to find a matrix A -1 such that
the product of A and A -1 is the identity matrix.
In other words, for every square matrix A which is non-singular there exist an
inverse matrix, with the property that:
418

𝐴𝐴−1 = 𝐴−1 𝐴 = 𝐼
Example: Find A-1, if it exists. If A-1 does not exist, write singular;
1 2
𝐴=[ ]
1 1
Step 1:
Write the doubly augmented matrix [𝐴|𝐼𝑛 ] .
1 2 1 0
[𝐴|𝐼] = [ ]
1 1 0 1
Step 2:
Apply elementary row operations to write the matrix in reduced row-echelon
form.
1 2 1 0
[ ] 𝑅 = 𝑅1 − 𝑅2
0 1 1 −1 2
1 0 −1 2
[ ] 𝑅 = −2𝑅2 + 𝑅1
0 1 1 −1 1
1 0 −1 2 −1
[ ] = [𝐼 |𝐴 ]
0 1 1 −1
The system has a solution.
Therefore, A is invertible and

−1 2
𝐴−1 = [ ]
1 −1
11.1.3.2 Simultaneous equations
This are system of equations of the form:
a𝑥 + by = h
c𝑥 + dy = k

Simultaneous equations can also be solved using matrices.


Example: Given the matrix equation AY = B, find the matrix Y?
If we multiply each side of the equation by A-1 (inverse of matrix A), we get
419

A-1A Y = A-1B
I Y = A -1B (AA -1= I, where I is the identity matrix)
Y = A -1B (IY = Y, any matrix multiply with the identity matrix will be unchanged)

Example: Using matrices, calculate the values of 𝒙 and y for the following
simultaneous equations:

2𝑥 – 2y – 3 = 0
8 y = 7𝑥 + 2
Solution:
Step 1: Write the equations in the form a𝑥 + by = c
2𝑥 – 2y – 3 = 0 ⇒ 2𝑥 – 2y = 3

8y = 7𝑥 + 2 ⇒ 7𝑥 – 8y = –2
Step 2: Write the equations in matrix form.
Coefficient of first equation 2 −2 𝑥 3 Constant of first equation
[ ] [𝑦] = [ ] Constant of 2nd equation
Coefficient of 2nd equation  7 −8 −2

Step 3: Find the inverse of the 2 × 2 matrix.


Determinant = (2 × –8) – (–2 × 7) = – 2
1 −8 2 4 −1
Inverse = − [ ]=[ ]
2 −7 2 3.5 −1
Step 4: Multiply both sides of the matrix equations with the inverse
4 −1 2 −2 𝑥 4 −1 3
[ ][ ] [𝑦 ] = [ ][ ]
3.5 −1 7 −8 3.5 −1 −2
1 0 𝑥 14
[ ] [𝑦 ] = [ ]
0 1 12.5
So, 𝑥 = 14 and y = 12.5
420

11.1.3.2 Problem Set 11.1


1.

2.

3.

4.

5.
421

6.
Write the following as a system of linear equations without matrices.

7.

8.
Write this system as a matrix equation of the form AX = B and solve using X = A-1B.
5x1 + 2x2 = 6
12x1 + 5x2 = 2
What is x1?
9.
Find the value of the determinant of A.

10.
Given that the value of determinant A is -12, what is the value of determinant B?
422

Chapter 12
12.1 Set Language and Notations
General objectives: understand the use of set language and notations
Specific objectives:
1. Define sets by listing and describing and by using set builder notation e.g.
A = {𝑥 : 𝑥 is a natural number}
B = {(𝑥, y): y = mx + c}
C = {𝑥 : a ≤ x ≤ b}
D = {a, b, c}
2. Use and interpret the following set notations:
A∪B union of A and B
A∩B intersection of A and B
n(A) number of elements in set A
∈ “…is an element of …”
∈ “…is not an element …”
Aꞌ complement of set A
Ø the empty set

or U universal set
A B A is a subset of B
A B A is a proper subset of B
A ⊈ B A is not a subset of B
A B A is not a proper subset of B
3. Use sets and Venn diagrams to solve problems involving not more than three
subsets of the universal set.
423

A set is a group/collection of objects or numbers. Sets are usually symbolized by


uppercase, italicized, boldface letters such as A, B, S, or Z. Each object or number
in a set is called a member or element of the set.
Examples include the set of all books in the world, the set of all mangoes on a
tree, and the set of all irrational numbers between 0 and 1.
Example: K = {5, 6, 7, 8, 9, 10}
12.1.1 Set-Builder Notation
A shorthand used to write sets often sets with an infinite number
of elements.
Note:

The set {𝑥 : 𝑥 > 0} is read like, "the set of all 𝑥 such that 𝑥 is greater than 0." It
is read aloud exactly the same way when the colon: is replaced by the vertical
line | as in {x | 𝑥 > 0}.
Examples:

{𝑥: 𝑥 ≠ 3} The set of all real


numbers except 3

{𝑥 | 𝑥 < 5} The set of all real


numbers less than 5

{𝑥 2 | 𝑥 is a The set of all real


real number} numbers greater than
or equal to 0

{2n + 1: n is an The set of all odd


integer} integers (e.g. ..., -3, -, 1,
3, 5,...).
424

Examples of sets notations:


Symbol Symbol Name Meaning/Definition Example

Set A collection of A={3,7,9,14}


{} elements B={9,14,28}

1: A∪B (union)
Objects that belong to set A or set B.
A ∪ B = {3,7,9,14,28}.
2: A∩B (intersection)
Objects that belong to set A and set B
A ∩ B = {9,14}.
3: n(A) , n(B)
number of elements in set A=4 and set B=3.
4: a∈A “Element of”
Set membership, A= {3,9,14}, 3∈ A.
5: x∉A “Not element of”
No set membership A= {3,9,14}, 1 ∉ A.
6: A’ “Complement”
All the objects that do not belong to set A.
7: Ø “Empty set”
Ø = {} A = Ø
8: {U} “Universal set”
Set of all possible values.
425

9: A⊆B “Subset”
A is a subset of B. Set A is included in set B. {9,14,28} ⊆ {9,14,28}
10: A⊂B “Proper subset / strict subset”
A is a subset of B, but A is not equal to B. {9,14} ⊂ {9,14,28}
11: A⊄B “Not subset”
Set A is not a subset of set B {9,66} ⊄ {9,14,28}
12: A⊇B “Superset”
A is a superset of B. set A includes set B {9,14,28} ⊇ {9,14,28}
13: A⊃B “Proper superset / strict superset”
A is a superset of B, but B is not equal to A. {9,14,28} ⊃ {9,14}
14: A⊅B “Not superset”
set A is not a superset of set B {9,14,28} ⊅ {9,66}

More Details on set theory:

Set of natural numbers = {1,2,3,…..}


Set of whole numbers = {0,1,2,3,…..}
Each object is called an element of the set.
The set that contains all the elements of a given collection is called the universal
set and is represented by the symbol ‘µ’, pronounced as ‘mu’.
For two sets A and B,
 n(AᴜB) is the number of elements present in either of the sets A or B.
 n(A∩B) is the number of elements present in both the sets A and B.
 n(AᴜB) = n(A) + (n(B) – n(A∩B)
426

For three sets A, B and C,


 n(AᴜBᴜC) = n(A) + n(B) + n(C) – n(A∩B) – n(B∩C) – n(C∩A) + n(A∩B∩C)
Example: In a class of 100 students, 35 like science and 45 like math. 10 like
both. How many students like either of them and how many like neither?

Solution:
Total number of students in a class, n(µ) = 100
Number of students who like science, n(S) = 35
Number of students who like Maths, n(M) = 45
Number of students who like both, n(M∩S) = 10
Number of students who like either of them,
n(MᴜS) = n(M) + n(S) – n(M∩S)
→ 45+35-10 = 70
Number of students who like neither = n(µ) – n(MᴜS) = 100 – 70 = 30
The easiest way to solve problems on sets is by drawing Venn diagrams, as shown
below.
µ=100

Math Science
45 10 35
427

12.1.2 Problem Set 12.1


1. There are 30 students in a class. Among them, 8 students are learning both
English and French. A total of 18 students are learning English. If every student is
learning at least one language, how many students are learning French in total?
2. If A = {1, 3, 5}, B = {3, 5, 6} and C = {1, 3, 7}
Verify that A ∪ (B ∩ C) = (A ∪ B) ∩ (A ∪ C)
3. From the data of Q2 Verify A ∩ (B ∪ C) = (A ∩ B) ∪ (A ∩ C)?
4. Let A = {𝑥 : 𝑥 is a natural number and a factor of 18} and B = {𝑥 : 𝑥 is a natural
number and less than 6}. Find A ∪ B.
5. Let A = {𝑥 : 𝑥 is a natural number and a factor of 18}

B = {𝑥 : 𝑥 is a natural number and less than 6}

Find A ∪ B and A ∩ B

6. If P = {multiples of 3 between 1 and 20} and Q = {even natural numbers up to


15}. Find the intersection of the two given set P and set Q.
7. A = {1, 2, 3} and B = {4, 5, 6}.
Find the difference between the two sets:
(i) A and B
(ii) B and A
8. If ξ = {1, 2, 3, 4, 5, 6, 7}

A = {1, 3, 7} find A'.

9. Write the cardinal number of each of the following sets:


(i)𝑥 = {letters in the word MALAYALAM}
428

(ii) Y = {5, 6, 6, 7, 11, 6, 13, 11, 8}


(iii) Z = {natural numbers between 20 and 50, which are divisible by 7}

10. From the adjoining Venn diagram, find the following sets.

(i) A B
4 2
3
(ii) B
5 6 9
(iii) A' 1

(iv) B' 8
10, 7
C
(v) C'

(vi) C - A

(vii) B - C

(viii) A - B

(ix) A ∪ B

(x) B ∪ C

(xi) A ∩ C

(xii) (B ∪ C)'

(xiii) (A ∩ B)'

S-ar putea să vă placă și